ITE Flashcards
What type of diabetes medications leads to hypoglycemia?
A. Biguanides
B. DPP-4 Inhibitors
C. SGLT-2 inhibitors
D. Sulfonylureas
E. Thiazolidinediones
D. Sulfonylureas like glipizide, glyburide and glimepiride
Biguanides= Metformin causeas diarrhea and lactic acidosis
DDP-4 Inhibitors: Saxagliptin, sitagliptin, linagliptin, alogliptin cause headache URI, elevated liver enzymes
SGLT2 inhibitors: canagliflozin, dapagliflozin, empagliflozin can cause UTI, candidiasis
Thiazolidinediones: Pioglitazone and rosiglitazone, weight gain, salt retention, edema
Vit D in new born
400 IU of vitamin D in infants
If breast feeding or partial formula
Exclusive formula may not need vit d
Scolosis progression
Magnitude of curve at presentation
Potential of future growth
Female sex
Cobb angle of 20
When titrating the dosage of opioids, the CDC recommends that you should consider prescribing naloxone when the opioid dosage reaches what morphine milligram equivalent (MME) per day. threshold?
A. 30
B. 50
C. 80
D. 90
E. 100
B. 50
A 62 y.o female with stage 3 chronic kidney disease and an estimated glomerular filtration rate of 37 mL/min/ 1.73 m2 is found to have a mildly low ionized calcium level. Which of the followign would you expect to see if her hypocalcemia is secondary to her chronic kidney disease?
A. Elevated PTH and Elevated phosphrous
B. Elevated PTH and low phosphorus
C. Low PTH and elevated phosphorus
D. Low PTH and low phosphorus
A. Elevated PTH and elevated phosphorus
In patients with CKD, phosphate is not appropriately excreted and the subsequent hyperphosphatemia leads to secondary hyperparathyroidism and binding of calcium.
Decreased production of calcitriol in patients with CKD also leads to hypocalcemic hyperparathyroidism.
When to evaluate sacral dimple
If small at anal verge without other skin or hair findings dont need evaluation to rule out spinal dysraphism (tethered cord)
> 0.5 cm diameter
within 2.5 cm of anal verge
Needs imaging
A 38 y.o female with a 6 month history of mild shortness of breath associated with some intermittent wheezing during upper respiratory infection presents for follow up. You previously prescribed albuterol (Proventil, Ventolin) via metered-dose inhaler, which she says helps her symptoms. You suspect asthma. PFT reveals a normal FEV1/ FVC ratio for her age. Which of the following would be the most appropriate next step?
A. Consider an alternative diagnosis
B. Assess her bronchodilator response
C. Perform methacholine challenge
D. Prescribe an inhaled corticosteroid
E. Proceed with treatment for COPD
C. Perform a methacholine challenge
Spirometry is central to confirming diagnosis of asthma. Normal FEV1/FVC, which doesnt rule in/out asthma.
A methacholine challenge is recommended in this scenario to assess for the airway hyper responsiveness that is the hallmark of asthma.
Bronchoconstriction (reduction in FEV1) observed at low levels of methacholine administration (<4 mg/mL) is consistent with asthma.
According to the most recent american college of cardiology/ American heart association guidelines, hypertension is defined as a blood pressure reading greater than
A. 120/80
B. 130/80
C. 135/85
D. 140/90
E. 150/90
B. 130/80
A 26-year-old G2P1001 at 30 weeks gestation was recently diagnosed with gestational diabetes and is ready to start testing her blood glucose at home. Which one of the following is the recommended goal for fasting blood glucose in this patient? (check one)
<75 mg/dL
<95 mg/dL
<120 mg/dL
<150 mg/dL
<180 mg/dL
The goal fasting blood glucose level in patients with gestational diabetes is <95 mg/dL. A fasting glucose
level <80 mg/dL is associated with increased maternal and fetal complications. The goal 2-hour
postprandial glucose level is <120 mg/dL and the goal 1-hour postprandial glucose level is <140 mg/dL.
A 75-year-old male sees you for evaluation of a unilateral resting tremor of his right hand. The tremor resolves if he is touched on the hand by someone. His wife notes that he seems to drag his feet now, but he has no history of falls.
Which one of the following has been shown to delay progression of his disease? (check one)
A. No currently available pharmacologic agents
B. Amantadine
C.Carbidopa/levodopa (Sinemet)
D. Rasagiline (Azilect)
E. Ropinirole
No treatment to delay only help symptoms
There are no currently available medications that have been shown to delay progression of Parkinson’s disease. However, guidelines recommend initiating the treatment of motor symptoms when they begin to affect the functions of daily life or decrease the quality of life. The first-line treatment for motor symptoms is carbidopa/levodopa due to its effectiveness for tremors, rigidity, and bradykinesia. It is a myth that delaying the use of levodopa will prevent a lack of efficacy later in the course of the illness, as what appears to be a lack of efficacy actually represents progression of the disease.
Amantadine can be used for patients under 65 years of age who are only experiencing tremors. Monoamine oxidase inhibitors such as rasagiline and non-ergot dopamine agonists such as ropinirole are not as effective as carbidopa/levodopa for motor symptoms, but they do not cause the dyskinesias and motor fluctuations seen with levodopa. Monoamine oxidase inhibitors are considered first-line therapy for patients under age 65 with mild motor symptoms.
Which one of the following is necessary to make the diagnosis of a functional gastrointestinal disorder? (check one)
A. Symptom-based clinical criteria
B. Noninvasive testing for Helicobacter pylori infection
C. Celiac serology
D. Gastric emptying studies
E. Esophagogastroduodenoscopy
In the absence of red-flag symptoms such as nocturnal defecation, weight loss, or gastrointestinal bleeding, functional gastrointestinal disorders can be diagnosed using symptom-based clinical criteria. Symptoms such as recurrent abdominal pain related to defecation, pain related to a change in the frequency of defecation, abdominal bloating and distension, and loose and watery or lumpy and hard stools are used to diagnose functional bowel disorders. Noninvasive testing for Helicobacter pylori, celiac serology, gastric emptying studies, and esophagogastroduodenoscopy are not required in order to make a diagnosis.
A 67-year-old male diagnosed with polymyalgia rheumatica is started on long-term prednisone therapy. Which one of the following is the recommended first-line agent to prevent steroid-induced osteoporosis? (check one)
Alendronate (Fosamax)
High-dose vitamin D
Raloxifene (Evista)
Teriparatide (Forteo)
Alendronate (Fosamax)
Supplementation of calcium (800–1000 mg) and vitamin D (400–800 IU) is also recommended.
A 54-year-old male develops chest pain while running. He is rushed to the emergency department of a hospital equipped for percutaneous coronary intervention. An EKG shows 3 mm of ST elevation in the anterior leads. He is diaphoretic and cool with ongoing chest pain. His blood pressure is 80/50 mm Hg, his pulse rate is 116 beats/min, and his oxygen saturation is 98% on room air.
You would immediately administer (check one)
A. a β-blocker
B. dual antiplatelet therapy and an anticoagulant
C. intravenous fibrinolytic therapy
D. an intravenous vasopressor
B. dual antiplatelet therapy and an anticoagulant
This patient is likely experiencing an acute anterior wall myocardial infarction with possible incipient
cardiogenic shock. Along with initiating the hospital’s protocol for myocardial infarction, immediate
treatment should include dual antiplatelet therapy with a 325-mg dose of nonenteric aspirin, a P2Y12
inhibitor (clopidogrel, prasugrel, or ticagrelor), and an anticoagulant (unfractionated heparin or
bivalirudin). Given the possibility of cardiogenic shock, -blockers should not be used. Unless more than
a 2-hour delay in percutaneous coronary intervention is expected, fibrinolytics should not be administered.
An intravenous vasopressor is not indicated.
A 28-year-old white female comes to your office at 37 weeks gestation with a 24-hour history of painful vesicles on the vulva. She does not have a past history of similar lesions. You make a presumptive diagnosis of genital herpes.
Of the following, the most sensitive and specific test is (check one)
exfoliative cytology (Tzanck test)
a polymerase chain reaction (PCR) test
an enzyme-linked immunosorbent assay (ELISA)
HSV serology (IgG/IgM)
PCR
When genital herpes occurs during pregnancy, the best method of diagnosis is either a tissue culture or a
polymerase chain reaction (PCR) test, which is more sensitive. Enzyme-linked immunosorbent assays are
sensitive, but not as sensitive or specific as PCR.
A 36-year-old male went skiing last year for the first time and when he made it to the top of the mountain he developed a headache, nausea, and dizziness, but no respiratory difficulty. That night he had difficulty sleeping. He asks for your recommendation on preventing a recurrence of the problem when he goes skiing again this year.
Which one of the following medications would you recommend he start the day before his ascent and continue until his descent is complete? (check one)
Acetazolamide (Diamox Sequels)
Aspirin
Dexamethasone (Decadron)
Tadalafil (Adcirca)
Zolpidem (Ambien)
Acetazolamide is the preferred agent for preventing acute mountain sickness (AMS). Multiple trials have
demonstrated its efficacy in preventing AMS. Dexamethasone is a first-line treatment for acute mountain
sickness of any severity but is a second-line drug for prevention because of its side-effect profile. Tadalafil
is advised as a second-line treatment after nifedipine for the prevention and treatment of high-altitude
pulmonary edema. Zolpidem may help with sleep but not AMS, and aspirin is not recommended for
prevention of AMS.
A 62-year-old male with diabetes mellitus recently underwent angioplasty with placement of a drug-eluting stent for the treatment of left main coronary artery disease and acute coronary syndrome. The patient is not considered at high risk for bleeding and you initiate dual antiplatelet therapy with aspirin and clopidogrel (Plavix).
For how long should this patient continue dual antiplatelet therapy? (check one)
1 month
3 months
6 months
9 months
At least 12 months
AT least 12 months
A 90-year-old male presents to the emergency department with chest pain, dyspnea, and diaphoresis. He has experienced these symptoms intermittently since his wife died last week. An EKG shows ST elevation in the anterior leads, and cardiac enzymes are elevated. An echocardiogram shows apical ballooning of the left ventricle. Cardiac catheterization does not reveal coronary vascular disease. You plan to discharge the patient after observation overnight.
Which one of the following would be the most appropriate management of this patient’s stress-induced (Takotsubo) cardiomyopathy after discharge? (check one)
Home medications only
A cardiac event monitor to detect any rhythm abnormalities
A diuretic, ACE inhibitor, and β-blocker until his symptoms and the abnormalities seen on the echocardiogram resolve
A statin, diuretic, ACE inhibitor, and β-blocker to be continued indefinitely
Pacemaker placement
Takotsubo cardiomyopathy, also known as stress-induced cardiomyopathy, can develop following
emotional distress and is characterized by the abrupt onset of dysfunction of the left ventricle. The clinical
presentation and laboratory studies can mirror acute coronary syndrome and should be treated similarly.
Once symptoms and cardiac abnormalities resolve, treatment is no longer indicated and may be withdrawn
if there are no signs of coronary disease. Because this patient currently has cardiomyopathic abnormalities,
a diuretic, ACE inhibitor, and -blocker are indicated. Ambulatory cardiac monitors are not indicated for
this patient with a known diagnosis of Takotsubo cardiomyopathy. A pacemaker is not indicated in the
absence of arrhythmias caused by conduction abnormalities.
A 55-year-old patient with a history of alcoholism is admitted through the emergency department with acute pancreatitis. Which one of the following tests performed at the time of admission can best predict the severity of pancreatitis? (check one)
Hematocrit
C-reactive protein
Serum amylase
Serum lipase
CT of the abdomen
Hematocrit
Knowing the severity of pancreatitis helps predict how aggressive management should be. Hematocrit,
BUN, and creatinine levels are the most useful predictors of the severity of pancreatitis, reflecting the
degree of intravascular volume depletion. C-reactive protein is often elevated, but it is not as useful as
hematocrit for predicting severity. Serum amylase and lipase have no prognostic value. CT evidence of
severe pancreatitis lags behind clinical and laboratory evidence, and early CT underestimates the severity
of the acute process.
A 30-year-old gravida 3 para 2 sees you for prenatal care at 13 weeks gestation. During her previous pregnancies she became hypertensive and had bilateral leg edema and proteinuria. These conditions resolved after delivery. Her only current medication is a prenatal vitamin.
In order to prevent this condition, which one of the following should be started today? (check one)
No new medications
Aspirin
Fish oil
Magnesium
Vitamin C
Preeclampsia take Aspirin
Aspirin, 81 mg daily, is recommended for high-risk pregnant patients to prevent preeclampsia. Prophylaxis should begin after 12 weeks gestation and continue until delivery. Fish oil, magnesium, and vitamin C are not beneficial in the prevention of preeclampsia.
A 34-year-old male with sickle cell disease has a new onset of mild to moderate thirst and polyuria. He ate a large meal about 2 hours ago.
An examination reveals a BMI of 32 kg/m2. Results of a urinalysis performed by your staff include 3+ glucose and no ketones. His blood glucose level is 288 mg/dL and his hemoglobin A1c is 5.2%.
Which one of the following would be most appropriate at this point to help diagnose and monitor this patient’s glycemic control? (check one)
A serum fructosamine level
A repeat hemoglobin A1c
A 2-hour glucose tolerance test
Hemoglobin electrophoresis
Referral to an endocrinologist
This patient with sickle cell disease has a new onset of diabetes mellitus. Hemoglobinopathies falsely lower
hemoglobin A1c as a result of hemolysis and abnormal glycation. Fructosamine correlates well with
hemoglobin A1c levels and is recommended instead of hemoglobin A1c for monitoring glucose control in
patients with diabetes and hemoglobinopathies. A 2-hour glucose tolerance test or hemoglobin
electrophoresis would not provide useful information. Referral to an endocrinologist is not indicated at this
point because the patient has not failed primary care management.
Premature adrenache
Laboratory studies and radiography warrant consideration if the
patient develops secondary sex characteristics before the age of 8, or if her height velocity increases
rapidly during the surveillance period.
An otherwise asymptomatic 7-year-old male has a blood pressure above the 95th percentile for gender, age, and height on serial measurements. Which one of the following studies would be most appropriate at this time? (check one)
Renin and aldosterone levels
24-hour urinary fractionated metanephrines and normetanephrines
Renal ultrasonography
Doppler ultrasonography of the renal arteries
A sleep study
Renal US
Renal parenchymal diseases such as glomerulonephritis, congenital abnormalities, and reflux nephropathy
are the most common cause of hypertension in preadolescent children. Preadolescent children with
hypertension should be evaluated for possible secondary causes and renal ultrasonography should be the
first choice of imaging in this age group.
Renin and aldosterone levels are indicated if there is a reason to suspect primary hyperaldosteronism, such
as unexplained hypokalemia. Measurement of 24-hour urinary fractionated metanephrines and
normetanephrines is used to diagnose pheochromocytomas, which are rare and usually present with a triad
of symptoms including headache, palpitations, and sweating. Doppler ultrasonography of the renal arteries
is useful for diagnosing renal artery stenosis, which should be suspected in patients with coronary or
peripheral atherosclerosis or young adults, especially women 19–39 years of age, who are more at risk for
renal artery stenosis due to fibromuscular dysplasia. Sleep studies are indicated in patients who are obese
or have signs or symptoms of obstructive sleep apnea.
A 30-year-old female presents with pain over the proximal fifth metatarsal after twisting her ankle. Radiographs reveal a nondisplaced tuberosity avulsion fracture of the fifth metatarsal.
Which one of the following would be the most appropriate initial management? (check one)
A short leg walking boot
A compressive dressing with weight bearing and range-of-motion exercises as tolerated
A posterior splint with no weight bearing, and follow-up in 3–5 days
A short leg cast with no weight bearing
Surgical fixation
The fifth metatarsal has the least cortical thickness of all of the metatarsals. There are strong ligaments and
capsular attachments on the proximal fifth metatarsal that can put significant stress on this area of the bone,
leading to fractures. Nondisplaced tuberosity fractures can generally be treated with compressive dressings
such as an Aircast or Ace bandage, with weight bearing and range-of-motion exercises as tolerated.
Minimally displaced (<3 mm) avulsion fractures of the fifth metatarsal tuberosity can be treated with a
short leg walking boot. If the displacement is >3 mm, an orthopedic referral is warranted.
Montelukast (Singulair) has an FDA boxed warning related to an increased risk of: (check one)
delirium
myocardial infarction
suicidality
venous thromboembolism
In March 2020, the FDA upgraded its warning label for montelukast to a boxed warning (black box warning) based on the trends for all neuropsychiatric adverse events, including suicidality, associated with montelukast use reported in the FDA Adverse Event Reporting System database from the date of FDA approval in February 1998 through May 2019 (SOR B). The boxed warning does not indicate an increased risk of delirium, myocardial infarction, or venous thromboembolism
A 25-year-old female who is 3 months post partum presents with multiple complaints, including
increasing weakness and fatigue, intolerance to warm environments, a weight loss of 30 lb
despite an increased appetite, difficulty sleeping, awareness that her heart is beating faster and
“pounding” in her chest, increasing restlessness and difficulty concentrating, increased
tremulousness, and a significant swelling in her neck. She takes no medication, has experienced
no recent trauma, and has not ingested large amounts of iodine.
When you examine her you find no exophthalmos or lid lag and no pretibial edema, but her skin
is warm, smooth, and moist. You also find a smooth, non-nodular, nontender, enlarged thyroid
gland, clear lungs, a resting tremor, and hyperactive reflexes.
Laboratory testing reveals a low TSH level, elevated free T3 and free T4, and high uptake on a
radioactive iodine uptake scan.
Which one of the following is the most likely diagnosis? (check one)
Postpartum thyroiditis
Silent thyroiditis
Subacute thyroiditis
Graves disease
Exogenous thyroid ingestion
This patient has symptoms consistent with hyperthyroidism, which could be caused by any of the options
listed. TSH is suppressed and free T4 and free T3 are elevated in all of these conditions.
**Only Graves
disease, however, will cause high radioactive iodine uptake on a thyroid scan. **
Uptake will be low in the
other conditions.
Which one of the following medications for the treatment of type 2 diabetes has been associated with ketoacidosis? (check one)
Dapagliflozin (Farxiga)
Liraglutide (Victoza)
Metformin
Pioglitazone (Actos)
Sitagliptin (Januvia)
Dapagliflozin (Farxiga)
SGLT2 inhibitors such as dapagliflozin have increasingly been shown to be associated with diabetic ketoacidosis under certain circumstances. Liraglutide, metformin, pioglitazone, and sitagliptin are not associated with diabetic ketoacidosis.
Three weeks after he had knee surgery, a 64-year-old male presents for follow-up of an emergency department visit for a pulmonary embolism. He has no previous history of pulmonary embolism and is otherwise in good health. He is being treated with apixaban (Eliquis).
The recommended duration of anticoagulation therapy for this patient is
(check one)
1 month
3 months
6 months
9 months
12 months
3 months
Patients who have a venous thromboembolism (VTE) require anticoagulation therapy for treatment and
prevention of recurrence. The risk of recurrence is greatest in the first year after the event and remains
elevated indefinitely. The risk for VTE recurrence is dependent on patient factors, such as active cancers
and thrombophilia. Current guidelines recommend treatment for at least 3 months. In patients who have
a reversible provoking factor such as surgery, anticoagulation beyond 3 months is not recommended.
Ref: Wilbur J, Shian B: Deep venous thrombosis and pulmonary embolism: Current therapy. Am Fam Physician
2017;95(5):295-302.
A 30-year-old gravida 2 para 1 in her second trimester is evaluated for hypothyroidism. The normal TSH range in pregnancy is (check one)
lower than in the nonpregnant state
higher than in the nonpregnant state
the same as in the nonpregnant state
not useful for evaluating hypothyroidism after the first trimester
Lower
The TSH reference range is lower during pregnancy because of the cross-reactivity of the -subunit of
hCG. Levels of hCG peak during weeks 7–13 of pregnancy, and hCG has mild TSH-like activity, leading
to slightly high free T4 levels in early pregnancy. This leads to a feedback decrease in TSH.
Ottawa Knee rules
Several decision support tools can help guide the decision to order imaging of an injured knee, such as the
Ottawa Knee Rule, the Pittsburgh Knee Rule, and American College of Radiology (ACR) criteria.
The inability to take four or more steps immediately after an injury or in the emergency setting is an indication
for radiography in all three rules.
Age is an indication for radiography in acute knee pain in patients over 55 years of age according to the
Ottawa rule, or under 12 or over 50 years of age according to the Pittsburgh rule. The patient’s sex does
not factor into the criteria for imaging.
Bony tenderness is an indication for imaging according to the ACR and Ottawa rules, but only if isolated
over the proximal fibula or over the patella without other bony tenderness. The inability to flex the knee
to 90° is also an indication for imaging according to the ACR and Ottawa rules.
A 32-year-old female presents with heat intolerance, excessive weight loss, and anxiety. She gave birth 6 months ago and recently stopped breastfeeding. On examination her thyroid gland is slightly diffusely enlarged and nontender. Laboratory studies reveal a decreased TSH level and elevated free T3 and T4 levels. You suspect that she has postpartum thyroiditis.
Which one of the following tests would be most useful to confirm the diagnosis? (check one)
Radioactive iodine uptake
Thyroid peroxidase antibody levels
Thyroid ultrasonography
Thyrotropin receptor antibody levels
Radioactive iodine uptake
Postpartum thyroiditis is defined as a transient or persistent thyroid dysfunction that occurs within 1 year
of childbirth, miscarriage, or medical abortion. Release of preformed thyroid hormone in the bloodstream
initially results in hyperthyroidism. During the hyperthyroid phase, radioactive iodine uptake will be low,
which can help to confirm the diagnosis. Pregnancy and breastfeeding are contraindications to radionuclide
imaging. Thyroid peroxidase antibody levels are elevated with chronic autoimmune thyroiditis
(Hashimoto’s thyroiditis), and patients present with symptoms of hypothyroidism. The Endocrine Society and American Association of Clinical Endocrinologists do not recommend routine thyroid ultrasonography
in patients with abnormal thyroid function tests if there is no palpable abnormality of the thyroid gland.
Thyrotropin receptor antibody levels are positive in Graves disease.
A 15-year-old male presents with a 2-day history of dark-colored urine, lower extremity edema, and fatigue. Approximately 2 weeks ago he said he had a “bad sore throat” that was treated empirically with amoxicillin. On examination his blood pressure is 144/92 mm Hg, his pulse rate is 76 beats/min, and his other vital signs are normal. Other than mild dependent edema there are no additional significant physical examination findings. A urinalysis dipstick shows 3+ hematuria.
Which one of the following findings on microscopic evaluation of the urine sediment would help to confirm the diagnosis in this patient? (check one)
Gram-positive cocci in chains
RBC casts
WBC casts
Eosinophils
Oxalate crystals
This is a classic presentation for acute poststreptococcal glomerulonephritis (APSGN), with the onset of
gross hematuria associated with hypertension and systemic edema. This is most commonly seen in
school-age children, usually 1–2 weeks after an episode of pharyngitis or 3–4 weeks after an episode of
impetigo, caused by so-called nephritogenic strains of Group A -hemolytic Streptococcus. The hematuria
is caused by immune complex–mediated glomerular injury.
Bacteriuria may be seen in both upper and lower urinary tract infections, but may also be a spurious
finding, especially with the combined presence of epithelial cells. The classic finding on microscopic
urinalysis for acute glomerulonephritis is the presence of RBC casts. WBC casts are seen with acute
pyelonephritis. The presence of urinary eosinophils indicates acute interstitial nephritis. Calcium oxalate
makes up the most common type of kidney stones.
Antibiotics prescribed for antecedent pharyngitis do not prevent APSGN. Treatment is supportive,
controlling blood pressure and edema with a thiazide or a loop diuretic. The prognosis for resolution and
full recovery of the vast majority of patients with APSGN is excellent, especially in the pediatric age
group.
A patient comes to your outpatient clinic with a persistent migraine that she has been unable to treat effectively at home. The symptoms began several hours ago and are typical for her. She has already tried her usual treatments of ibuprofen, 800 mg, and rizatriptan (Maxalt), 10 mg, but they have not provided any relief. She took a second dose of rizatriptan 2 hours later without benefit. She is in significant pain, which is causing mild nausea, and she has photophobia and phonophobia.
Which one of the following would be most appropriate at this point? (check one)
Oral butalbital/acetaminophen/caffeine (Fioricet)
Oral ergotamine/caffeine (Cafergot)
Subcutaneous sumatriptan (Imitrex)
Intramuscular morphine
Intramuscular prochlorperazine
Multiple studies have determined that parenteral antiemetics have benefits for the treatment of acute
migraine beyond their effect on nausea. Most outpatient clinics do not have the ability to administer
intravenous metoclopramide, which is the preferred treatment. However, most clinics do have the ability
to administer intramuscular prochlorperazine or promethazine. Due to concerns about oversedation,
misuse, and rebound, treatment with parenteral opiates is discouraged but may be an option if other
treatments fail. Oral butalbital/acetaminophen/caffeine and oral ergotamine/caffeine have less evidence of
success in the treatment of acute migraine. Sumatriptan is contraindicated within 24 hours of the use of
rizatriptan.
A 72-year-old female presents for a routine health maintenance visit. Which one of the following medications in her current regimen places her at risk for osteoporosis? (check one)
Atorvastatin (Lipitor)
Hydrochlorothiazide
Metformin (Glucophage)
Phenytoin (Dilantin)
Ranitidine (Zantac)
Medications reported to be associated with osteoporosis and increased fracture risk include antiepileptic
drugs, long-term heparin, cyclosporine, tacrolimus, aromatase inhibitors, glucocorticoids,
gonadotropin-releasing hormone agonists, thiazolidinediones, excessive doses of levothyroxine, proton
pump inhibitors, SSRIs, parenteral nutrients, medroxyprogesterone contraceptives, methotrexate, and
aluminum antacids. Atorvastatin, hydrochlorothiazide, metformin, and ranitidine are not associated with
osteoporosis.
A 42-year-old bricklayer was diagnosed with acute bronchitis at an urgent care center 6–7 weeks ago. A chest radiograph was negative for pneumonia but revealed a solitary pulmonary nodule. No previous chest radiograph was available, so a follow-up chest radiograph was ordered for 4–6 weeks after the initial one. He is following up with you today to review those results.
The patient’s bronchitis has since resolved, and he feels well. He has no significant past medical history and does not take any medications. He has no constitutional symptoms and a physical examination today is within normal limits. He has a 10-pack-year cigarette smoking history and quit 15 years ago.
If a lesion is noted on the chest radiograph, which one of the following characteristics would be most suspicious for malignancy? (check one)
A diameter of 5 mm
Concentric calcifications
Doubling in size in less than 1 month
A nonsolid “ground glass” appearance
Smooth borders
Characteristics that are more commonly associated with malignant lesions include a nonsolid “ground glass” appearance, a size >6 mm, noncalcified lesions, a lesion size or volume doubling time between 1 month and 1 year, and irregular or spiculated borders. Findings on a chest radiograph that are more commonly associated with benign lesions include a lesion size <6 mm, concentric or “popcorn-like”
calcifications, doubling times of <1 month or >2–4 years, dense solid-appearing lesions, and lesions with smooth regular borders. Other diagnostic imaging modalities are also utilized, including CT and PET, and a biopsy is sometimes necessary to establish the diagnosis. Chest radiographs are still useful for monitoring
patients with multiple findings that correlate most often with benign lesions. Informed decision making by the patient and family physician can sensibly guide the follow-up of patients with solitary pulmonary nodules without automatically referring them to specialists or ordering the most sophisticated imaging
You are the team physician for the local high school track team. During a meet one of the athletes inadvertently steps off the edge of the track and inverts her right foot forcefully. She is able to bear weight but with significant pain. She reports pain across her right midfoot. An examination reveals edema over the lateral malleolus and diffuse tenderness, but she does not have any pain with palpation of the navicular, the base of the fifth metatarsal, or the posterior distal lateral and medial malleoli.
Which one of the following would be most appropriate at this time?
A) Radiographs of the right ankle only
B) Radiographs of the right foot only
C) Radiographs of the right foot and ankle
D) Lace-up ankle support, ice, compression, and clinical follow-up
E) Crutches and no weight bearing for 2 weeks, followed by a slow return to weight
bearing
ANSWER: D
The Ottawa foot and ankle rules should be used to determine the need for radiographs in foot and ankle injuries. A radiograph of the ankle is recommended if there is pain in the malleolar zone along with the inability to bear weight for at least four steps immediately after the injury and in the physician’s office or emergency department (ED), or tenderness at the tip of the posterior medial or lateral malleolus. A radiograph of the foot is recommended if there is pain in the midfoot zone along with the inability to bear weight for four steps immediately after the injury and in the physician’s office or ED, or tenderness at the base of the fifth metatarsal or over the navicular bone. The Ottawa foot and ankle rules are up to 99% sensitive for detecting fractures, although they are not highly specific. In this case there are no findings that would require radiographs, so treatment for the ankle sprain would be recommended. Compression combined with lace-up ankle support or an air cast, along with cryotherapy, is recommended and can increase mobility. Early mobilization, including weight bearing as tolerated for daily activities, is associated with better long-term outcomes than prolonged rest.
Increases risk of afib
Alcohol
A 10-year-old male has an 8-mm induration 2 days after a tuberculin skin test. He shares a bedroom with his 18-year-old brother who was recently diagnosed with tuberculosis. There are no other historical or physical examination findings to suggest active tuberculosis infection and a chest radiograph is normal.
Which one of the following would be most appropriate at this point?
A) Monitoring with annual tuberculin skin testing
B) Observation and repeat tuberculin skin testing in 3 weeks
C) Rifampin (Rifadin) daily for 4 months
D) Isoniazid daily for 9 months
E) Once-weekly isoniazid and rifampin for 3 months
D. Isoniazid for 9 months
For children age 2–11 years, treatment with isoniazid, 10–20 mg/kg daily or 20–40 mg/kg twice weekly for 9 months, is the preferred and most efficacious treatment regimen. The shorter 6-month treatment course is considered an acceptable option for adults, but it is not recommended for children. The use of rifampin alone or in combination with isoniazid is also an acceptable option for adults but not for children under the age of 12.
> 5 mm for exposed
10 for risk factors
15 mm for no risks
A 28-year-old female presents with a 3-month history of fatigue and postural lightheadedness. On examination she is diffusely hyperpigmented, especially her skin creases and areolae. A CBC and basic metabolic panel are normal except for an elevated potassium level. You order a corticotropin stimulation test.
Prior to the corticotropin injection, you should order which one of the following tests to confirm that this patient has a primary insufficiency and not a secondary (pituitary) disorder?
A) ACTH
B) Aldosterone
C) Melanocyte-stimulating hormone
D) Renin
E) TSH
A
A plasma ACTH level is recommended to establish primary adrenal insufficiency. The sample can be obtained at the same time as the baseline sample in the corticotropin test. A plasma ACTH greater than twice the upper limit of the reference range is consistent with primary adrenal insufficiency. Aldosterone and renin levels should be obtained to establish the presence of adrenocortical insufficiency, but these do not differentiate primary from secondary adrenal insufficiency. The hyperpigmentation of Addison’s disease is caused by the melanocyte-stimulating hormone (MSH)–like effect of the elevated plasma levels of ACTH. ACTH shares some amino acids with MSH and also produces an increase in MSH in the blood. TSH is not part of the feedback loop of adrenal insufficiency.
A 32-year-old female requests a physical examination prior to participating in an adult soccer league. Her blood pressure is 118/70 mm Hg and her pulse rate is 68 beats/min. The examination is otherwise normal except for a systolic murmur that intensifies with Valsalva maneuvers. She says that she has recently been experiencing mild exertional dyspnea and moderate chest pain. The chest pain has been atypical and is not necessarily related to exertion. Echocardiography reveals hypertrophic cardiomyopathy.
In addition to referring the patient to a cardiologist, you recommended initiating therapy with
A) amiodarone (Cordarone)
B) amlodipine (Norvasc)
C) furosemide (Lasix)
D) lisinopril (Prinivil, Zestril)
E) metoprolol
E
Hypertrophic cardiomyopathy is the most common primary cardiomyopathy, with a prevalence of 1:500 persons. Many patients with hypertrophic cardiomyopathy are asymptomatic and are diagnosed during family screening, by auscultation of a heart murmur, or incidentally after an abnormal result on electrocardiography. On examination physicians may hear a systolic murmur that increases in intensity during Valsalva maneuvers. The main goals of therapy are to decrease exertional dyspnea and chest pain and prevent sudden cardiac death. -Blockers are the initial therapy for patients with symptomatic hypertrophic cardiomyopathy. Nondihydropyridine calcium channel blockers such as verapamil can be used if -blockers are not well tolerated.
High intensity statin
high-intensity statin regimens if tolerated. These regimens include atorvastatin, 40–80 mg/day, and rosuvastatin, 20–40 mg/day.
A 62-year-old female who is a new patient requests a thyroid evaluation because she has a history of abnormal thyroid test results. You obtain a copy of her records, which include a TSH level of 0.2 U/mL (N 0.4–4.2) and a free T4 level of 2.0 ng/dL (N 0.8–2.7) from 3 years ago. She reports feeling well and has no other health conditions. She does not take any medications.
A physical examination reveals normal vital signs, a BMI of 23.0 kg/m2, no neck masses, a normal thyroid size, and normal heart sounds. Laboratory studies reveal a TSH level of 0.1 U/mL, a free T4 level of 2.5 ng/dL, and a free T3 level of 3.1 pg/mL (N 2.3–4.2).
Treatment for this condition would be indicated if the patient has an abnormal
A) calcium level
B) DXA scan
C) glucose level
D) lipid level
E) thyroid ultrasonography study
ANSWER: B
This patient has subclinical hyperthyroidism as evidenced by her low TSH level with normal free T4 and free T3 levels. Common causes of subclinical hyperthyroidism include Graves disease, autonomous functioning thyroid adenoma, and multinodular toxic goiter. Subclinical hyperthyroidism may progress to overt hyperthyroidism; this is more likely in patients with TSH levels <0.1 U/mL. Even in the absence of overt hyperthyroidism these patients are at higher risk for several health conditions, including atrial fibrillation, heart failure, and osteoporosis. For this reason it is important to assess for these conditions and consider treating the underlying thyroid condition, as well as the complication. The American Thyroid Association recommends treating patients with complications who are either over age 65 or have a TSH level <0.1 U/mL.
Lipid and glucose abnormalities are not known to be related to subclinical hyperthyroidism. Calcium levels may be abnormal in hyperparathyroidism but not hyperthyroidism. Thyroid ultrasonography may be helpful to determine the cause of hyperthyroidism but is not used to help decide when to treat subclinical hyperthyroidism.
A 7-year-old female with asthma is brought to your office because of her fourth episode of wheezing in the last 3 months. She has also had to use her short-acting -agonist rescue inhaler more frequently.
Which one of the following should be added to reduce the frequency of asthma exacerbations?
A) A leukotriene receptor antagonist
B) A long-acting -agonist
C) An inhaled corticosteroid
D) Inhaled cromolyn via nebulizer
ANSWER: C
Pediatric asthma is the most commonly encountered chronic illness, occurring in nearly one out of seven individuals. Short-acting -agonists in the form of metered-dose inhalers are clearly favored for acute exacerbations, as well as for intermittent asthma. Treatment for persistent asthma requires the use of inhaled corticosteroids, with short-acting -agonists used for exacerbations. For patients not well controlled with those options, either a long-acting -agonist or a leukotriene receptor antagonist may be added. While both cromolyn and nedocromil are fairly devoid of adverse effects, their use is limited because of a lack of efficacy in the prevention of acute asthma exacerbations.
Woods lamp colors
Corynebacterium minutissimum infection, and use of an ultraviolet light would reveal a coral pink color. Pale blue fluorescence occurs with Pseudomonas infections, yellow with tinea infections, and totally white with vitiligo. A lime green fluorescence is not characteristic of a particular skin condition.
Hypertensive emergency
> 180 or > 110
when to add insulin
A1C >9
Which one of the following antihypertensive drugs may reduce the severity of sleep apnea?
A) Amlodipine (Norvasc)
B) Hydralazine
C) Lisinopril (Prinivil, Zestril)
D) Metoprolol
E) Spironolactone (Aldactone)
Spironolactone
Diuretics lessen the severity of obstructive sleep apnea and reduce blood pressure. Aldosterone antagonists offer further benefit beyond that of traditional diuretics. Resistant hypertension is common in patients with obstructive sleep apnea. Resistant hypertension is also associated with higher levels of aldosterone, which can lead to secondary pharyngeal edema, increasing upper airway obstruction.
Which one of the following malignancies is associated with hereditary hemochromatosis?
A) Biliary carcinoma
B) Chronic myeloid leukemia
C) Hepatocellular carcinoma
D) Multiple myeloma
E) Pancreatic cancer
C
Hereditary hemochromatosis is a genetic disorder of iron regulation and subsequent iron overload. Possible end-organ damage includes cardiomyopathy, cirrhosis of the liver, and hepatocellular carcinoma. Symptoms are often nonspecific early on, but manifestations of iron overload eventually occur. The diagnosis should be suspected in patients with liver disease or abnormal iron studies indicative of iron overload. A liver biopsy can confirm the diagnosis and the degree of fibrosis. Identification of such patients and proper ongoing treatment with phlebotomy may prevent the development of hepatocellular carcinoma and other complications of this disease. There is some data that suggests an association of breast cancer with hereditary hemochromatosis but not with any of the other malignancies listed.
You admit a previously healthy 62-year-old female to the hospital for intractable nausea and vomiting with intravascular volume depletion and hypotension. She lives in rural northern New Mexico. Prior to the onset of her symptoms she had been gardening and cleaning out a chicken coop, where she encountered several rodents. She is febrile and you obtain blood and urine cultures. Two out of four blood culture bottles are positive for gram-negative rods.
Which one of the following is the most likely pathogen?
A) Brucella melitensis
B) Coxiella burnetii
C) Escherichia coli
D) Listeria monocytogenes
E) Yersinia pestis
Yersinia pestis is an aerobic fermentative gram-negative rod. It causes a zoonotic infection with humans as the accidental host. The disease is spread by a bite from a flea vector, direct contact with infected tissue, or inhalation of infectious aerosols from a person with pulmonary plague. Plague occurs in two regions in the western United States. One region includes northern New Mexico, northern Arizona, and southern Colorado, and the other region includes California, southern Oregon, and far western Nevada.
Escherichia coli is also an aerobic fermentative gram-negative rod but it generally causes symptoms of gastroenteritis, hemolytic-uremic syndrome, urinary tract infection, intra-abdominal infection, and meningitis. E. coli infection does not have a specific regional distribution. Listeria monocytogenes is a gram-positive rod and causes an influenza-like illness with or without gastroenteritis in adults. Infection occurs through ingestion of contaminated food products such as milk, cheese, processed meats, and raw vegetables. Outbreaks can occur in any geographic distribution.
25
Coxiella burnetii is a gram-negative intracellular bacterium that causes Q fever. Human infections are associated with contact with infected cattle, sheep, goats, dogs, and cats. Brucella melitensis is a gram-negative coccobacilli that causes brucellosis. Humans are accidental hosts who can develop the disease from contact with tissues rich in erythritol, and from shedding of organisms in milk, urine, and birth products from goats and sheep.
A 21-year-old female is being evaluated for secondary causes of refractory hypertension. Which one of the following would be most specific for fibromuscular dysplasia?
A) A serum creatinine level
B) An aldosterone:renin ratio
C) 24-hour urine for metanephrines
D) Renal ultrasonography
E) Magnetic resonance angiography of the renal arteries
ANSWER: E
In young adults diagnosed with secondary hypertension, evaluation for fibromuscular dysplasia of the renal arteries with MR angiography or CT angiography is indicated (SOR C). The aldosterone/renin ratio is the most sensitive test to diagnose primary hyperaldosteronism. Renal ultrasonography is an indirect test that is not as sensitive or specific for fibromuscular dysplasia. Serum creatinine elevation shows renal involvement but does not identify the cause. Testing for metanephrines is indicated only if a pheochromocytoma is suspected.
Which one of the following diabetes mellitus medications is MOST likely to cause weight gain?
A) Empagliflozin (Jardiance)
B) Glimepiride (Amaryl)
C) Liraglutide (Victoza)
D) Metformin (Glucophage)
E) Sitagliptin (Januvia)
NSWER: B
Since many patients with diabetes mellitus are obese, the impact of medications on the patient’s weight is important to consider. Treatment with sulfonylureas, including glimepiride, is associated with weight gain. Empagliflozin, liraglutide, metformin, and sitagliptin are not associated with weight gain. In particular, the SGLT2 inhibitors such as empagliflozin and the GLP1 agonists such as liraglutide are associated with clinically significant weight loss.
A 45-year-old male presents to your office with a 2-month history of a nonproductive cough, mild shortness of breath, fatigue, and a 5-lb weight loss. On examination his lungs are clear. A PPD skin test is negative. A chest radiograph shows bilateral hilar adenopathy and his angiotensin converting enzyme level is elevated. A biopsy of the lymph node shows a noncaseating granuloma.
Which one of the following would be the most appropriate initial treatment?
A) Azathioprine (Imuran)
B) Fluconazole (Diflucan)
C) Isoniazid
D) Levofloxacin (Levaquin)
E) Prednisone
ANSWER: E
This patient has sarcoidosis that has been confirmed by a biopsy. He is symptomatic so treatment would be indicated. The recommended initial treatment for sarcoidosis is oral corticosteroids. Anti-infective agents are not appropriate treatment for sarcoidosis. Immunosuppressants are second- and third-line therapy for sarcoidosis and would not be recommended as first-line treatment.
Tdap at what week
27-36
A 25-year-old gravida 1 para 0 at 24 weeks gestation comes to your office with right lower extremity swelling and pain. Her pregnancy has been uncomplicated so far and her only medication is a prenatal vitamin. She does not have chest pain, shortness of breath, or fever. She recently started feeling the baby move, and an anatomy scan at 20 weeks gestation was normal.
Lower extremity Doppler ultrasonography confirms a right lower extremity deep vein thrombosis (DVT). Laboratory studies including a CBC, coagulation studies, and renal function are normal.
Which one of the following would be the most appropriate initial treatment of her DVT?
A) Oral apixaban (Eliquis)
B) Oral aspirin
C) Oral warfarin (Coumadin)
D) Subcutaneous enoxaparin (Lovenox)
E) Subcutaneous heparin
ANSWER: D
Enoxaparin is the most appropriate pharmacologic therapy for anticoagulation in patients who are pregnant. Aspirin is not used as treatment for deep vein thrombosis. Apixaban, warfarin, and heparin either have not been studied for use in pregnancy or there is data indicating potential fetal harm.
A 67-year-old female who was recently diagnosed with colon cancer presented to the emergency department 2 days ago with acute shortness of breath and was diagnosed with a pulmonary embolism. She was started on enoxaparin (Lovenox) and was hemodynamically stable during her stay in the hospital. Her shortness of breath has improved and her oxygen saturation is currently 95% on room air.
Which one of the following would be most appropriate for this patient?
A) Continue enoxaparin upon discharge
B) Discontinue enoxaparin and start rivaroxaban (Xarelto)
C) Discontinue enoxaparin and start warfarin (Coumadin)
D) Start warfarin and continue enoxaparin until the INR is 2.0
ANSWER: A
Enoxaparin and other low molecular weight heparins are effective and are the preferred agents for acute and long-term anticoagulation in patients with an active malignancy (SOR B). Warfarin has been shown to be less effective in cancer patients and is not recommended to treat venous thromboembolic disease in this setting (SOR B). The novel oral anticoagulants including rivaroxaban have not been studied in the setting of malignancy and are not recommended.
A 67-year-old female with hypertension and atrial fibrillation has been taking warfarin (Coumadin) for the past 10 years. She has been hemodynamically stable for many years with no complications from her atrial fibrillation. She is scheduled to undergo elective bladder sling surgery for urinary incontinence. She does not have any other significant past medical history.
Which one of the following would be the most appropriate perioperative management of her warfarin?
A) Continue warfarin without interruption
B) Discontinue warfarin the day prior to surgery and provide bridge therapy with low
molecular weight heparin
C) Discontinue warfarin 2 days prior to surgery and restart it 2 days postoperatively
unless there is a bleeding complication
D) Discontinue warfarin 2 days prior to surgery and restart it 5 days postoperatively
unless there is a bleeding complication
E) Discontinue warfarin 5 days prior to surgery and restart it 12–24 hours
postoperatively unless there is a bleeding complication
ANSWER: E
Perioperative management of chronic anticoagulation requires an assessment of the patient’s risk for thromboembolism and the risk of bleeding from the surgical procedure. High-risk patients include those with mechanical heart valves, a stroke or TIA within the past 3 months, venous thromboembolism within the past 3 months, or coronary stenting within the previous 12 months. High-risk patients require bridging therapy with low molecular weight heparin, while patients at low risk do not require bridging anticoagulation. For low-risk patients, it is recommended that warfarin be discontinued 5 days prior to surgery and restarted 12–24 hours postoperatively. This patient is at low risk for thromboembolism because her CHA2DS2-VASc score is 3. A patient with atrial fibrillation should receive bridging therapy with a CHA2DS2-VASc score 6. This patient’s surgery is associated with a high risk for bleeding, so it is preferable to stop her warfarin 5 days before the operation.
A 27-year-old female with a past medical history of polycystic ovary syndrome (PCOS) would like to become pregnant. Which one of the following treatments for PCOS is associated with greater live-birth and ovulation rates?
A) Finasteride (Proscar)
B) Letrozole (Femara)
C) Metformin (Glucophage)
D) Spironolactone (Aldactone)
ANSWER: B
In a double-blind randomized trial, letrozole was associated with greater live-birth and ovulation rates compared to clomiphene (SOR A). A Cochrane review indicated that metformin does not increase fertility in patients diagnosed with polycystic ovary syndrome (PCOS). Spironolactone and finasteride are both used to treat PCOS in women who do not desire pregnancy.
A 58-year-old male with a history of tobacco and alcohol abuse presents with the sudden onset of many well circumscribed brown, oval, rough papules with a “stuck-on” appearance on his trunk and proximal extremities. On examination you also note an unintentional 6-kg (13-lb) weight loss over the last 3 months and conjunctival pallor. A review of systems is positive for more frequent stomachaches, decreased appetite, and mild fatigue.
You order a laboratory workup. Which one of the following would be most appropriate at this point?
A) Reassurance that the skin lesions are benign
B) A skin biopsy
C) Referral to a dermatologist
D) CT of the abdomen and pelvis
E) Upper and lower endoscopy
ANSWER: E
This patient’s age, risk factors, red-flag symptoms, and other clinical findings indicate the need for endoscopy. The Leser-Trélat sign may be defined as the abrupt onset of multiple seborrheic keratoses, which is an unusual finding that often indicates an underlying malignancy, most commonly an adenocarcinoma of the stomach. CT is not an initial approach for diagnosing a suspected malignancy of the stomach or colon. Further skin evaluation and lifestyle changes, which are indicated, will not address the need for evaluation of weight loss and other abnormal symptoms and findings.
A patient with hypercalcemia has low parathyroid hormone levels and elevated parathyroid hormon related peptid levels.
A. Familial hypocalciuric hypercalcemia
B. Pseudohypoparathyroidism
C. Vitamin D deficiency
D. Humoral hypercalcemia of malignancy
D. Humoral hypercalcemia of malignancy
Medication for reducing tension headaches
Amitryptyline
Antibiotic assocaited with c d.iff
clindamycin
What medications increased risk of ventilator associated pneumonia
HIstamine H2 blockers (famotidine)
PPI
Hb goal level in thalessemia
9-10.5
Pertussis prophylaxis
azithromycin within 21 days of exposure
Bishop score
Dilation
0 closed
1 1-2 cm
2 3-4 cm
3 >= 5 cm
Effacement
0 0-30%
1 40-50%
2 60-70%
3 >=80%
Fetal station
0 -3
1 -2
2 -1,0
3 +1, +2
> 5 = successful vaginal delivery
Weight in pregnancy
underweight 28-40 lb
normal weight 25-35
over weight 15-25
obese 11-20
New alzheimer drug
Aducanumab (aduhelm)
Triptans are contraindicated in who
cardiovascualr disease
cerbrovascular disease
uncontrolled htn
Cluter headache prophylaxis
Verapamil
Orthostatic hypotension definity
change in blood pressure of 20 mm Hg or more systolic of 10 mm Hg or more diastolic
Transietn global amnesia due to
confusion for up to 24 hours due to impairment of venous drainage from hippocampus
A 49-year-old male is concerned about lesions on his penis that he has noticed over the past 6 months. He was circumcised as a child and has had the same female sexual partner for 5 years. He does not have any pain, itching, or dysuria. On examination you note multiple reddish-blue papules on the scrotum and a few similar lesions on the shaft of the penis.
The most likely diagnosis is
A) pearly penile papules
B) lichen nitidus
C) lichen sclerosus
D) angiokeratomas
E) squamous cell carcinoma in situ (Bowen’s disease)
ANSWER: D
Penile lesions are usually easily diagnosed from clinical findings. Pearly penile papules are common and benign, and present as small, skin-colored, dome-shaped papules in a circular pattern around the coronal sulcus.
Lichen nitidus is benign but uncommon. It presents as discrete, pinhead-sized hypopigmented papules that are asymptomatic. Papules are often found scattered all over the penis, as well as on the abdomen and upper extremities.
Lichen sclerosus is more common and appears as hypopigmented lesions with the texture of cellophane. The lesions are usually located on the glans or prepuce. Atrophy, erosions, and bullae are common, and patients often present with itching, pain, bleeding, and possibly phimosis or obstructed voiding. Lichen sclerosus is associated with squamous cell cancer in a small percentage of cases.
Carcinoma in situ is a premalignant condition that is more common in uncircumcised males over age 60. Lesions are typically beefy red, raised, irregular plaques and can be found on the glans, meatus, frenulum, coronal sulcus, and prepuce. Lesions can be ulcerated or crusted. Pruritus and pain are common. A biopsy is important for making the diagnosis.
Angiokeratomas are lesions that are usually asymptomatic, circumscribed, red or bluish papules. They may appear solely on the glans of the penis, but are also found on the scrotum, abdomen, thighs, groin, and extremities. They may be misdiagnosed as pearly papules or carcinoma. Treatment is not necessary unless the lesions are bleeding or extensive. It is important to realize that angiokeratomas on the shaft of the penis, the suprapubic region, or the sacral region can be associated with Fabry disease. Patients with this finding should be promptly referred.
You see a patient who is being treated for opioid use disorder with buprenorphine. Which one of the following can be used as adjuvant treatment to reduce stress-related opioid cravings and increase abstinence?
A) Clonidine (Catapres)
B) Methadone
C) Naloxone
D) Naltrexone (Vivitrol)
E) Nifedipine (Procardia)
ANSWER: A
Clonidine (0.1–0.3 mg every 6–8 hours) is a useful adjunct to buprenorphine in the treatment of opioid use disorder to help increase the rates of abstinence and decrease stress-related opioid cravings (SOR C). Naloxone is an opioid antagonist used to treat overdoses. Nifedipine is a common antihypertensive like clonidine but it has no role in the treatment of opioid use disorder. Methadone and naltrexone are used to treat opioid use disorder but neither of these agents would be used simultaneously with buprenorphine.
In a 60-year-old patient who has not previously received pneumococcal vaccine, which one of the following would be an indication for both 13-valent pneumococcal conjugate vaccine (PCV13, Prevnar 13) and 23-valent pneumococcal polysaccharide vaccine (PPSV23, Pneumovax 23)?
A) Alcoholism
B) Chronic renal failure
C) Cigarette smoking
D) COPD
E) Diabetes mellitus
ANSWER: B
Both 13-valent pneumococcal conjugate vaccine (PCV13) and 23-valent pneumococcal polysaccharide vaccine (PPSV23) are recommended for patients with chronic renal failure. Indications for PPSV23 alone in immunocompetent persons younger than 65 include chronic lung disease, diabetes mellitus, chronic heart disease, smoking, and alcoholism.
A 36-year-old male presents with a 2-day history of painless right-sided facial droop. There are no associated symptoms and his medical history is otherwise unremarkable. An examination is remarkable for an unfurrowed right brow, mouth droop, a sagging right lower eyelid, and a complete inability to move the muscles of the right face and forehead. No other weakness is elicited and no rash is seen.
Which one of the following would be the most appropriate management at this point?
A) Reassurance only
B) Valacyclovir (Valtrex) alone
C) A tapering dose of prednisone alone
D) Valacyclovir and a tapering dose of prednisone
E) Immediate transfer to the emergency department
ANSWER: D
Early recognition and effective treatment of acute Bell’s palsy (idiopathic facial paralysis) has been shown to decrease the risk of chronic partial paralysis and pain. Corticosteroids have been shown in a meta-analysis to decrease chronic symptoms, but a Cochrane meta-analysis of 10 studies concluded that antiviral medication along with corticosteroids is significantly more effective than corticosteroids alone. The medications are most effective if started within 72 hours of symptom onset. The same analysis showed that antiviral medications alone were less effective than corticosteroids alone. This patient’s presentation is not consistent with stroke or another emergency. Because supranuclear input to the facial nerves comes from both cerebral hemispheres, strokes and other central pathologies affecting the facial nerves typically spare the forehead, which is not the case in this patient.
In a patient presenting with truncal obesity, hypertension, type 2 diabetes mellitus, hirsutism, osteopenia, and skin fragility, which one of the following tests is needed to confirm the diagnosis of Cushing syndrome?
A) A dexamethasone suppression test
B) Inferior petrosal sinus sampling
C) Plasma corticotropin
D) Plasma free cortisol
E) Urinary free cortisol
ANSWER: E
In a patient presenting with obesity, hypertension, type 2 diabetes mellitus, and hirsutism, who also has thin skin and osteopenia, an elevated 24-hour collection showing high urinary free cortisol confirms the presence of Cushing syndrome. The dexamethasone suppression test, though still commonly used, no longer has a place in the diagnosis and treatment of patients with Cushing syndrome. Corticotropin-dependent and corticotropin-independent causes of Cushing syndrome can be separated by measuring plasma corticotropin. Plasma free cortisol measurements should be obtained only to determine the success or failure of transsphenoidal microadenomectomy or adrenalectomy. Inferior petrosal sinus sampling is used to confirm the source of corticotropin secretion before surgical intervention.
A 50-year-old gravida 2 para 2 who is 3 years post menopausal presents with fatigue, headache, galactorrhea, and loss of libido. Your evaluation reveals elevated serum prolactin and a pituitary adenoma of 5–6 mm.
You recommend
A) bromocriptine (Parlodel)
B) estrogens
C) haloperidol
D) testosterone
E) neurosurgical consultation
ANSWER: A
This patient has a pituitary microadenoma. Microadenomas <10 mm in size that are secreting prolactin may be treated with a dopaminergic agent such as bromocriptine. This will lower the prolactin level and shrink the adenoma. Nonprolactin-secreting adenomas, especially those >10 mm in size (macroadenomas), require neurosurgical evaluation.
> 10 mm go to surgery!!
Coordination of the eyes in newborns improved by what age
6 months
A 47-year-old male presents with a 3-day history of fever, chills, low back pain, and urinary frequency. He does not have any nausea, vomiting, or abdominal pain. There is no significant past medical history.
The patient’s vital signs include a temperature of 38.1°C (100.6°F), a pulse rate of 88 beats/min, and a respiratory rate of 14/min. The examination reveals a mildly tender lower abdomen with no guarding or rebound tenderness; no costovertebral angle tenderness; and an enlarged, homogeneous, exquisitely tender prostate.
Which one of the following is indicated to help guide this patient’s treatment?
A) A serum prostate-specific antigen level
B) A culture of prostate secretions after massage of the prostate
C) A culture of midstream voided urine
D) CT of the abdomen and pelvis with intravenous and oral contrast
E) An ultrasound-guided prostate biopsy
ANSWER: C
This patient has clinically diagnosable acute bacterial prostatitis, and no further testing, including imaging, is required to establish the diagnosis. Culture of a midstream voided urine may aid in identifying the pathogen, but prostate massage should be avoided because it may increase the risk of bacteremia. A prostate biopsy is not indicated in the presence of acute infection, and a prostate-specific antigen level is not indicated because it is likely to be elevated in the presence of infection.
A 57-year-old female is admitted to the hospital with lower lobe pneumonia. She has no history of diabetes mellitus. She has not met sepsis criteria but had a blood glucose level of 172 mg/dL in the emergency department.
Insulin should be started if this patient has a persistent blood glucose level greater than or equal to
A) 120 mg/dL
B) 140 mg/dL
C) 160 mg/dL
D) 180 mg/dL
ANSWER: D
Insulin therapy should be initiated in hospitalized patients with persistent hyperglycemia, starting at a threshold of 180 mg/dL. Once insulin therapy is started, a target glucose range of 140–180 mg/dL is recommended for the majority of hospitalized patients, regardless of whether they have a critical illness.
A 21-year-old gravida 1 para 0 is diagnosed with overt hyperthyroidism early in the first trimester. The most appropriate management at this time is
A) observation only
B) methimazole (Tapazole)
C) propylthiouracil
D) radioactive iodine
E) thyroidectomy
ANSWER: C
Overt hyperthyroidism during pregnancy is associated with adverse effects to the mother and fetus, so treatment is required. Since methimazole is associated with birth defects when used in the first trimester, propylthiouracil is preferred. Methimazole should be considered after the first trimester because the risk of congenital anomalies is less than the risk of liver failure associated with propylthiouracil. Surgery and radioactive iodine should only be used if there is a clear indication, and radioactive iodine would not be appropriate during pregnancy.
A 45-year-old male with a 30-pack-year smoking history reports a chronic cough with a small amount of phlegm production and dyspnea with strenuous exercise. You order spirometry, which shows a pre- and postbronchodilator FEV1/FVC ratio of 0.6 and an FEV1 of 85% of predicted.
Which one of the following agents would be the best initial pharmacologic management?
A) An inhaled corticosteroid
B) A short-acting anticholinergic
C) A long-acting anticholinergic
D) A long-acting beta 2-agonist
E) Theophylline
ANSWER: B
This patient has COPD and is in a risk category of A (low risk, fewer symptoms) based on the Global Initiative for Chronic Obstructive Lung Disease (GOLD) combined assessment of COPD. As a result, either a short-acting anticholinergic or a short-acting 2-agonist should be selected as the initial pharmacologic management. Long-acting 2-agonists or long-acting anticholinergics are indicated for patients with a GOLD combined assessment category of B or worse. Long-acting inhaled corticosteroids are indicated for patients with a GOLD combined assessment category of C or worse. Due to its narrow therapeutic window, modest benefit, and need for monitoring, theophylline is not recommended as an initial agent and should be considered as an alternative only for patients with severe refractory symptoms.
A 30-year-old white male presents to the emergency department with a 4-day history of fever to 101°F, a sore throat, rhinorrhea, and cough. An examination reveals rhinorrhea and a boggy nasal mucosa, but is otherwise unremarkable. A chest radiograph shows a questionable infiltrate.
Which one of the following would help determine if antibiotic treatment would be appropriate?
A) A C-reactive protein level
B) A procalcitonin level
C) A WBC count with differential
D) An erythrocyte sedimentation rate
E) CT of the chest
ANSWER: B
Using a procalcitonin-guided therapy algorithm reduces antibiotic use by 3.47 days without increasing either morbidity or mortality in adults with acute respiratory infections. If the procalcitonin level is <0.10 mg/dL, a bacterial infection is highly unlikely and it is strongly recommended that antibiotics not be prescribed. If the procalcitonin level is 0.10–0.24 mg/dL a bacterial infection is still unlikely and it is recommended that antibiotics not be used. If the level is 0.25–0.50 mg/dL a bacterial infection is likely and antibiotics are recommended. It is strongly recommended that antibiotics be given if the level is >0.50 mg/dL, because a bacterial infection is very likely.
An 80-year-old male sees you for the first time. He is asymptomatic except for some fatigue. His pulse rate is 50 beats/min. An EKG shows a prolonged PR interval.
Which one of the following medications in his current regimen is the most likely explanation for these findings?
A) Donepezil (Aricept)
B) Escitalopram (Lexapro)
C) Lisinopril (Prinivil, Zestril)
D) Memantine (Namenda)
E) Zolpidem (Ambien)
ANSWER: A
The 2015 American Geriatrics Society Beers Criteria for potentially inappropriate medication use in older adults 65 years of age states that donepezil use should be avoided in patients with syncope, due to an increased risk of bradycardia (Moderate Evidence Level; Strong Strength of Recommendation). Donepezil is a cholinesterase inhibitor. Due to their cholinergic effect, these medications have a vagotonic effect on the sinoatrial and atrioventricular nodes. This can cause bradycardia or heart block in patients with or without underlying cardiac conduction abnormalities. Syncope has been reported with these medications.
A 14-year-old female is brought to your office for an annual well child check and sports preparticipation physical examination. She says she does a lot of running during basketball practices and games but has trouble controlling her weight. Most of her family is overweight. She does not have any difficulty participating in sports, and has no symptoms such as chest pain, shortness of breath, or headaches. She has no significant past medical history.
On examination the patient’s height is 154 cm (61 in) and she weighs 63 kg (139 lb). Her BMI is 26.4 kg/m2, which places her in the 90th percentile for her age. Her blood pressure is 130/85 mm Hg, which places her between the 95th and 99th percentile for her age, height, and sex. Her chart reveals that her blood pressure was at this level at the last two visits. The physical examination is otherwise normal.
In addition to counseling and support for weight loss, which one of the following would be most appropriate at this point?
A) Informing the patient and her parents that she is prehypertensive and having her return for a blood pressure check in 3 months
B) Plasma renin and catecholamine levels
C) An imaging study of the renal arteries
D) A fasting basic metabolic panel, a lipid profile, and a urinalysis
E) Antihypertensive drug therapy
ANSWER: D
In a pediatric patient, blood pressure should be evaluated using comparisons based on age, sex, and height. Although this adolescent’s blood pressure is prehypertensive for an adult according to JNC 8 guidelines, it is stage 1 hypertension (between 95% and 99%) for her age, sex, and height. All pediatric patients with confirmed hypertension should have further evaluation to check for renal dysfunction as well as other cardiac risk factors. Additionally, renal ultrasonography is recommended to evaluate for renal disease and echocardiography to evaluate for end-organ damage that would affect treatment goals. Additional studies, such as plasma renin and catecholamine levels or renovascular imaging, may be indicated in children with abnormalities on initial evaluation that suggest secondary causes of hypertension.
Pharmacologic therapy is usually recommended for pediatric patients with symptomatic hypertension, secondary hypertension, target organ damage, diabetes mellitus, or persistent hypertension in spite of nonpharmacologic treatment. A low-sodium diet may be helpful for decreasing blood pressure, and given this patient’s obesity, intensive counseling about lifestyle changes is appropriate.
An otherwise healthy 57-year-old male presents with mild fatigue, decreased libido, and erectile dysfunction. A subsequent evaluation of serum testosterone reveals hypogonadism.
Which one of the following would you recommend at this time?
A) No further diagnostic testing
B) A prolactin level
C) A serum iron level and total iron binding capacity
D) FSH and LH levels
E) Karyotyping
ANSWER: D
In men who are diagnosed with hypogonadism with symptoms of testosterone deficiency and unequivocally and consistently low serum testosterone concentrations, further evaluation with FSH and LH levels is advised as the initial workup to distinguish between primary and secondary hypogonadism. If secondary hypogonadism is indicated by low or inappropriately normal FSH and LH levels, prolactin and serum iron levels and measurement of total iron binding capacity are recommended to determine secondary causes of hypogonadism, with possible further evaluation to include other pituitary hormone levels and MRI of the pituitary. If primary hypogonadism is found, karyotyping may be indicated for Klinefelter’s syndrome.
A 35-year-old female comes to your office for evaluation of a tremor. During the interview you note jerking movements first in one hand and then the other, but when the patient is distracted the symptom resolves. Aside from the intermittent tremor the neurologic examination is unremarkable. She does not drink caffeinated beverages and takes no medications.
Which one of the following is the most likely diagnosis?
A) Parkinson’s disease
B) Cerebellar tremor
C) Essential tremor
D) Physiologic tremor
E) Psychogenic tremor
ANSWER: E
Psychogenic tremor is characterized by an abrupt onset, spontaneous remission, changing characteristics, and extinction with distraction. Cerebellar tremor is an intention tremor with ipsilateral involvement on the side of the lesion. Neurologic testing will reveal past-pointing on finger-to-nose testing. CT or MRI of the head is the diagnostic test of choice. Parkinsonian tremor is noted at rest, is asymmetric, and decreases with voluntary movement. Bradykinesia, rigidity, and postural instability are generally noted. For atypical presentations a single-photon emission CT or positron emission tomography may help with the diagnosis. One of the treatment options is carbidopa/levodopa.
Patients who have essential tremor have symmetric, fine tremors that may involve the hands, wrists, head, voice, or lower extremities. This may improve with ingestion of small amounts of alcohol. There is no specific diagnostic test but the tremor is treated with propranolol or primidone. Enhanced physiologic tremor is a postural tremor of low amplitude exacerbated by medication. There is usually a history of caffeine use or anxiety.
A nulliparous 34-year-old female comes to your office for evaluation of fatigue, hair loss, and anterior neck pain. These symptoms have been gradually worsening for the past few months. Her past medical history is unremarkable. She has gained 5 kg (11 lb) since her last office visit 18 months ago. Examination of the thyroid gland reveals tenderness but no discrete nodules. Her TSH level is 7.5 U/mL (N 0.4–4.2), her T4 level is low, and her thyroid peroxidase antibodies are elevated.
Which one of the following would be the most appropriate next step?
A) Continue monitoring TSH every 6 months
B) Begin thyroid hormone replacement and repeat the TSH level in 6–8 weeks
C) Begin thyroid hormone replacement and repeat the TSH level along with a T3 level in
6–8 weeks
D) Order ultrasonography of the thyroid
E) Order fine-needle aspiration of the thyroid
ANSWER: B
This patient has thyroiditis with biochemical evidence for autoimmune (Hashimoto’s) thyroiditis. The most appropriate plan of care is to begin thyroid hormone replacement and monitor with a repeat TSH level 6–8 weeks later. It is not necessary to include a T3 level when assessing the levothyroxine dose. There is no need to routinely order thyroid ultrasonography when there are no palpable nodules on a thyroid examination. Fine-needle aspiration may be necessary to rule out infectious thyroiditis when a patient presents with severe thyroid pain and systemic symptoms.
thyroid peroxidase antibodies
Hashimoto
Mobitz type 1 vs mobitz type 2
Mobitz type 1 cardiac monitoring
- prolonging P
Mobitz type 2 need pace maker
- Same p wave
antibiotic for travelers diarrhea
azithromycin
A 30-year-old gravida 1 para 0 develops erythematous patches with slightly elevated scaly borders during her first trimester. There was a 2-cm herald patch 2 weeks before multiple smaller patches appeared. The rash on the back has a “Christmas tree” pattern. She has not had any prenatal laboratory work.
This condition is associated with
A) no additional pregnancy risk
B) a small-for-gestational-age newborn
C) congenital cataracts
D) multiple birth defects
E) spontaneous abortion
ANSWER: E
This patient has classic pityriasis rosea. This is generally a benign disease except in pregnancy. The epidemiology and clinical course suggest an infectious etiology. Pregnant women are more susceptible to pityriasis rosea because of decreased immunity. Pityriasis rosea is associated with an increased rate of spontaneous abortion in the first 15 weeks of gestation. It is not associated with an increased risk for a small-for-gestational-age newborn, congenital cataracts, or multiple birth defects.
herald patch
pityrasis rosea
cough, coryza, conjunctivitis, and Koplik spots.
Measles
Mumps
myalgia, fatigue, loss of appetite, fever, and headache
orchitiis
A 5-year-old female is brought to your office with a progressive rash on her legs (shown below) and buttocks. No rash is noted above the level of the mid-torso. Her mother also reports that the child had two episodes of bloody diarrhea 3 days ago. She also has abdominal pain and on examination she has abdominal tenderness with no rigidity but some voluntary guarding. You also note swelling and tenderness in her left wrist and right knee. A CBC, platelets, prothrombin time, and partial thromboplastin time are normal. A urinalysis reveals mild proteinuria and 5–10 RBCs/hpf.
Which one of the following is the most likely diagnosis?
A) Erythema infectiosum (fifth disease)
B) Gianotti-Crosti syndrome
C) Hemolytic uremic syndrome
D) Henoch-Schönlein purpura
E) Thrombotic thrombocytopenic purpura
ANSWER: D
Henoch-Schönlein purpura is an IgA vasculitis that is usually diagnosed clinically. It presents as palpable purpura of the lower extremities without thrombocytopenia or coagulopathy. It is often associated with arthralgias and arthritis, abdominal pain, and renal dysfunction. It is self-limited and treatment is supportive only. Erythema infectiosum (fifth disease) can be identified by an erythematous rash on the cheeks and a lacy reticular rash on the extremities. Gianotti-Crosti syndrome is a sudden papular or papulovesicular eruption on the extensor surfaces of the arms, legs, buttocks, and face, and it is not purpuric. Hemolytic uremic syndrome presents with the classic triad of hemolytic anemia, thrombocytopenia, and kidney injury. Thrombotic thrombocytopenic purpura is rare in the pediatric age group.
LDCT start at what age
50-80
A 32-year-old female presents with heat intolerance, excessive weight loss, and anxiety. She gave birth 6 months ago and recently stopped breastfeeding. On examination her thyroid gland is slightly diffusely enlarged and nontender. Laboratory studies reveal a decreased TSH level and elevated free T3 and T4 levels. You suspect that she has postpartum thyroiditis.
Which one of the following tests would be most useful to confirm the diagnosis?
A) Radioactive iodine uptake
B) Thyroid peroxidase antibody levels
C) Thyroid ultrasonography
D) Thyrotropin receptor antibody levels
ANSWER: A
Postpartum thyroiditis is defined as a transient or persistent thyroid dysfunction that occurs within 1 year of childbirth, miscarriage, or medical abortion. Release of preformed thyroid hormone in the bloodstream initially results in hyperthyroidism. During the hyperthyroid phase, radioactive iodine uptake will be low, which can help to confirm the diagnosis. Pregnancy and breastfeeding are contraindications to radionuclide imaging. Thyroid peroxidase antibody levels are elevated with chronic autoimmune thyroiditis (Hashimoto’s thyroiditis), and patients present with symptoms of hypothyroidism. The Endocrine Society and American Association of Clinical Endocrinologists do not recommend routine thyroid ultrasonography in patients with abnormal thyroid function tests if there is no palpable abnormality of the thyroid gland. Thyrotropin receptor antibody levels are positive in Graves disease.
A 13-year-old female is brought to your office by her father for evaluation of left knee pain. It has been slowly worsening over the past several weeks and she does not recall any inciting injury or event. She plays soccer and participates in practice 3 days a week with games on the weekends. She is finding it increasingly difficult to participate fully due to pain. She notes some swelling and tenderness just below her kneecap. She has tried icing the area after activity and has recently started taking ibuprofen with moderate relief of the pain.
On examination you note prominence of the tibial tubercle with tenderness to palpation. The remainder of the knee examination is unremarkable.
Which one of the following is indicated at this time to further assess this condition?
A) No imaging
B) Radiography
C) Ultrasonography
D) CT
E) MRI
ANSWER: A
This patient has Osgood-Schlatter disease, a common cause of knee pain in active children with immature skeletons. It occurs as a result of abnormal development, injury, or overuse of the growth plate and the surrounding ossification centers. Osteochondrosis is a more general term for this condition, which can occur at growth plates around other joints, including the hip, foot, elbow, and back. In Osgood-Schlatter disease repetitive traction of the patellar tendon on the tibial tubercle ossification center leads to inflammation and pain. Imaging is not required to make the diagnosis when patients present with typical symptoms and physical examination findings. Radiographs may be obtained if there is uncertainty about the diagnosis. Radiographic findings in Osgood-Schlatter disease include soft-tissue swelling and fragmentation of the tibial tubercle.
This condition is self-limited and treatment consists of activity modification and the use of acetaminophen or NSAIDs. An orthopedic referral is indicated if symptoms persist in a patient with a mature skeleton.
A 75-year-old male nursing home resident is brought to the emergency department with a cough and fever. His past medical history is significant for coronary artery disease, COPD, hypertension, and osteoarthritis. On examination he has a blood pressure of 145/90 mm Hg, a pulse rate of 84 beats/min, and an oxygen saturation of 89% on room air. A physical examination is remarkable for mildly labored breathing and crackles in his left lower lung field. A chest radiograph confirms left lower lobe pneumonia. He is admitted to the hospital for intravenous antibiotics.
Which one of the following would be the most appropriate antibiotic treatment?
A) Cefdinir only
B) Piperacillin/tazobactam (Zosyn) only
C) Ceftriaxone and azithromycin (Zithromax)
D) Cefixime (Suprax) and vancomycin (Vancocin)
E) Piperacillin/tazobactam, vancomycin, and ciprofloxacin (Cipro)
ANSWER: C
Current recommendations state that nursing home–acquired pneumonia should be treated as community-acquired pneumonia unless patients have severe illness, chronic wounds, foreign bodies in the airway, a history of antibiotic use in the last 90 days or recent hospitalization, colonization with multidrug-resistant pathogens, or very low functional status, or reside in a facility with a high prevalence of multidrug-resistant pathogens. Community-acquired pneumonia should be treated with either a respiratory fluoroquinolone or an advanced macrolide plus a -lactam antibiotic. Doxycycline could also be used in place of the macrolide.
A 56-year-old male with a history of hepatitis C cirrhosis is admitted to the hospital with gastrointestinal (GI) bleeding. The patient has been stable, taking only furosemide (Lasix) and spironolactone (Aldactone). Upper GI endoscopy confirms variceal bleeding and the gastroenterologist performs appropriate variceal banding.
A nurse calls you because laboratory studies that were ordered in the emergency department reveal a serum ammonia level of 120 g/dL (N 39–90). The patient has no signs of confusion, insomnia, or decreased mental alertness. A physical examination reveals mild ascites but no other abnormalities.
Which one of the following would be most appropriate for addressing the elevated ammonia level?
A) No additional treatment
B) Lactulose (Kristalose)
C) Methotrexate
D) Neomycin
E) Prednisone
ANSWER: A
Elevated ammonia levels may occur with urea cycle disorders, portosystemic shunting, urinary tract infection from urease-producing organisms, gastrointestinal bleeding, shock, renal disease, heavy exercise, smoking, parenteral nutrition, salicylate intoxication, use of medications including diuretics, and alcohol use. In patients with chronic liver disease, hepatic encephalopathy is diagnosed based on the overall clinical presentation and not on ammonia levels. A normal ammonia level does not exclude the diagnosis of hepatic encephalopathy, nor does an elevated ammonia level establish the diagnosis. This patient had an elevated serum ammonia level that was found incidentally during his hospital admission for gastrointestinal bleeding. Because there was no clinically significant encephalopathy, treatment based on ammonia levels is not indicated. The patient’s elevated ammonia level was probably from diuretic use. Lactulose, methotrexate, neomycin, or prednisone would not be appropriate.
A 37-year-old female presents with a 3-day history of left thumb pain. She first noted diffuse pain in her left thumb after a fall while skiing. She does not recall the mechanism of injury. The pain is greatest at the medial metacarpophalangeal joint, but there is no mass or instability. A radiograph does not show any fractures.
An examination reveals mild laxity in her ulnar collateral ligament with 30° of joint opening when abduction stress is applied to the distal thumb while stabilizing the metacarpal. The right side has only 15° of joint opening.
Which one of the following would be most appropriate at this point?
A) Reassurance and follow-up if symptoms do not improve
B) Anti-inflammatory medication for 7–10 days
C) Immobilization of the left thumb in a thumb spica cast or brace for 6 weeks
D) Immediate referral to an orthopedic surgeon for surgical repair
ANSWER: C
Ulnar collateral ligament (UCL) disruption, or “skier’s thumb,” should be suspected in traumatic thumb injuries. It is important to recognize and treat this injury because it can lead to joint instability and a weak pincer grip if untreated. Initial treatment of UCL disruption involves immobilization of the affected thumb in a thumb spica cast or brace for 6 weeks. In the absence of an avulsion fracture, indications for referral to an orthopedic surgeon would include 35°–40° of joint opening or no end point on stress abduction testing. A Stener lesion (entrapment of the UCL outside of the adductor aponeurosis) would usually present with joint instability and a tender mass and would necessitate an orthopedic referral.
A 65-year-old female with hypertension, osteoporosis, and GERD presents to your office for a well woman visit. She reports no new symptoms or concerns. A review of laboratory work performed prior to her visit reveals lipid levels at goal, normal glucose and sodium levels, a calcium level of 10.6 mg/dL (N 8.6–10.3), an albumin level of 4.1 g/dL (N 3.6–5.1), and a 25-hydroxyvitamin D level of 35 ng/mL (N 20–50). Her calcium level was 10.5 mg/dL on a basic metabolic panel 6 months ago.
The patient’s medications include hydrochlorothiazide, 12.5 mg daily; lisinopril (Prinivil, Zestril), 10 mg daily; alendronate (Fosamax), 70 mg weekly; omeprazole (Prilosec), 20 mg daily as needed; and vitamin D, 2000 IU daily. The patient’s blood pressure is 110/60 mm Hg. An examination is normal.
In addition to ordering follow-up laboratory studies and scheduling a follow-up visit in 1 month, which one of the following would be most appropriate?
A) Discontinue alendronate
B) Discontinue hydrochlorothiazide
C) Discontinue lisinopril
D) Discontinue omeprazole and begin ranitidine (Zantac)
E) Increase vitamin D to 5000 IU daily
ANSWER: B
This patient has hypercalcemia with a normal albumin level. Hydrochlorothiazide can cause drug-induced hypercalcemia. Alendronate, lisinopril, and omeprazole do not cause hypercalcemia. A high vitamin D level can cause hypercalcemia, so increasing vitamin D is not appropriate at this point. A laboratory evaluation can help differentiate between PTH- and non–PTH-mediated hypercalcemia.
A 48-year-old male is brought to the medical tent of a marathon after he became confused and dizzy in the middle of the race. On examination his rectal temperature is 41.1°C (106.0°F).
Which one of the following should be initiated immediately?
A) Passive cooling in an air-conditioned space
B) Gradual cooling with a cold water spray and a fan
C) Ice packs applied to the groin, neck, and axilla
D) Full-body immersion in an ice bath
E) Cold intravenous fluids
ANSWER: D
Heatstroke can be nonexertional from prolonged exposure to a high heat index, or it can be exertional, as in this case. A core temperature >40°C (104°F) is consistent with heatstroke. In treating patients with either clinical variant of heatstroke, cold or ice-water immersion is the most effective treatment and should be initiated as soon as possible, without delaying for transfer to the hospital setting (SOR A). Treatment should continue until the core body temperature is <39°C (102°F). If cold water immersion is not possible other forms of cooling such as cold intravenous fluids, ice packs, cold water immersion of the extremities, and evaporative cooling have been shown to have some benefit. Once the body temperature is decreased patients should be transferred to a hospital for evaluation for known complications of heatstroke, including coagulopathy, renal and hepatic dysfunction, hypoglycemia, electrolyte disturbance, and rhabdomyolysis.
basophilic stippling
anemia associated with lead toxicity
Schistocytes
hemolytic anemia.
You are administering a mental status examination to a 92-year-old male with suspected dementia. You give the patient a pencil and ask him to show how it is used. He gives you a bewildered look and eventually puts the pencil in his mouth and demonstrates using it as a toothbrush.
This task assesses
A) executive functioning
B) gnosia
C) orientation
D) praxis
E) visuospatial proficiency
ANSWER: D
Praxis is the ability to carry out intentional motor acts and is commonly assessed by giving the patient a common object such as a hairbrush or pencil and asking the patient to show how it is used. A patient unable to carry out such motor acts is referred to as having apraxia (SOR C). Several other common components of the cognitive assessment will be impaired in persons with dementia. Executive functioning is the ordering and implementation of cognitive functions necessary to engage in appropriate behavior and is often assessed by asking a patient to draw a clock with the hands set at a certain time.
Gnosia is the ability to name objects and their function and is often assessed by showing a patient a common object such as a pen, watch, or stethoscope and asking whether he or she can identify it and describe how it is used. Orientation is the ability of the patient to recognize his or her place in time and space. Orientation is commonly assessed by asking a patient the date, the current location, his or her name, and his or her place of birth. Visuospatial proficiency is the ability to perceive and manipulate objects and shapes in space. It is often assessed by asking the patient to copy intersecting pentagons or a three-dimensional cube on paper.
A 58-year-old male with well controlled type 2 diabetes presents for follow-up of ongoing exertional chest pain that has been present for months with no significant change. He has a remote history of myocardial infarction and recently underwent cardiac catheterization, which showed non-obstructing coronary artery disease with a left ventricular ejection fraction of 60%. His current medications include aspirin, 81 mg; atorvastatin (Lipitor), 80 mg; lisinopril (Prinivil, Zestril), 40 mg; metoprolol succinate (Toprol-XL), 100 mg; and metformin (Glucophage), 1000 mg twice daily. Home blood pressure monitoring shows an average blood pressure of 142/92 mm Hg and a pulse rate of 58 beats/min. A physical examination, including a cardiovascular examination, is unremarkable.
Which one of the following medications would be the best addition to his current regimen?
A) Amlodipine (Norvasc), 2.5 mg daily
B) Diltiazem extended release (Cardizem LA), 120 mg daily
C) Isosorbide mononitrate extended release, 30 mg daily
D) Nifedipine (Procardia), 30 mg 3 times daily
E) Ranolazine (Ranexa), 500 mg twice daily
ANSWER: A
This patient has a blood pressure that is elevated according to all major current guidelines. Amlodipine, a long-acting dihydropyridine calcium channel blocker (CCB), is the best pharmacologic choice because it will lower blood pressure and treat angina without the risk of heart block. Short-acting CCBs such as non–extended-release nifedipine may cause reflex tachycardia and are not recommended. Nondihydropyridine CCBs such as diltiazem would put this patient at risk for heart block because he is already taking long-acting metoprolol and his heart rate is in the 50s. Long-acting nitrates and ranolazine are options to treat stable angina but would not be effective blood pressure medications.
An 8-year-old male was brought to your office 7 months ago because of frequent diarrhea and abdominal discomfort that had been present for 11⁄2–2 years. An examination revealed no abnormalities. A CBC, a serum iron level, and a metabolic panel were normal. Serology was consistent with celiac disease and a duodenal biopsy confirmed the diagnosis. The patient’s symptoms resolved after his parents took him to a nutritionist who recommended a gluten-free diet. Today you see the child for preventive care and he remains asymptomatic.
In addition to normal well child care, which one of the following would you recommend?
A) No further testing
B) Gradual reintroduction of gluten into the diet
C) IgG antigliadin antibody
D) IgA tissue transglutaminase antibody
E) A duodenal mucosal biopsy to ensure healing
ANSWER: D
Celiac disease affects approximately 1% of the U.S. population and can affect all ages. Individuals with northern European ancestry are most commonly affected. The condition is caused by autoimmunity induced by gluten-containing foods in susceptible individuals. Untreated celiac disease is associated with anemia, malabsorption, osteoporosis, weight loss, and gastrointestinal lymphomas. In children, growth stunting and delayed puberty are also common. With strict adherence to a gluten-free diet most complications from celiac disease are preventable and, in children, growth and development return to normal. The World Gastroenterology Organisation recommends annual monitoring of children and adolescents with celiac disease by anthropometry, pubertal development, and celiac serology. The preferred serology is IgA antibody to tissue transglutaminase (IgA anti-tTG) due to its balance of good performance and low cost. Failure of IgA anti-tTG titers to decrease in 6 months suggests continued ingestion of gluten. Repeat duodenoscopy with a biopsy would be unnecessary and would subject the child to potential harm.
An unimmunized child is brought to the local health department and diagnosed with measles. All individuals at the day care facility that the child attends were exposed to this child about 48 hours prior to the diagnosis.
Which one of the following should be given the MMR vaccine as postexposure prophylaxis?
A) An otherwise healthy 9-month-old child who is up to date on all age-appropriate immunizations
B) A 5-year-old child with asthma who received a second dose of MMR 2 weeks ago
C) A pregnant 24-year-old day care staff member who received two doses of MMR as a
child
D) A pregnant 36-year-old day care staff member who had one dose of MMR vaccine last
year when she started working at the day care facility
E) A 52-year-old nurse who works part-time at the day care facility and received two doses
of MMR as an adult
ANSWER: A
Measles outbreaks are becoming more common and the CDC has outlined who should receive postexposure prophylaxis with the MMR vaccine. To be effective as postexposure prophylaxis the vaccine must be administered within 72 hours of exposure.
Infants <12 months of age are considered to be at high risk for complications from measles and should receive postexposure MMR vaccine, although intramuscular immunoglobulin is also an option. Children who are otherwise fully vaccinated do not need additional prophylaxis. Pregnant women cannot receive the MMR vaccine due to fetal risk, but they should receive intravenous immunoglobulin if they do not have evidence of immunity. Health care workers only need to be given the MMR vaccine as prophylaxis if they did not receive two doses previously.
A 58-year-old female presents to your office to discuss a new diagnosis of COPD. She has a 40-pack-year smoking history, and she quit using tobacco 18 months ago. Spirometry performed last week showed an FEV1/FVC ratio of 0.62 with an FEV1 that is 75% of predicted. She was first treated for an exacerbation last month and has never required hospitalization for any respiratory illness. You administer a COPD assessment test and she rates herself at 16/40, which is a moderately high score.
Which one of the following inhaled medications would be appropriate to initiate today?
A) Albuterol (Proventil, Ventolin)
B) Fluticasone (Flovent)
C) Fluticasone/salmeterol (Advair)
D) Ipratropium (Atrovent)
E) Tiotropium (Spiriva)
ANSWER: E
This patient has airflow obstruction consistent with a diagnosis of COPD, evidenced by an FEV1/FVC ratio <0.7. An evidence-based standard approach to COPD is found in the annual guidelines published by the Global Initiative for Chronic Obstructive Lung Disease (GOLD). According to the 2019 report, this patient’s FEV1 of 75% of predicted puts her in the GOLD grade 2 (moderate) category of airflow limitation. Her symptoms and risk of exacerbations places her in GOLD group B. Patients in this category have symptoms that bother them regularly without having frequent COPD exacerbations. Patients in this category benefit from daily use of long-acting bronchodilators, either long-acting -agonists (LABAs) or long-acting muscarinic agents (LAMAs). Long-acting agents such as tiotropium (a LAMA) or salmeterol (a LABA) are preferred over the short-acting agents ipratropium and albuterol for patients in this category of disease severity. Patients with persistent symptoms while using one of these agents may benefit from a combination of a LABA and a LAMA.
An otherwise healthy 55-year-old male who is visiting from Arizona presents to your office with a 4-week history of intermittent fevers, night sweats, dry cough, weight loss, and myalgia. The patient has no other recent history of travel.
Of the following, the most likely cause of his symptoms is
A) blastomycosis
B) coccidioidomycosis C) cryptococcosis
D) histoplasmosis E) mucormycosis
ANSWER: B
Knowledge of endemic fungi capable of causing infection in otherwise healthy patients can be very helpful in ensuring an appropriate evaluation. Coccidioidomycosis is a common infection in the southwestern United States. In addition to the symptoms in this patient, coccidioidomycosis can also present with a rash such as erythema nodosum. Histoplasmosis is most common in the Midwest and with low-level exposure symptoms are usually mild or absent. Blastomycosis is also present in the Midwest, as well as in the Atlantic and southeastern states. Symptoms include an abrupt onset of fever, chills, pleuritic chest pain, arthralgias, and myalgias. The cough is initially nonproductive but frequently becomes purulent. Cryptococcosis and mucormycosis are more opportunistic infections occurring in immunocompromised hosts.
A 25-year-old male presents with a 4-month history of crampy abdominal pain, diarrhea, and fatigue. His symptoms began gradually but have become more severe and he is now experiencing rectal bleeding. He says that his abdominal pain seems to temporarily improve after eating. He has smoked five cigarettes per day for the past 8 years. He is surprised to learn that he has lost 7 kg (15 lb) when he is weighed today.
His vital signs include a blood pressure of 116/70 mm Hg, a heart rate of 76 beats/min, a respiratory rate of 12/min, and a temperature of 37.7°C (99.9°F). A physical examination reveals abdominal tenderness and mild distention. An anorectal examination is significant for a perianal fistula. A laboratory evaluation is notable for mild anemia. His kidney and liver function are normal.
Which one of the following is the most likely diagnosis?
A) Celiac disease
B) Chronic pancreatitis
C) Crohn’s disease
D) Irritable bowel syndrome
E) Ulcerative colitis
ANSWER: C
Crohn’s disease may present insidiously with diarrhea, abdominal pain, rectal bleeding, fever, weight loss, and fatigue. Red-flag symptoms include perianal lesions, a first degree relative with inflammatory bowel disease, weight loss of 5% of the patient’s usual weight, abdominal pain for more than 3 months, nocturnal diarrhea, fever, the absence of abdominal pain for 30–45 minutes after eating, and the absence of rectal urgency. This patient exhibits symptoms consistent with Crohn’s disease. While anemia is also common in celiac disease, rectal bleeding is not. Chronic pancreatitis does not generally present with improved pain after eating. Irritable bowel syndrome is not associated with fever, rectal bleeding, anemia, or perianal fistulas. Ulcerative colitis is not associated with perianal lesions.
An 18-month-old female is brought to your office in January for evaluation of a cough and fever. She has no chronic medical conditions. She abruptly developed a barking cough and hoarseness with a low-grade fever 2 days ago. The cough is worse at night. She has been drinking normally but is not interested in eating. On examination she is alert and resists the examination. Her respiratory rate and effort are normal. She has no stridor or wheezing.
Which one of the following would be most appropriate at this point?
A) A nasal swab for influenza testing
B) A chest radiograph
C) A single dose of oral dexamethasone
D) Azithromycin (Zithromax)
E) Oseltamivir (Tamiflu)
ANSWER: C
This patient has symptoms consistent with croup, a lower respiratory infection that is common in the winter months in children ages 6 months to 3 years. The diagnosis is clinical and should be suspected in children with a history of a sudden onset of a deep cough, hoarseness, and a low-grade fever. Randomized studies have shown that even with mild croup (an occasional barking cough with no stridor at rest), oral corticosteroids provide some benefit.
A Cochrane review of two randomized trials with a total of 2024 patients found that chest radiographs did not change the outcome of ambulatory children with lower respiratory tract infections. A patient such as this would not need antiviral treatment for influenza.
34-year-old female at 32 weeks gestation presents with a right-sided, pounding headache that began 8 hours ago and is similar to headaches she has had in the past. She is sensitive to light and sound, and has vomited several times since the onset of pain. She has taken acetaminophen without relief. She takes prenatal vitamins but no other routine medications. On examination her blood pressure is normal.
Which one of the following would be the most appropriate treatment for this patient?
A) Dihydroergotamine
B) Metoclopramide (Reglan)
C) Naproxen
D) Oxycodone (OxyContin)
E) Sumatriptan (Imitrex)
ANSWER: B
Metoclopramide and acetaminophen are the only two medications considered safe for abortive migraine treatment during pregnancy (SOR B). The dopamine antagonist antiemetics are considered second-line abortive treatments in the general population. Dihydroergotamine should not be used during pregnancy due to its oxytocic properties and the potential risk of intrauterine growth restriction with its use. NSAIDs are not considered safe during pregnancy, particularly in the first and third trimesters. Opioids are only moderately useful for migraine treatment and should be avoided during pregnancy due to their abuse potential. Triptans are generally considered safe during the first trimester but not in the second and third trimesters. Their use has been associated with uterine atony, increased risk of bleeding during delivery, and increased risk of preterm birth.
Which one of the following U-100 insulin products has the longest duration of action?
A) Degludec (Tresiba)
B) Glargine (Lantus)
C) Isophane NPH (Humulin N)
D) Lispro (Humalog)
E) Regular (Humulin R)
ANSWER: A
Among the available U-100 insulin products, the one with the longest duration of action is ultralong-acting degludec, which lasts 42 hours. The duration of action of rapid-acting lispro is 3–6.5 hours, short-acting regular is 5–8 hours, intermediate-acting isophane is 12–16 hours, and long-acting glargine is 11–24 hours.
A 9-year-old male with a history of moderate persistent asthma is brought to the emergency department with an acute exacerbation. His symptoms began with a runny nose and nasal congestion 2 days ago. His parents state that he has not had any fevers or chills and he was eating and drinking well until a few hours ago when his breathing started to appear more labored. After multiple treatments with inhaled albuterol (Proventil, Ventolin) and oral prednisolone he remains tachypneic and wheezy.
Which one of the following intravenous medications should be added to the patient’s current treatment to reduce the likelihood of hospital admission?
A) Ketorolac
B) Magnesium sulfate
C) Methylprednisolone
D) Omalizumab (Xolair)
E) Theophylline
ANSWER: B
Children who present to the emergency department with an asthma exacerbation and fail to improve adequately with inhaled short-acting bronchodilators and corticosteroids may benefit from treatment with intravenous (IV) magnesium sulfate. A 2016 Cochrane review of three randomized, controlled trials found that this reduced hospital admissions by 68%. Ketorolac is not known to have any benefit in the treatment of asthma. Oral administration of corticosteroids is as effective as IV administration, so there is no reason to give IV methylprednisolone. Omalizumab may be used to prevent exacerbations in patients with severe asthma who do not achieve adequate control with high-dose inhaled corticosteroids, but it has no role in the management of acute exacerbations. IV theophylline is not recommended for asthma exacerbations given its safety profile and poor efficacy compared to short-acting bronchodilators.
corticotropin stimulation
For cushings
Antitrypsin organs effect
Lungs and liver
Good pasture vasculits affects
Lungs and kidneys
Wellbutrin false positive on drug test for what
amphetamines
False positive for marijuana on drug screen
PPI
False positive for opioids
Levofloxacin
Falso positive for benzo
Sertraline
For patients with atrial fibrillation, which one of the following comorbid conditions represents the strongest indication for thromboprophylaxis with warfarin (Coumadin), rather than a direct oral anticoagulant?
A) A CHA2DS2-VASc score 3
B) End-stage chronic kidney disease
C) A mechanical heart valve
D) Mild mitral stenosis
E) Severe mitral regurgitation
ANSWER: C
According to 2019 guidelines from the American College of Cardiology, American Heart Association, and Heart Rhythm Society, patients with nonvalvular atrial fibrillation and an elevated CHA2DS2-VASc score (2 in men and 3 in women) should receive anticoagulation, preferably with a direct-acting oral anticoagulant (DOAC), rather than warfarin (level of evidence A). Recent evidence has shown that DOAC options are not inferior to, and in some studies are superior to, warfarin for preventing strokes and systemic embolic events, with a lower risk of serious bleeding. However, warfarin is still recommended over a DOAC for valvular atrial fibrillation that occurs in the presence of moderate to severe mitral stenosis or a mechanical heart valve. For atrial fibrillation in patients with other forms of valvular heart disease, including mitral regurgitation and mild mitral stenosis, DOAC therapy is preferred over warfarin. For patients with atrial fibrillation and end-stage chronic kidney disease, both apixaban (a direct factor Xa inhibitor) and warfarin are comparable options.
You are examining a 65-year-old male from Central America with a history of rheumatic valvular disease. Which one of the following is the principal auscultatory finding of aortic regurgitation?
A) An S3 gallop heard best at the cardiac apex with the patient supine
B) A triphasic pericardial friction rub with the patient seated and leaning forward
C) A low-pitched decrescendo diastolic murmur that is loudest at the lower left sternal
border with the patient seated and leaning forward
D) A high-pitched crescendo/decrescendo midsystolic murmur that is loudest at the right
upper sternal border and radiates to the carotid arteries with the patient sitting upright
E) A harsh holosystolic murmur that is loudest at the lower left sternal border and radiates
to the left lateral chest wall with the patient in the left lateral decubitus position
ANSWER: C
Aortic regurgitation in an older adult may be due to a congenital bicuspid aortic valve, which often is accompanied by aortic stenosis. Rheumatic aortic valvular disease may also cause aortic regurgitation, which is the most common cause in the developing world but less common in the United States.
The hallmark murmur of aortic regurgitation in either case is a “blowing” decrescendo diastolic murmur along the tract from the aortic valve (upper right sternal border) down to the lower left sternal border, where it is loudest. It is best heard with the patient sitting, leaning forward, and holding his or her breath in expiration.
Both bicuspid aortic valve and rheumatic valve disease may also be associated with aortic stenosis. The typical murmur of aortic stenosis is a mid- to long crescendo/decrescendo systolic murmur, loudest at the right upper sternal border, and often radiating to the carotid arteries. An S3 gallop may also be present in decompensating aortic regurgitation, due to the associated left ventricular dilatation, but this is a secondary finding in later stages.
A 50-year-old male with no significant past medical history presents with a 5-day history of fever to 101°F, chills, and mild diffuse joint and muscle pains. He also reports a mild headache but has not had any sore throat, rhinorrhea, cough, shortness of breath, nausea, vomiting, or change in bowel habits. He noticed a round red rash (shown below) a few days ago on his leg that has grown in size since then. It is minimally pruritic but not painful. He has no other rashes. He recently traveled to Vermont for his family’s annual fall hiking trip but does not recall any insect bites. He does not take any medications and has no drug allergies. He has a blood pressure of 120/74 mm Hg, a pulse rate of 84 beats/min, and a temperature of 37.8°C (100.0°F). Cardiac, pulmonary, musculoskeletal, and abdominal examinations are normal.
Which one of the following is the most likely diagnosis?
A) Adenovirus
B) Ehrlichiosis
C) Influenza
D) Lyme disease
E) Rocky Mountain spotted fever
ANSWER: D
This patient has a classic presentation for Lyme disease. Lyme disease is transmitted by a tick bite, but not all patients remember being bitten by a tick. The classic erythema migrans lesion appears a couple of weeks after the tick bite. The first-line treatment for Lyme disease is either amoxicillin or doxycycline. Macrolides can be used if patients have true allergies to -lactams and doxycycline, but they are less effective. This patient is not exhibiting the respiratory symptoms typically associated with adenovirus. Ehrlichiosis and Rocky Mountain spotted fever typically present with headaches and fevers but not with an erythema migrans rash. Although influenza should be considered in the differential diagnosis, it would not present with an erythema migrans rash.
One of your patients will turn 65 in 2 weeks and your practice manager routinely encourages scheduling a Welcome to Medicare preventive visit soon after patients’ 65th birthdays. This patient continues to work full time and is currently insured through his employer-sponsored health insurance.
In order to bill for a Welcome to Medicare visit after this patient turns 65, which one of the following is true?
A) The patient must be enrolled in Medicare Part A
B) The patient must be enrolled in Medicare Part B
C) The patient must be enrolled in Medicare Part D
D) The patient must be over age 65 and the specific type of Medicare enrollment is not relevant
ANSWER: B
The Welcome to Medicare preventive visit, also known as an Initial Preventive Physical Examination (IPPE), is a one-time service that can be provided within the first year of a patient’s enrollment in Medicare Part B. Medicare Part B covers provider visits and outpatient services such as laboratory testing. Beneficiaries are automatically enrolled in Part A when they apply to Medicare, which provides coverage for hospital-based and hospice care. Because Part A does not typically carry a monthly premium, some working older adults who continue to have insurance through their employer may opt to obtain Part A only, and wait on Part B coverage, which does have a monthly premium. Medicare Part D is prescription drug coverage.
Elevated potassium give
Calcium gluconate
Ottawa knee get xray base on
1) > 55
2) Isolated tenderness of patella
3) Tenderness at head of fibula
4) Inability to flex to 90
5) Inability to bear weight 4 steps
Routine follow-up blood tests for colorectal cancer survivors should include
A) carcinoembryonic antigen (CEA) levels only
B) liver function tests only
C) CBCs and CEA levels only
D) CBCs and liver function tests only
E) CBCs, CEA levels, and liver function tests
ANSWER: A
The Choosing Wisely campaign recommends checking only carcinoembryonic antigen (CEA) levels following curative treatment for colorectal cancer (SOR C). No routine laboratory studies such as a CBC or liver function tests should be ordered for follow-up.
Coccidioides immitis i
present in the desert regions of the U.S. southwest
muscle and joint pain,
Histoplasma
Mississippi and Ohio River valleys of the Midwest and the South.
Muscle and joint pain
A 57-year-old female with a history of diabetes mellitus, hypertension, and depression sees you for a routine follow-up visit. Her vital signs include a heart rate of 88 beats/min, a blood pressure of 162/84 mm Hg, and a BMI of 32 kg/m2.
The recommended antihypertensive regimen for reducing cardiovascular events in this patient is an ACE inhibitor plus
A) an -blocker
B) an angiotensin receptor blocker
C) a calcium channel blocker
D) a loop diuretic
ANSWER: C
The ACCOMPLISH trial demonstrated that an ACE inhibitor (ACEI) in combination with a calcium channel blocker (CCB) reduced both fatal and nonfatal cardiovascular events in patients with diabetes mellitus and hypertension. The benefit of an ACEI and a CCB for reducing cardiovascular events was greater than that of an ACEI and a thiazide diuretic. Evidence has shown that combination therapy for most patients should include a CCB, an ACEI or angiotensin receptor blocker (ARB), or a thiazide diuretic (SOR A). The American College of Cardiology/American Heart Association guidelines recommend against centrally acting medications such as -blockers for first-line therapy. Combining ACEIs and ARBs is not recommended, as the risk of side effects such as hyperkalemia outweighs the benefits. Loop diuretics are not considered first-line antihypertensive agents.
An elderly homeless male is brought to the emergency department. He is clearly hypothermic due to cold exposure and has superficial frostbite of his extremities. He is still conscious and shivering.
In addition to rewarming him, which one of the following should you administer?
A) Acetazolamide (Diamox Sequels)
B) Amitriptyline
C) Ceftriaxone
D) Ibuprofen
E) tPA
ANSWER: D
Frostbite is a freezing injury that occurs when initial cooling causes vasoconstriction and localized ischemia. Continued cold exposure leads to ice crystal formation, which causes cellular lysis, electrolyte abnormalities, and microvascular occlusion. Rewarming creates an inflammatory response. Ibuprofen is the most appropriate agent for the treatment of frostbite until the wounds heal or surgery is performed (SOR C). Acetazolamide can cause frostbite at high altitudes. Amitriptyline is used to treat the pain of immersion foot (also called trench foot), which is a nonfreezing injury that happens when the foot is exposed to prolonged wet conditions above 0°C (32°F). Antibiotics are indicated if open or dirty wounds are present (SOR B). tPA has a role in treating patients with frostbite, but it is used only to decrease the risk of amputation when rewarming patients with grade 3, grade 4, or deep frostbite (SOR B).
positive anti–thyrotropin-receptor (thyroid-stimulating immunoglobulin) antibody
Graves disease
Graves disease
positive anti–thyrotropin-receptor (thyroid-stimulating immunoglobulin) antibody
Medication that can elevated calcium
Lithium
What medication to not use if you have gout
Thiazides (chlorthalidone)
aphthous stomatitis
canker sores
Dose of ceftriaxone for gonorrhea
500 mg IM (< 150 kg)
Testing for ulcerative colitis
Fecal calprotectin in children rules out UC
Diagnosis of alcoholic hepatitis
Acute onset jaundice
Medication to stop drinking
Acamprosate and baclofen
Tx hereditary hemochromoatosis
Lifelong phlebotomy with goal to keep ferritin level around 50 ng per mL
Medication for PAD
Cilostazol
Improved initial and absolute walking distances
Geographic tongue associated with
Benign migratory glossitis
associated with psoriasis
VTE anticoagulation
VTE include PE and DVT
3 months, then start aspirin
Unprovoked VTE should continue anticoagulation indefinitely
Pregnancy and VTE prophylaxis
Postpartum prophylaxis for 6 weeks
Chlamydia tx
Doxycycline 100 mg 2x daily for 7 days
REM sleep behavior disorders are linked to
Antidepressant use and early indicator of Parkinson disease, Lewy body dementia and multiple system atrophy
Medication that can cause RLS
Dopamine antagonists (metoclopramide (Reglan), prochlorperazine (Compazine), droperidol (Inapsine), and promethazine (Phenergan))
Antipsychotics
Serotonergic antidepressants (SSRI)
Ulcerative colitis seen with
Salmonella or Campylobacter infection
First line therapy for UC
Biologics
Tx for severe alcoholic hepatits
Corticosteroids
Tests for alcohol hepatitis
Maddrey Discriminant function tool (at least 32)
Model for end stage liver disease (> 21)
Tx psorasis rash
Triamcinolone acetonide 0.1% cream
Constipation maintenance therapy
for at least 2 months
Leading cause of nonsmoking lung cancer
Radon exposure
Tx NSCLC
Immunotherapy PD-L1 (programmed death ligand 1)
SCLC tx
etoposide (etopopphos) plus cisplatin chemotherapy
Epilepsy criteria
2 unprovoked seizures occurring at least 24 hours apart
Medications that can increase risk of seizure
Wellbutrin
Benadryl (diphenhydramine)
Tramadol
HIV preexposure prophylaxis (PrEP)
emtricitabine and tenofovir alafenamide
Expediated partner therapy for gonorrhea
Cefixime 800 mg
Rapidly enlarging painful nodules and weight loss
Diffuse large B cell lymphoma
Blistering skin lesions, itchy, rapidly progressive
Bullous pemphigoid
Purple reddish blue lesions to legs, genitals, face and oral mucosa
Kaposi sarcoma
Boutonniere deformity aka
Central slip injury, FOced flexion of extended finger at PIP joint
Forced extension of partially flexed finger, inability to flex finger
Jersey finger
Jersey finger
Forced extension of partially flexed finger, inability to flex finger
Mallet finger
Sudden blow to tip of extended finger, drooping or flexed posture at the DIP joint
Mammogram screening
biannual 50-75
Multiple patchy areas of consolidation of CT chest
Interstital lung disease
Medication for sickle cell
hydroxyurea
Positive ANA, next for lupus testing
Low Complement levels
Anticyclic citrullinated peptide antibodies
RA
Anti-smooth muscle antibodies
Autoimmune hepatitis
Anti-centromere antibodies
Systemic sclerosis
HLA-B27
Psoratric arthritis
What to check for presence of severe bacterial infection
Procalcitonin
Weightloss, Low blood pressure, patches of nonpigmented skin on hands, Low sodium, High potassium
Addison’s disease or adrenal insufficency
Patches are vitiligo
Get ACTH suppresion test (low morning cortisol)
17- hydroxyprogesterone deficiency
Congenital adrenal hyperplasia
Late night salivary cortisol
Initial test for cushing syndrome
Dexamethasone confirms disorder
Antibiotic taht increases INR
Trimethoprim/sulfame (Bactrim)
Coral red on woods lamp, TX
Corynebacteria
Red brown amcules
Erythromycin gel
Female hair loss tx
Minoxidil 5% foam indefinitely
Medication for refractory cough not improving
Gabapentin
Medication that can decrease libido
Opioids
SSRi
Measure vit D levels only in
Decreased kidney function
Skeletal disease
Hypercalcemia
Tx Bullous impetigo
Oral trimethoprim/sulfamethoxazole (Bactrim)
diabetes values
Fasting glucose >= 126
2 hour >= 200
Random glucose >= 200
Tumor lysis syndrome
Hyperphosphatemia
Hyperkalemia
Hyperuricemia
Calcium decreased due to binding with free phosphosus
Thumb and index finger pinch nerve
Ulnar
Radial nerve injury
Weakness of wrist extension
Carpal tunnel nerve
Median nerve
Weakness of supination of the forearm nerve
Musculocutanoeus nerve
Most common complication of gastric sleeve
GERD
A 52-year-old male with known hypertension and hyperlipidemia comes to your office for a follow-up visit. His last visit was more than a year ago. He was unemployed for several months and lost his health insurance. Two months ago he ran out of his medications, which included amlodipine (Norvasc), hydrochlorothiazide, and atorvastatin (Lipitor). He says that he feels fine and has not had any chest pain, changes in vision, difficulty breathing, or lower extremity edema. He is a nonsmoker, and he does not drink alcohol or use illicit drugs. He drinks one cup of coffee daily. He does not take any over-the-counter medications.
On examination his vital signs include a blood pressure of 190/120 mm Hg, a pulse rate of 80 beats/min, and an oxygen saturation of 96% on room air. You recheck his blood pressure after he sits quietly for 30 minutes and there is no significant change. A physical examination, including fundoscopy, is normal.
Which one of the following would be the most appropriate next step?
A) Administer clonidine (Catapres), 0.1 mg orally, and recheck his blood pressure in 30 minutes
B) Administer nifedipine (Procardia), 60 mg orally, and recheck his blood pressure in 30 minutes
C) Order laboratory studies to look for end-organ damage, and tell him to restart his previous medications
D) Admit him to the intensive-care unit for intravenous treatment to lower his blood pressure
E) Call 911 and have him transported to the emergency department
ANSWER: C
A hypertensive urgency is defined as a confirmed blood pressure >180/110–120 mm Hg without symptoms or signs of end-organ damage. Patients without symptoms in the setting of severe hypertension rarely have end-organ damage. The most common cause of hypertensive urgency in patients with known hypertension is nonadherence to the use of antihypertensive medications.
When a significantly elevated blood pressure is measured, it should be repeated after 20–30 minutes of quiet rest. Blood pressures should be taken in both arms and a thigh to confirm elevation. One-third of patients with an initially elevated blood pressure will have significantly lower pressure after rest.
Patients who are asymptomatic with persistently elevated blood pressures can be safely treated with oral antihypertensives with close follow-up (SOR C). There is no standard workup for patients with hypertensive urgencies, but common practice includes obtaining a basic metabolic panel, CBC, urinalysis, EKG, and troponin to rule out end-organ damage.
Oral medications to lower blood pressure in a patient with a hypertensive urgency are not indicated unless the patient is symptomatic. Symptoms such as headache or epistaxis warrant acute lowering of blood pressure. Preferred medications include clonidine, labetalol, and captopril, among others. Oral nifedipine is not recommended due to unpredictable blood pressure responses.
Patients with physical or laboratory evidence of end-organ damage should be admitted to the intensive-care unit for intravenous treatment of blood pressure. Without symptoms of end-organ damage there is no need to transport patients to the emergency department, as hypertensive urgencies can be managed with outpatient care.
A 67-year-old male sees you for a Medicare annual wellness visit. He tells you that his best friend had a stroke and he asks about his risk for stroke. He has no history of stroke, TIA, or neurologic symptoms. He has a family history of cardiovascular disease in his father, who had a myocardial infarction at age 65 and died from a thrombotic stroke at age 71. The patient exercises regularly and has a BMI of 27 kg/m2. His only current medical condition is hyperlipidemia, and his cholesterol level is at goal on rosuvastatin (Crestor), 10 mg daily. He also takes aspirin, 81 mg daily. His blood pressure is 125/78 mm Hg.
Based on U.S. Preventive Services Task Force guidelines, which one of the following would be most appropriate at this time?
A) No additional testing for stroke risk
B) Auscultation for carotid bruits
C) Carotid duplex ultrasonography
D) Magnetic resonance angiography
E) CT angiography of the carotid arteries
ANSWER: A
Carotid artery disease affects extracranial carotid arteries and is caused by atherosclerosis. This patient is asymptomatic and has no history of an ischemic stroke, neurologic symptoms referable to the carotid arteries such as amaurosis fugax, or TIA. He has risk factors for cardiovascular disease (age, male sex, hyperlipidemia), but the U.S. Preventive Services Task Force recommends against specific screening for asymptomatic carotid artery stenosis (D recommendation), which has a low prevalence in the general adult population. Stroke is a leading cause of disability and death in the United States, but asymptomatic carotid artery stenosis causes a relatively small proportion of strokes. Auscultation of the carotid arteries for bruits has been found to have poor accuracy for detecting carotid stenosis and is not a reasonable screening approach. Appropriate modalities for detecting carotid stenosis include carotid duplex ultrasonography, magnetic resonance angiography, and computed tomography, but these are not recommended for screening asymptomatic patients.
A 57-year-old female with diabetes mellitus comes to your office for a routine follow-up. Her current medications include metformin (Glucophage), 1000 mg twice daily. She tells you that she does not exercise regularly and finds it difficult to follow a healthy diet. A hemoglobin A1c today is 7.5%. She does not want to add medications at this time, but she does want to get her hemoglobin A1c below 7%, which is the goal that was previously discussed.
Which one of the following would be the most effective way to improve glucose control for this patient?
A) Discuss the components of a healthy diabetic diet and encourage her to follow it more closely
B) Discuss the importance of regular exercise and encourage her to exercise 30–45 minutes daily
C) Recommend that she check her glucose level 1–3 times daily to help determine what adjustments need to be made
D) Start her on an additional medication
E) Refer her to a diabetes educator for medical nutrition therapy
ANSWER: E
Counseling by a diabetic educator or team of educators for medical nutrition therapy lowers hemoglobin A1c by 0.2–0.8 percentage points in patients with type 2 diabetes. While a healthy diabetic diet and regular exercise are important, simply reminding the patient of that fact is not likely to be as successful as comprehensive diabetic education. According to the Society of General Internal Medicine in the Choosing Wisely campaign, patients with type 2 diabetes who are not on insulin therapy should not check their blood glucose level daily. An additional medication will likely decrease the hemoglobin A1c, but this patient has expressed a desire to avoid additional medication, is near goal, and is not currently managing her diabetes with adequate lifestyle changes, so it would be appropriate to respect her wishes and pursue proven interventions that do not require medication.
A 3-year-old male has developed multiple large areas of bullous impetigo on the legs, buttocks, and trunk after being bitten numerous times by ants. Which one of the following would be the most appropriate treatment?
A) Topical mupirocin ointment
B) Oral azithromycin (Zithromax)
C) Oral tetracycline
D) Oral trimethoprim/sulfamethoxazole (Bactrim)
E) Intramuscular penicillin G benzathine (Bicillin L-A)
ANSWER: D
Impetigo may be caused by Streptococcus pyogenes or Staphylococcus aureus, but bullous impetigo is caused exclusively by S. aureus. Oral trimethoprim/sulfamethoxazole is an appropriate treatment for skin infections caused by S. aureus, including susceptible cases of methicillin-resistant S. aureus (MRSA). Topical mupirocin ointment is not practical in very widespread cases or in cases with large bullae. Neither azithromycin nor penicillin is a preferred treatment for impetigo, due to a high rate of treatment failure. Tetracycline should be avoided in children under 8 years of age due to a propensity to cause permanent staining of the teeth.
A 57-year-old male recently diagnosed with acute lymphoblastic leukemia presents to the emergency department with intractable nausea, vomiting, and myalgias. His first chemotherapy infusion was administered earlier in the day.
Which one of the following electrolyte disturbances would be consistent with tumor lysis syndrome?
A) Hypocalcemia B) Hypokalemia C) Hyponatremia
D) Hypophosphatemia E) Hypouricemia
ANSWER: A
Tumor lysis syndrome is a common complication of chemotherapy in hematologic malignancies, such as acute leukemia. Homeostasis is overwhelmed with phosphorus, potassium, calcium, and uric acid released into the bloodstream due to acute cell lysis. Hyperphosphatemia, hyperkalemia, and hyperuricemia are indicative of tumor lysis syndrome. Calcium levels are decreased due to binding with free phosphorus and a depletion of calcium in the bloodstream. Sodium electrolyte levels are not as likely to be affected.
A 30-year-old male is diagnosed with hereditary hemochromatosis. Periodic therapeutic phlebotomy may be appropriate to prevent
A) chronic liver disease
B) chronic renal disease
C) encephalopathy
D) myelofibrosis
E) Wilson disease
ANSWER: A
Hereditary hemochromatosis is a common inherited disorder of iron metabolism. Iron deposits in the liver may lead to chronic liver disease and hepatocellular cancer. Screening for hereditary hemochromatosis includes serum ferritin levels, a family history, and genetic testing. Chronic renal disease, encephalopathy, myelofibrosis, and Wilson disease (disorder of copper transport) do not result from iron overload.
After a thorough history and examination you determine that a 30-year-old male has an upper respiratory infection with a persistent cough. He is afebrile and is otherwise healthy.
The best treatment for symptomatic relief of his persistent cough would be intranasal
A) antibiotics
B) antihistamines
C) corticosteroids
D) ipratropium (Atrovent)
E) saline
ANSWER: D
Upper respiratory tract infections are the most common acute illness in the United States. Symptoms are self-limited and can include nasal congestion, rhinorrhea, sore throat, cough, general malaise, and a low-grade fever. According to a Cochrane review of 10 trials without a meta-analysis, antitussives and expectorants are no more effective than placebo for cough. Intranasal ipratropium is the only medication that improves persistent cough related to upper respiratory infection in adults. Intranasal antibiotics, antihistamines, corticosteroids, and saline would not improve this patient’s cough.
A 30-year-old female presents for evaluation of chronic abdominal bloating, cramping, diarrhea, and recent weight loss. An abdominal examination is unremarkable, and stool guaiac testing is negative. She requests testing for celiac disease.
Which one of the following would be most likely to cause a false-negative result on serologic testing for celiac disease?
A) A recent increase in dietary wheat consumption
B) Recent use of loperamide (Imodium A-D)
C) A skin rash consistent with dermatitis herpetiformis
D) IgA deficiency
E) Iron deficiency anemia
ANSWER: D
Celiac disease is a chronic malabsorptive disorder with an estimated worldwide prevalence of 1.4%. The preferred initial diagnostic test includes a serum IgA transglutaminase-2 (TG2) antibody level, which has a 98% sensitivity and 98% specificity for the diagnosis of celiac disease. False-negative serologic results may occur in patients with an IgA deficiency, which includes up to 3% of patients with celiac disease. Therefore, when a diagnosis of celiac disease is strongly suspected despite a negative IgA TG2 antibody test, a total IgA level should be obtained. Diagnostic confirmation for patients with positive serologic testing is accomplished with endoscopic mucosal biopsy.
Dietary elimination of gluten, not an increase in gluten intake, prior to serologic testing may lead to false-negative results. Recent use of medications, including loperamide, would not be expected to interfere with the accuracy of serologic testing for celiac disease. Dermatitis herpetiformis is a widespread pruritic papulovesicular rash that occurs in less than 10% of patients with celiac disease, although is essentially pathognomonic for the condition, as nearly all patients with this rash have evidence of celiac disease on an intestinal biopsy. Iron deficiency anemia often occurs in patients with celiac disease due to poor iron absorption, although the presence of iron deficiency anemia does not decrease the sensitivity of serologic testing.
A 20-year-old male presents with a painful second finger after his right hand was stepped on 3 days ago while he was playing basketball. He has marked pain as well as numbness of the distal finger. There are no open wounds and the skin color and nail appear normal other than moderate edema of the fingertip. A radiograph reveals a distal phalanx fracture.
Which one of the following would be the most appropriate next step?
A) Treat symptomatically with ice and an anti-inflammatory medication
B) Tape the first and second digits together until symptoms resolve
C) Splint the affected digit for 2–4 weeks
D) Remove the nail to evaluate for a nail bed injury
E) Refer to a hand surgeon
ANSWER: C
Tuft fractures are the most common type of distal phalanx fracture. They rarely require orthopedic referral but often result in up to 6 months of hyperesthesia, pain, and numbness. Treatment involves splinting the affected digit for 2–4 weeks, followed by range of motion and strengthening exercises. Symptomatic treatment may also be involved, but splinting is needed. Taping digits would likely not provide enough stability for the second digit distal phalanx, which extends beyond the first digit. Patients with distal finger injuries need careful physical examination to evaluate for a nail bed injury, but in this case there is no evidence of nail bed damage or laceration.
A 72-year-old female with a history of type 2 diabetes and hypertension presents to your clinic because of fatigue and depression for the last 5–6 months. She has gained about 7 kg (15 lb) and now has a BMI of 32 kg/m2. A physical examination is otherwise unremarkable. Laboratory studies reveal a TSH level of 8.2 U/mL (N 0.4–4.0). A repeat test 1 month later shows a TSH level of 7.4 U/mL and a free T4 level of 1.6 ng/dL (N 0.8–2.8).
Treatment of this patient with L-thyroxine
A) has no proven benefit
B) can increase grip strength
C) can increase her energy level
D) can help improve depression symptoms
E) can reduce her BMI
ANSWER: A
Subclinical hypothyroidism (SCH) is defined as an elevation in TSH level with a normal free T4 level. It is relatively common in adults over the age of 65, with a prevalence of 20%. The TRUST (Thyroid Hormone Replacement for Subclinical Hypothyroidism) trial and subsequent meta-analyses of randomized, controlled trials demonstrate that there is no benefit in treating SCH. Symptoms such as muscle strength, fatigue or tiredness, depression, and BMI do not improve with L-thyroxine treatment (SOR A), and up to 60% of cases resolve within 5 years without intervention in older adults.
Appropriate management of an elevated TSH level includes repeat testing in 1–3 months along with a free T4 level. If SCH is diagnosed, levels should be monitored yearly. Only 2%–4% of patients with SCH develop overt hypothyroidism.
A 72-year-old male with a history of hypertension, heart failure, and chronic kidney disease sees you for evaluation of gradually worsening lumbar pain. The pain worsens with walking but improves when he sits. He says that the pain radiates to the buttocks and down the right leg, especially with activity. He has not had any fevers, chills, or new urinary symptoms. MRI indicates severe degenerative changes resulting in moderate to severe canal stenosis at the L4-L5 level.
Which one of the following would be most appropriate at this point?
A) Oral diclofenac, 75 mg twice daily
B) Oral pregabalin (Lyrica), 75 mg twice daily
C) Physical therapy
D) Referral to an orthopedic surgeon for elective surgical resolution
E) Referral to a neurosurgeon for urgent surgical resolution
ANSWER: C
Lumbar spinal stenosis is a common cause of low back pain in older adults, with varying reports of prevalence but at least 10% in most studies. It is the most common reason for lumbar spinal surgery in the United States. Management of this condition is delayed due to the lack of strong evidence for definitively efficacious nonsurgical approaches, and by high rates of major complications with surgical approaches. Focused physical therapy has the best evidence for initial management. Given this patient’s cardiac and renal comorbidities, chronic use of oral NSAIDs is likely to cause significant harm. While some oral pain medications may be considered, pregabalin has not been found to be any more effective than placebo. Both orthopedic and neurosurgical subspecialists perform lumbar spinal surgeries across the United States. In this case, there is no indication for urgent or emergent surgical management. Given the high complication rate, elective surgical management should be considered only after more conservative options have been found ineffective.
A 42-year-old female presents to your office to discuss bariatric surgery and its potential complications. Her BMI is 40 kg/m2 and she has hypertension, type 2 diabetes, and osteoarthritis of both knees.
If she opts to have a sleeve gastrectomy, which one of the following complications is most likely in the first 6 months?
A) Cholelithiasis
B) Dumping syndrome
C) GERD
D) Leaking at the surgical site
E) Small bowel obstruction
ANSWER: C
Sleeve gastrectomy is currently the most common bariatric procedure. The most common complication is development of GERD, which occurs in 20% of patients. Since this procedure does not produce a malabsorption component, complications such as cholelithiasis, dumping syndrome, and small bowel obstruction are not as likely as with other available procedures. A postoperative leak develops in <2% of cases.
A sleeve gastrectomy involves removing the majority of the greater curvature of the stomach, which creates a tubular stomach. Roux-en-Y gastric bypass and biliopancreatic diversion with duodenal switch both combine volume restriction and nutrient malabsorption.
A 14-year-old female is brought to your office for a well child check and a sports physical examination. During the substance abuse screening she says that she does not drink alcohol or smoke marijuana or traditional cigarettes, but occasionally uses e-cigarettes with her friends.
Which one of the following statements describing e-cigarettes is true?
A) E-cigarettes do not contain heavy metals
B) The nicotine in e-cigarettes is not addictive
C) Teens who use e-cigarettes are less likely to use marijuana
D) More teens use traditional cigarettes than e-cigarettes
E) E-cigarette use among teens increases the likelihood of cigarette smoking
ANSWER: E
E-cigarette use has become quite popular among youth in the United States, with rates surpassing traditional cigarette use in 2014. Among teens who have never smoked, the odds of cigarette smoking are 3–6 times higher in those who have used e-cigarettes within the last year. Nicotine is highly addictive regardless of the source, and heavy metal toxicants are still present when using e-cigarettes, although less than with traditional cigarettes. E-cigarette use is associated with an increased risk of future marijuana use.
A 52-year-old male with known hypertension and hyperlipidemia comes to your office for a follow-up visit. His last visit was more than a year ago. He was unemployed for several months and lost his health insurance. Two months ago he ran out of his medications, which included amlodipine (Norvasc), hydrochlorothiazide, and atorvastatin (Lipitor). He says that he feels fine and has not had any chest pain, changes in vision, difficulty breathing, or lower extremity edema. He is a nonsmoker, and he does not drink alcohol or use illicit drugs. He drinks one cup of coffee daily. He does not take any over-the-counter medications.
On examination his vital signs include a blood pressure of 190/120 mm Hg, a pulse rate of 80 beats/min, and an oxygen saturation of 96% on room air. You recheck his blood pressure after he sits quietly for 30 minutes and there is no significant change. A physical examination, including fundoscopy, is normal.
Which one of the following would be the most appropriate next step?
A) Administer clonidine (Catapres), 0.1 mg orally, and recheck his blood pressure in 30 minutes
B) Administer nifedipine (Procardia), 60 mg orally, and recheck his blood pressure in 30 minutes
C) Order laboratory studies to look for end-organ damage, and tell him to restart his previous medications
D) Admit him to the intensive-care unit for intravenous treatment to lower his blood pressure
E) Call 911 and have him transported to the emergency department
ANSWER: C
A hypertensive urgency is defined as a confirmed blood pressure >180/110–120 mm Hg without symptoms or signs of end-organ damage. Patients without symptoms in the setting of severe hypertension rarely have end-organ damage. The most common cause of hypertensive urgency in patients with known hypertension is nonadherence to the use of antihypertensive medications.
When a significantly elevated blood pressure is measured, it should be repeated after 20–30 minutes of quiet rest. Blood pressures should be taken in both arms and a thigh to confirm elevation. One-third of patients with an initially elevated blood pressure will have significantly lower pressure after rest.
Patients who are asymptomatic with persistently elevated blood pressures can be safely treated with oral antihypertensives with close follow-up (SOR C). There is no standard workup for patients with hypertensive urgencies, but common practice includes obtaining a basic metabolic panel, CBC, urinalysis, EKG, and troponin to rule out end-organ damage.
Oral medications to lower blood pressure in a patient with a hypertensive urgency are not indicated unless the patient is symptomatic. Symptoms such as headache or epistaxis warrant acute lowering of blood pressure. Preferred medications include clonidine, labetalol, and captopril, among others. Oral nifedipine is not recommended due to unpredictable blood pressure responses.
Patients with physical or laboratory evidence of end-organ damage should be admitted to the intensive-care unit for intravenous treatment of blood pressure. Without symptoms of end-organ damage there is no need to transport patients to the emergency department, as hypertensive urgencies can be managed with outpatient care.
A 28-year-old female presents with a depressed mood and sleep disturbance. She tells you that this has occurred for the past 4 years but only during the winter months. Her past medical history and a physical examination are unremarkable.
Which one of the following interventions has the strongest evidence for preventing recurrence of her condition?
A) Exercise
B) Light therapy
C) Cognitive-behavioral therapy
D) Bupropion (Wellbutrin XL)
E) Fluoxetine (Prozac)
ANSWER: D
This patient has seasonal affective disorder (SAD) that has recurred and is likely to continue to recur. Bupropion is the only medication beneficial for prevention of SAD. Light therapy and SSRIs are helpful for treating this disorder but do not prevent it. Exercise and cognitive-behavioral therapy are beneficial adjuncts to treatment but would not prevent recurrence.
An 85-year-old female with a previous history of diabetes mellitus, hypertension, dementia, and peptic ulcer disease has been in a skilled nursing facility for 4 weeks for rehabilitation after a hip fracture repair secondary to a fall during an ischemic stroke. She is transported to the emergency department today when she develops confusion, shortness of breath, and diaphoresis. Her blood pressure is 172/98 mm Hg, her heart rate is 122 beats/min with an irregular rhythm, and her respiratory rate is 22/min. An EKG demonstrates atrial fibrillation and 0.2 mV ST-segment elevation compared to previous EKGs. Her first troponin level is elevated.
Which one of the following conditions in this patient is considered an ABSOLUTE contraindication to fibrinolytic therapy?
A) Poorly controlled hypertension
B) Peptic ulcer disease
C) Alzheimer’s dementia
D) Hip fracture repair
E) Ischemic stroke
ANSWER: E
A history of an ischemic stroke within the past 3 months is an absolute contraindication to fibrinolytic therapy in patients with an ST-elevation myocardial infarction (STEMI), unless the stroke is diagnosed within 41⁄2 hours. Poorly controlled hypertension, dementia, peptic ulcer disease, and major surgery less than 3 weeks before the STEMI are relative contraindications that should be considered on an individual basis.
You are the team physician for the local high school track team. During a meet one of the athletes inadvertently steps off the edge of the track and inverts her right foot forcefully. She is able to bear weight but with significant pain. She reports pain across her right midfoot. An examination reveals edema over the lateral malleolus and diffuse tenderness, but she does not have any pain with palpation of the navicular, the base of the fifth metatarsal, or the posterior distal lateral and medial malleoli.
Which one of the following would be most appropriate at this time?
A) Radiographs of the right ankle only
B) Radiographs of the right foot only
C) Radiographs of the right foot and ankle
D) Lace-up ankle support, ice, compression, and clinical follow-up
E) Crutches and no weight bearing for 2 weeks, followed by a slow return to weight
bearing
ANSWER: D
The Ottawa foot and ankle rules should be used to determine the need for radiographs in foot and ankle injuries. A radiograph of the ankle is recommended if there is pain in the malleolar zone along with the inability to bear weight for at least four steps immediately after the injury and in the physician’s office or emergency department (ED), or tenderness at the tip of the posterior medial or lateral malleolus. A radiograph of the foot is recommended if there is pain in the midfoot zone along with the inability to bear weight for four steps immediately after the injury and in the physician’s office or ED, or tenderness at the base of the fifth metatarsal or over the navicular bone. The Ottawa foot and ankle rules are up to 99% sensitive for detecting fractures, although they are not highly specific. In this case there are no findings that would require radiographs, so treatment for the ankle sprain would be recommended. Compression combined with lace-up ankle support or an air cast, along with cryotherapy, is recommended and can increase mobility. Early mobilization, including weight bearing as tolerated for daily activities, is associated with better long-term outcomes than prolonged rest.
A 65-year-old male with type 2 diabetes mellitus, hypertension, and obstructive sleep apnea sees you for follow-up. He does not use tobacco or other drugs, and his alcohol consumption consists of two drinks per day. His BMI is 31.0 kg/m2, and he just started a fitness program. The patient tells you that his brother was recently diagnosed with atrial fibrillation and he asks you if this increases his own risk.
Which one of the following factors would increase the risk of atrial fibrillation in this patient?
A) Alcohol use
B) Treatment with lisinopril (Prinivil, Zestril)
C) Treatment with pioglitazone (Actos)
D) Use of a continuous positive airway pressure (CPAP) device
E) Physical stress
ANSWER: A
Alcohol consumption greater than one drink/day has been associated with atrial fibrillation. While not recommended to prevent atrial fibrillation, pioglitazone and lisinopril have both been associated with lower rates of atrial fibrillation compared to alternative therapies. Treatment of obstructive sleep apnea, along with a regular fitness regimen, has been associated with a decrease in the recurrence of atrial fibrillation.
You see an adult patient who has chronic urticaria and no other known chronic conditions. He continues to experience hives after a 3-month course of daily loratadine (Claritin).
Which one of the following would be the most appropriate addition to his treatment regimen at this time?
A) A short course of oral corticosteroids
B) Cyclosporine
C) Ranitidine (Zantac)
D) Narrow-band UV light treatment
ANSWER: C
First- and second-generation H1 antihistamine receptor antagonists are generally considered first-line treatment for chronic urticaria, and approximately 60% of patients experience a satisfactory result. Second-generation options such as loratadine have the added benefit of a lower likelihood of side effects such as drowsiness. For those who fail to achieve the desired result with monotherapy using an H1 antihistamine receptor antagonist, the addition of an H2 antihistamine receptor antagonist such as cimetidine or ranitidine is often beneficial. The tricyclic antidepressant doxepin has strong H1 and H2 antihistamine receptor antagonist effects and has been used as an off-label treatment option in some studies. A short course of oral corticosteroids, narrow-band UV light treatment, or cyclosporine can be used in the management of recalcitrant chronic urticaria, but these are considered second- or third-line adjunctive options.
A 62-year-old female with diabetes mellitus presents to your office with left lower quadrant pain and guarding. She has a previous history of a shellfish allergy that caused hives and swelling.
Further evaluation of this patient should include which one of the following?
A) Ultrasonography of the abdomen
B) CT of the abdomen and pelvis with oral and intravenous (IV) contrast
C) Oral corticosteroids and antihistamines, then CT of the abdomen and pelvis with oral
and IV contrast
D) Intravenous corticosteroids and antihistamines, then CT of the abdomen and pelvis
with oral and IV contrast
E) Laparotomy
ANSWER: B
Evaluation of this patient should include CT of the abdomen and pelvis with oral and intravenous (IV) contrast. There is no reason to inquire about shellfish allergies prior to CT with IV contrast, because premedication is not needed. There is no correlation between shellfish allergies and allergic reactions to contrast. Patients with moderately severe to severe reactions to IV contrast in the past would need pretreatment with corticosteroids.
A 15-year-old female presents with a 3-month history of intermittent abdominal pain and headaches. She does not have any associated weight loss, fever, nausea, change in bowel habits, or other worrisome features. An examination is unremarkable. She does report being stressed at school and has a PHQ-2 score of 4.
Which one of the following would be most appropriate at this point?
A) Further evaluation for depression
B) Laboratory studies
C) Abdominal imaging
D) Citalopram (Celexa)
E) Fluoxetine (Prozac)
ANSWER: A
The U.S. Preventive Services Task Force recommends depression screening for all adolescents 12–18 years of age. Although this patient has abdominal pain, the history and physical examination suggest that depression may be playing a role in her somatic complaints. She had a positive initial depression screen on her PHQ-2. This is a brief screening tool, and a positive result merits further evaluation. The evaluation should include a full PHQ-A or a discussion with a qualified clinician. If the patient meets the criteria for major depressive disorder then she should receive treatment for her depression, which could include medication. Both fluoxetine and citalopram have been approved by the FDA to treat depression in this age group. She could also be referred for psychotherapy. Further laboratory studies and imaging may be appropriate at some point, but the most urgent need is to evaluate her positive depression screen.
A 2-year-old African-American male with a history of sickle cell disease is brought to your office for a well child check. Which one of the following would be most appropriate for screening at this time?
A) A chest radiograph
B) A DXA scan
C) Abdominal ultrasonography
D) Renal Doppler ultrasonography
E) Transcranial Doppler ultrasonography
ANSWER: E
Individuals with sickle cell disease are at increased risk for vascular disease, especially stroke. All sickle cell patients 2–16 years of age should be screened with transcranial Doppler ultrasonography (SOR A). A chest radiograph, abdominal ultrasonography, a DXA scan, and renal Doppler ultrasonography are not recommended for screening patients with sickle cell disease.
A 32-year-old female requests a physical examination prior to participating in an adult soccer league. Her blood pressure is 118/70 mm Hg and her pulse rate is 68 beats/min. The examination is otherwise normal except for a systolic murmur that intensifies with Valsalva maneuvers. She says that she has recently been experiencing mild exertional dyspnea and moderate chest pain. The chest pain has been atypical and is not necessarily related to exertion. Echocardiography reveals hypertrophic cardiomyopathy.
In addition to referring the patient to a cardiologist, you recommended initiating therapy with
A) amiodarone (Cordarone)
B) amlodipine (Norvasc)
C) furosemide (Lasix)
D) lisinopril (Prinivil, Zestril)
E) metoprolol
ANSWER: E
Hypertrophic cardiomyopathy is the most common primary cardiomyopathy, with a prevalence of 1:500 persons. Many patients with hypertrophic cardiomyopathy are asymptomatic and are diagnosed during family screening, by auscultation of a heart murmur, or incidentally after an abnormal result on electrocardiography. On examination physicians may hear a systolic murmur that increases in intensity during Valsalva maneuvers. The main goals of therapy are to decrease exertional dyspnea and chest pain and prevent sudden cardiac death. -Blockers are the initial therapy for patients with symptomatic hypertrophic cardiomyopathy. Nondihydropyridine calcium channel blockers such as verapamil can be used if -blockers are not well tolerated.
A 62-year-old female who is a new patient requests a thyroid evaluation because she has a history of abnormal thyroid test results. You obtain a copy of her records, which include a TSH level of 0.2 U/mL (N 0.4–4.2) and a free T4 level of 2.0 ng/dL (N 0.8–2.7) from 3 years ago. She reports feeling well and has no other health conditions. She does not take any medications.
A physical examination reveals normal vital signs, a BMI of 23.0 kg/m2, no neck masses, a normal thyroid size, and normal heart sounds. Laboratory studies reveal a TSH level of 0.1 U/mL, a free T4 level of 2.5 ng/dL, and a free T3 level of 3.1 pg/mL (N 2.3–4.2).
Treatment for this condition would be indicated if the patient has an abnormal
A) calcium level
B) DXA scan
C) glucose level
D) lipid level
E) thyroid ultrasonography study
ANSWER: B
This patient has subclinical hyperthyroidism as evidenced by her low TSH level with normal free T4 and free T3 levels. Common causes of subclinical hyperthyroidism include Graves disease, autonomous functioning thyroid adenoma, and multinodular toxic goiter. Subclinical hyperthyroidism may progress to overt hyperthyroidism; this is more likely in patients with TSH levels <0.1 U/mL. Even in the absence of overt hyperthyroidism these patients are at higher risk for several health conditions, including atrial fibrillation, heart failure, and osteoporosis. For this reason it is important to assess for these conditions and consider treating the underlying thyroid condition, as well as the complication. The American Thyroid Association recommends treating patients with complications who are either over age 65 or have a TSH level <0.1 U/mL.
Lipid and glucose abnormalities are not known to be related to subclinical hyperthyroidism. Calcium levels may be abnormal in hyperparathyroidism but not hyperthyroidism. Thyroid ultrasonography may be helpful to determine the cause of hyperthyroidism but is not used to help decide when to treat subclinical hyperthyroidism.
Intensive behavioral intervention has more benefit than other treatment modalities in treating children who have been diagnosed with
A) attention-deficit/hyperactivity disorder
B) autism
C) depression
D) obsessive-compulsive disorder
E) posttraumatic stress disorder
ANSWER: B
The only evidence-based treatment that confers significant benefits to children with autism is intensive behavioral interventions, which should be initiated before 3 years of age. Attention-deficit/hyperactivity disorder can be treated with cognitive-behavioral therapy (CBT) but medication is often required. CBT is as effective, if not more effective, than medication for treating anxiety, depression, and trauma-related disorders.
You are notified by the nurse that a 66-year-old female who was admitted for pain control for her bone metastases is still having breakthrough pain. You gave her 10 mg of immediate-release oxycodone (Roxicodone) 15 minutes ago.
You are hoping to optimize pain control and minimize sedation, so you advise the nurse that the last dose will have its peak effect
A) now
B) 1 hour after it was given
C) 2 hours after it was given
D) 4 hours after it was given
ANSWER: B
Most orally administered immediate-release opioids such as morphine, oxycodone, and hydromorphone reach their peak effect at about 1 hour, at which time additional medication can be given if the patient is still in pain. Intravenous opioids reach their peak effect at about 10 minutes and intramuscular and subcutaneous opioids at about 20–30 minutes. Additional medication may therefore be given at those intervals if additional pain relief is required.
A 62-year-old female has a history of COPD graded as moderate on pulmonary function testing, with an FEV1 of 65% of predicted and a PaO2 of 57 mm Hg. Because her symptoms of dyspnea on exertion and fatigue seem out of proportion to her pulmonary function tests, you order echocardiography, which shows a pulmonary artery systolic pressure of 50 mm Hg, indicating pulmonary hypertension.
Which one of the following would be most effective for decreasing mortality in this situation?
A) Supplemental oxygen
B) An endothelin receptor antagonist such as bosentan (Tracleer)
C) A calcium channel blocker such as nifedipine (Procardia)
D) A phosphodiesterase 5 inhibitor such as sildenafil (Revatio)
E) Referral for pulmonary artery endarterectomy
ANSWER: A
The only proven therapy for pulmonary hypertension related to COPD is supplemental oxygen. Supplemental oxygen should be recommended when the PaO2 is <60 mm Hg, because it has been shown to improve mortality by lowering pulmonary arterial pressures. Treatments effective for pulmonary artery hypertension should not be used. Pulmonary vasodilators such as nifedipine, sildenafil, and bosentan may cause a ventilation-perfusion mismatch. Pulmonary endarterectomy may be indicated for pulmonary hypertension caused by chronic thromboembolic disease.
A 13-year-old male sees you because of pain in his throwing arm. He is a very dedicated football quarterback and has been practicing throws and playing games every day for 2 months. The pain started gradually over the season, and there is no history of acute injury. The patient is right-hand dominant, and on examination he has pain when he raises his right arm above his shoulder. There is also tenderness to palpation of the proximal and lateral humerus.
Which one of the following would be most appropriate at this point?
A) Injection of 10 mL of lidocaine into the subacromial space
B) Plain radiographs of the shoulder
C) Ultrasonography of the supraspinatus muscle
D) MRI of the shoulder
E) A bone scan of the shoulder
ANSWER: B
Pain in the shoulder of a young athlete can be caused by many problems, including acromioclavicular strain, biceps tendinitis, glenohumeral instability, and rotator cuff pathology. Although rotator cuff pathologies are the most frequent cause of shoulder pain in adults, they are uncommon in children. Unique to children, however, is a repetitive use injury causing disruption at the proximal growth plate of the humerus. This condition is referred to as Little League shoulder and can be seen on plain radiographs as widening, demineralization, or sclerosis at the growth plate. If the radiograph is normal but suspicion for this condition is high, a bone scan or MRI can be ordered.
You admit a previously healthy 62-year-old female to the hospital for intractable nausea and vomiting with intravascular volume depletion and hypotension. She lives in rural northern New Mexico. Prior to the onset of her symptoms she had been gardening and cleaning out a chicken coop, where she encountered several rodents. She is febrile and you obtain blood and urine cultures. Two out of four blood culture bottles are positive for gram-negative rods.
Which one of the following is the most likely pathogen?
A) Brucella melitensis
B) Coxiella burnetii
C) Escherichia coli
D) Listeria monocytogenes
E) Yersinia pestis
Yersinia pestis is an aerobic fermentative gram-negative rod. It causes a zoonotic infection with humans as the accidental host. The disease is spread by a bite from a flea vector, direct contact with infected tissue, or inhalation of infectious aerosols from a person with pulmonary plague. Plague occurs in two regions in the western United States. One region includes northern New Mexico, northern Arizona, and southern Colorado, and the other region includes California, southern Oregon, and far western Nevada.
Escherichia coli is also an aerobic fermentative gram-negative rod but it generally causes symptoms of gastroenteritis, hemolytic-uremic syndrome, urinary tract infection, intra-abdominal infection, and meningitis. E. coli infection does not have a specific regional distribution. Listeria monocytogenes is a gram-positive rod and causes an influenza-like illness with or without gastroenteritis in adults. Infection occurs through ingestion of contaminated food products such as milk, cheese, processed meats, and raw vegetables. Outbreaks can occur in any geographic distribution.
Coxiella burnetii is a gram-negative intracellular bacterium that causes Q fever. Human infections are associated with contact with infected cattle, sheep, goats, dogs, and cats. Brucella melitensis is a gram-negative coccobacilli that causes brucellosis. Humans are accidental hosts who can develop the disease from contact with tissues rich in erythritol, and from shedding of organisms in milk, urine, and birth products from goats and sheep.
A 21-year-old female is being evaluated for secondary causes of refractory hypertension. Which one of the following would be most specific for fibromuscular dysplasia?
A) A serum creatinine level
B) An aldosterone:renin ratio
C) 24-hour urine for metanephrines
D) Renal ultrasonography
E) Magnetic resonance angiography of the renal arteries
ANSWER: E
In young adults diagnosed with secondary hypertension, evaluation for fibromuscular dysplasia of the renal arteries with MR angiography or CT angiography is indicated (SOR C). The aldosterone/renin ratio is the most sensitive test to diagnose primary hyperaldosteronism. Renal ultrasonography is an indirect test that is not as sensitive or specific for fibromuscular dysplasia. Serum creatinine elevation shows renal involvement but does not identify the cause. Testing for metanephrines is indicated only if a pheochromocytoma is suspected.
The novel anticoagulants (NOACs) include apixaban (Eliquis), dabigatran (Pradaxa), edoxaban (Savaysa), and rivaroxaban (Xarelto). Which one of the following should be considered when starting or adjusting the dosage of a NOAC?
A) Serum albumin
B) INR
C) Liver enzymes
D) Partial thromboplastin time
E) Renal function
ANSWER: E
Because debilitating knee osteoarthritis is a frequent health concern in older adults, physicians should try to identify and possibly modify factors that increase the risk for this condition. Pooled data from many large studies has been sufficient to clearly identify several major risk factors for the development and progression of osteoarthritis of the knees. Overweight and obesity have consistently been found to approximately double the risk for developing knee osteoarthritis. Other factors that have been identified as risk factors include female sex, advancing age (50–75 years of age), and previous trauma. Smoking, inactivity, moderate physical activity, and socioeconomic status have not been shown to affect one’s risk for developing knee osteoarthritis. However, any of these factors in the extreme may be detrimental to joint health in general.
A 20-year-old football player presents with pain in the proximal fifth metatarsal. The pain was initially present only after practices, but now it causes push-off pain during practice. There is tenderness to palpation. Plain films show no signs of fracture.
Which one of the following would be most appropriate at this point?
A) Start NSAIDs and allow him to continue practicing as tolerated
B) Place him at non–weight bearing for 2 weeks and repeat the plain films
C) Place him in a hard shoe for 3 weeks and then reexamine
D) Order MRI of the foot
E) Order a bone scan of the foot
ANSWER: D
A stress fracture in the proximal fifth metatarsal is particularly prone to nonunion and completion of the fracture. Because complete non–weight bearing or surgical intervention may be necessary with this high-risk fracture, MRI is indicated as the most sensitive test. Bone scans are sensitive but nonspecific. Most stress fractures of the metatarsals occur distally and can be managed with a hard shoe initially, with progressive activity as tolerated. NSAIDs are discouraged because of possible effects on fracture healing.
An 84-year-old female with severe dementia due to Alzheimer’s disease is a resident of a long-term care facility. She has been hitting the staff while receiving personal care and recently had an altercation with another resident. Behavioral interventions have been unsuccessful in managing her symptoms and you suggest to the patient’s family that she be started on low-dose risperidone (Risperdal). They ask about appropriate use of the drug and the potential for side effects.
Which one of the following would be appropriate advice?
A) Extrapyramidal side effects are more common compared to typical antipsychotics
B) Dementia-related psychosis is an FDA-approved indication
C) No monitoring will be necessary
D) The risk of diabetes mellitus is decreased
E) The risk of mortality is increased
ANSWER: E
Both typical and atypical antipsychotics increase the risk of mortality in patients with dementia. The FDA has a black box warning on these medications, including risperidone, about the increased risk of mortality in patients with dementia. Risperidone is not approved by the FDA for dementia-related psychosis. The typical antipsychotics are more commonly associated with extrapyramidal side effects. Diabetes mellitus and agranulocytosis are associated with the atypical antipsychotics, including risperidone. Periodic monitoring of serum glucose levels and CBCs is recommended.
Which one of the following diabetes mellitus medications is MOST likely to cause weight gain?
A) Empagliflozin (Jardiance)
B) Glimepiride (Amaryl)
C) Liraglutide (Victoza)
D) Metformin (Glucophage)
E) Sitagliptin (Januvia)
ANSWER: B
Since many patients with diabetes mellitus are obese, the impact of medications on the patient’s weight is important to consider. Treatment with sulfonylureas, including glimepiride, is associated with weight gain. Empagliflozin, liraglutide, metformin, and sitagliptin are not associated with weight gain. In particular, the SGLT2 inhibitors such as empagliflozin and the GLP1 agonists such as liraglutide are associated with clinically significant weight loss.
A 52-year-old male presents for evaluation of a long-standing facial rash. He reports that the rash is itchy, with flaking and scaling around his mustache and nasolabial folds.
Which one of the following is most likely to be beneficial?
A) Topical antibacterial agents
B) Topical antifungal agents
C) Topical vitamin D analogues
D) Oral zinc supplementation
ANSWER: B
Seborrheic dermatitis is commonly seen in the office setting and affects the scalp, eyebrows, nasolabial folds, and anterior chest. The affected skin appears as erythematous patches with white to yellow greasy scales. The etiology is not exactly known, but it is likely that the yeast Malassezia plays a role. Topical antifungals are effective and recommended as first-line agents. Topical low-potency corticosteroids are also effective alone or when used in combination with topical antifungals, but they should be used sparingly due to their adverse effects. The other agents listed have no role in the management of seborrheic dermatitis (SOR A).
A 52-year-old male sees you for a routine health maintenance examination. He does not take any medications, does not drink alcohol, and is feeling well. A physical examination is normal with the exception of a BMI of 33.2 kg/m2. Routine laboratory studies reveal mild elevations of ALT (SGPT) and AST (SGOT), which remain elevated on repeat testing 2 months later. Hepatitis B and hepatitis C testing are negative.
In addition to ultrasonography of the liver, which one of the following laboratory studies should be ordered to further evaluate this patient?
A) Serum ferritin
B) Serum phosphorus
C) -Fetoprotein
D) Carcinoembryonic antigen (CEA)
E) Serum protein electrophoresis
ANSWER: A
Mild asymptomatic elevations (<5 times the upper limit of normal) of ALT and AST are common in primary care. It is estimated that approximately 10% of the U.S. population has elevated transaminase levels. The most common causes of elevated transaminase levels are nonalcoholic fatty liver disease and alcoholic liver disease. The initial evaluation should include assessment for metabolic syndrome and insulin resistance. Waist circumference, blood pressure, a fasting lipid level, and a fasting glucose level or hemoglobin A1c should be obtained. A CBC with platelets and measurement of serum albumin, iron, total iron-binding capacity, and ferritin levels would also be indicated. Iron studies should be ordered to rule out hereditary hemochromatosis, which is an autosomal recessive disease that causes increased iron absorption in the intestines and release by tissue macrophages.
A 25-year-old gravida 1 para 0 at 24 weeks gestation comes to your office with right lower extremity swelling and pain. Her pregnancy has been uncomplicated so far and her only medication is a prenatal vitamin. She does not have chest pain, shortness of breath, or fever. She recently started feeling the baby move, and an anatomy scan at 20 weeks gestation was normal.
Lower extremity Doppler ultrasonography confirms a right lower extremity deep vein thrombosis (DVT). Laboratory studies including a CBC, coagulation studies, and renal function are normal.
Which one of the following would be the most appropriate initial treatment of her DVT?
A) Oral apixaban (Eliquis)
B) Oral aspirin
C) Oral warfarin (Coumadin)
D) Subcutaneous enoxaparin (Lovenox)
E) Subcutaneous heparin
ANSWER: D
Enoxaparin is the most appropriate pharmacologic therapy for anticoagulation in patients who are pregnant. Aspirin is not used as treatment for deep vein thrombosis. Apixaban, warfarin, and heparin either have not been studied for use in pregnancy or there is data indicating potential fetal harm.
A 75-year-old white male presents to your office following hospitalization for an episode of heart failure. His edema has resolved but he still becomes symptomatic with minor exertion such as walking less than a block. A recent chest radiograph shows cardiomegaly, and echocardiography reveals an ejection fraction of 25%. He is currently taking furosemide (Lasix), 20 mg daily; carvedilol (Coreg), 25 mg twice daily; and lisinopril (Prinivil, Zestril), 20 mg daily. His vital signs include a pulse rate of 60 beats/min, a blood pressure of 110/70 mm Hg, a respiratory rate of 18/min, and a temperature of 37.0°C (98.6°F). No crackles or hepatojugular reflux are noted on auscultation.
Which one of the following would improve this patient’s symptoms and decrease his mortality risk?
A) Digoxin
B) Hydralazine and isosorbide dinitrate (BiDil)
C) Hydrochlorothiazide
D) Spironolactone (Aldactone)
ANSWER: D
For patients with left ventricular systolic dysfunction, clinical trials have demonstrated that ACE inhibitors, -blockers, angiotensin receptor blockers, and aldosterone antagonists decrease hospitalizations and all-cause mortality. In African-American patients, all-cause mortality and hospitalizations have been
reduced by hydralazine and isosorbide dinitrate.
A 67-year-old female with hypertension and atrial fibrillation has been taking warfarin (Coumadin) for the past 10 years. She has been hemodynamically stable for many years with no complications from her atrial fibrillation. She is scheduled to undergo elective bladder sling surgery for urinary incontinence. She does not have any other significant past medical history.
Which one of the following would be the most appropriate perioperative management of her warfarin?
A) Continue warfarin without interruption
B) Discontinue warfarin the day prior to surgery and provide bridge therapy with low
molecular weight heparin
C) Discontinue warfarin 2 days prior to surgery and restart it 2 days postoperatively
unless there is a bleeding complication
D) Discontinue warfarin 2 days prior to surgery and restart it 5 days postoperatively
unless there is a bleeding complication
E) Discontinue warfarin 5 days prior to surgery and restart it 12–24 hours
postoperatively unless there is a bleeding complication
ANSWER: E
Perioperative management of chronic anticoagulation requires an assessment of the patient’s risk for thromboembolism and the risk of bleeding from the surgical procedure. High-risk patients include those with mechanical heart valves, a stroke or TIA within the past 3 months, venous thromboembolism within the past 3 months, or coronary stenting within the previous 12 months. High-risk patients require bridging therapy with low molecular weight heparin, while patients at low risk do not require bridging anticoagulation. For low-risk patients, it is recommended that warfarin be discontinued 5 days prior to surgery and restarted 12–24 hours postoperatively. This patient is at low risk for thromboembolism because her CHA2DS2-VASc score is 3. A patient with atrial fibrillation should receive bridging therapy with a CHA2DS2-VASc score 6. This patient’s surgery is associated with a high risk for bleeding, so it is preferable to stop her warfarin 5 days before the operation.
A 27-year-old female with a past medical history of polycystic ovary syndrome (PCOS) would like to become pregnant. Which one of the following treatments for PCOS is associated with greater live-birth and ovulation rates?
A) Finasteride (Proscar)
B) Letrozole (Femara)
C) Metformin (Glucophage)
D) Spironolactone (Aldactone)
ANSWER: B
In a double-blind randomized trial, letrozole was associated with greater live-birth and ovulation rates compared to clomiphene (SOR A). A Cochrane review indicated that metformin does not increase fertility in patients diagnosed with polycystic ovary syndrome (PCOS). Spironolactone and finasteride are both used to treat PCOS in women who do not desire pregnancy.
A 58-year-old male with a history of tobacco and alcohol abuse presents with the sudden onset of many well circumscribed brown, oval, rough papules with a “stuck-on” appearance on his trunk and proximal extremities. On examination you also note an unintentional 6-kg (13-lb) weight loss over the last 3 months and conjunctival pallor. A review of systems is positive for more frequent stomachaches, decreased appetite, and mild fatigue.
You order a laboratory workup. Which one of the following would be most appropriate at this point?
A) Reassurance that the skin lesions are benign
B) A skin biopsy
C) Referral to a dermatologist
D) CT of the abdomen and pelvis
E) Upper and lower endoscopy
ANSWER: E
This patient’s age, risk factors, red-flag symptoms, and other clinical findings indicate the need for endoscopy. The Leser-Trélat sign may be defined as the abrupt onset of multiple seborrheic keratoses, which is an unusual finding that often indicates an underlying malignancy, most commonly an adenocarcinoma of the stomach. CT is not an initial approach for diagnosing a suspected malignancy of the stomach or colon. Further skin evaluation and lifestyle changes, which are indicated, will not address the need for evaluation of weight loss and other abnormal symptoms and findings.
A 49-year-old male is concerned about lesions on his penis that he has noticed over the past 6 months. He was circumcised as a child and has had the same female sexual partner for 5 years. He does not have any pain, itching, or dysuria. On examination you note multiple reddish-blue papules on the scrotum and a few similar lesions on the shaft of the penis.
The most likely diagnosis is
A) pearly penile papules
B) lichen nitidus
C) lichen sclerosus
D) angiokeratomas
E) squamous cell carcinoma in situ (Bowen’s disease)
ANSWER: D
Penile lesions are usually easily diagnosed from clinical findings. Pearly penile papules are common and benign, and present as small, skin-colored, dome-shaped papules in a circular pattern around the coronal sulcus.
Lichen nitidus is benign but uncommon. It presents as discrete, pinhead-sized hypopigmented papules that are asymptomatic. Papules are often found scattered all over the penis, as well as on the abdomen and upper extremities.
Lichen sclerosus is more common and appears as hypopigmented lesions with the texture of cellophane. The lesions are usually located on the glans or prepuce. Atrophy, erosions, and bullae are common, and patients often present with itching, pain, bleeding, and possibly phimosis or obstructed voiding. Lichen sclerosus is associated with squamous cell cancer in a small percentage of cases.
Carcinoma in situ is a premalignant condition that is more common in uncircumcised males over age 60. Lesions are typically beefy red, raised, irregular plaques and can be found on the glans, meatus, frenulum, coronal sulcus, and prepuce. Lesions can be ulcerated or crusted. Pruritus and pain are common. A biopsy is important for making the diagnosis.
Angiokeratomas are lesions that are usually asymptomatic, circumscribed, red or bluish papules. They may appear solely on the glans of the penis, but are also found on the scrotum, abdomen, thighs, groin, and extremities. They may be misdiagnosed as pearly papules or carcinoma. Treatment is not necessary unless the lesions are bleeding or extensive. It is important to realize that angiokeratomas on the shaft of the penis, the suprapubic region, or the sacral region can be associated with Fabry disease. Patients with this finding should be promptly referred.
You are evaluating a 64-year-old female in the emergency department for pyelonephritis. Her past medical history is negative and she has previously been in good health. The patient appears acutely ill but is oriented. On examination her weight is 100 kg (220 lb), her temperature is 38.9°C (102.0°F), her pulse rate is 110 beats/min, her respiratory rate is 24/min, her blood pressure is 136/72 mm Hg, and her oxygen saturation is 94% on room air. Initial laboratory findings include a venous lactate level of 4.0 mmol/L (N 0.6–1.7).
You decide to start normal saline intravenously. Which one of the following would be the most appropriate initial rate?
A) 100 mL/hr
B) 150 mL/hr
C) 200 mL/hr
D) 3000 mL over 30 minutes
E) 3000 mL over 3 hours
ANSWER: E
The Surviving Sepsis Campaign recommends that patients with elevated serum lactate or hypotension receive isotonic intravenous fluids such as normal saline or lactated Ringer’s solution at an initial rate of 30 mL/kg in the first 3 hours using small boluses of approximately 500 mL. A serum lactate value >36 g/dL (4 mmol/L) is correlated with increased severity of illness and poorer outcomes even if hypotension is not yet present. Patients who are hypotensive or whose serum lactate level is >36 g/dL require intravenous fluids or colloid to expand their circulating volume and effectively restore perfusion pressure. The administration of 30 mL/kg of fluid is recommended as a fluid challenge, which should be started as early as possible in the course of septic shock.
You see a patient who is being treated for opioid use disorder with buprenorphine. Which one of the following can be used as adjuvant treatment to reduce stress-related opioid cravings and increase abstinence?
A) Clonidine (Catapres)
B) Methadone
C) Naloxone
D) Naltrexone (Vivitrol)
E) Nifedipine (Procardia)
ANSWER: A
Clonidine (0.1–0.3 mg every 6–8 hours) is a useful adjunct to buprenorphine in the treatment of opioid use disorder to help increase the rates of abstinence and decrease stress-related opioid cravings (SOR C). Naloxone is an opioid antagonist used to treat overdoses. Nifedipine is a common antihypertensive like clonidine but it has no role in the treatment of opioid use disorder. Methadone and naltrexone are used to treat opioid use disorder but neither of these agents would be used simultaneously with buprenorphine.
In a 60-year-old patient who has not previously received pneumococcal vaccine, which one of the following would be an indication for both 13-valent pneumococcal conjugate vaccine (PCV13, Prevnar 13) and 23-valent pneumococcal polysaccharide vaccine (PPSV23, Pneumovax 23)?
A) Alcoholism
B) Chronic renal failure
C) Cigarette smoking
D) COPD
E) Diabetes mellitus
ANSWER: B
Both 13-valent pneumococcal conjugate vaccine (PCV13) and 23-valent pneumococcal polysaccharide vaccine (PPSV23) are recommended for patients with chronic renal failure. Indications for PPSV23 alone in immunocompetent persons younger than 65 include chronic lung disease, diabetes mellitus, chronic heart disease, smoking, and alcoholism.
A 36-year-old male presents with a 2-day history of painless right-sided facial droop. There are no associated symptoms and his medical history is otherwise unremarkable. An examination is remarkable for an unfurrowed right brow, mouth droop, a sagging right lower eyelid, and a complete inability to move the muscles of the right face and forehead. No other weakness is elicited and no rash is seen.
Which one of the following would be the most appropriate management at this point?
A) Reassurance only
B) Valacyclovir (Valtrex) alone
C) A tapering dose of prednisone alone
D) Valacyclovir and a tapering dose of prednisone
E) Immediate transfer to the emergency department
ANSWER: D
Early recognition and effective treatment of acute Bell’s palsy (idiopathic facial paralysis) has been shown to decrease the risk of chronic partial paralysis and pain. Corticosteroids have been shown in a meta-analysis to decrease chronic symptoms, but a Cochrane meta-analysis of 10 studies concluded that antiviral medication along with corticosteroids is significantly more effective than corticosteroids alone. The medications are most effective if started within 72 hours of symptom onset. The same analysis showed that antiviral medications alone were less effective than corticosteroids alone. This patient’s presentation is not consistent with stroke or another emergency. Because supranuclear input to the facial nerves comes from both cerebral hemispheres, strokes and other central pathologies affecting the facial nerves typically spare the forehead, which is not the case in this patient.
In a patient presenting with truncal obesity, hypertension, type 2 diabetes mellitus, hirsutism, osteopenia, and skin fragility, which one of the following tests is needed to confirm the diagnosis of Cushing syndrome?
A) A dexamethasone suppression test
B) Inferior petrosal sinus sampling
C) Plasma corticotropin
D) Plasma free cortisol
E) Urinary free cortisol
ANSWER: E
In a patient presenting with obesity, hypertension, type 2 diabetes mellitus, and hirsutism, who also has thin skin and osteopenia, an elevated 24-hour collection showing high urinary free cortisol confirms the presence of Cushing syndrome. The dexamethasone suppression test, though still commonly used, no longer has a place in the diagnosis and treatment of patients with Cushing syndrome. Corticotropin-dependent and corticotropin-independent causes of Cushing syndrome can be separated by measuring plasma corticotropin. Plasma free cortisol measurements should be obtained only to determine the success or failure of transsphenoidal microadenomectomy or adrenalectomy. Inferior petrosal sinus sampling is used to confirm the source of corticotropin secretion before surgical intervention.
A 74-year-old female with a long-standing history of coronary artery disease is hospitalized for pneumonia. The patient improves with treatment and is hemodynamically stable. An EKG performed on the third day of hospitalization is shown below.
[Progressive PR prolongation w drop beat]
Which one of the following would be the most appropriate next step?
A) Cardiac rhythm monitoring with no additional treatment
B) Atropine
C) Transcutaneous pacing
D) Transvenous pacing
ANSWER: A
Second degree Mobitz type I (Wenckebach) heart block is characterized by an intermittent blockade of electrical impulses from the atria to the ventricles at the level of the atrioventricular node. This prevents generation of a QRS complex. It is characterized by progressive prolongation of the PR interval until a P wave is not followed by a QRS complex. P waves come at regular intervals so PP intervals are normal. Following the missed QRS complex, the PR interval returns to its baseline duration. A pacemaker is not recommended in patients with second degree Mobitz type I heart block who are asymptomatic. It is recommended in symptomatic patients, however, and is guided by electrophysiologic studies.
A U.S. hospital or birthing center seeking to be certified as “Baby-Friendly” by the Baby-Friendly Hospital Initiative must satisfy which one of the following criteria in addition to meeting other requirements?
A) Demonstrating proper use of an infant car seat to parents prior to discharge
B) Providing no other food or fluids to breastfeeding infants without a medical
indication
C) Providing a pacifier to each baby prior to discharge
D) Providing easy access to a variety of infant formulas
E) Providing on-site daycare facilities for staff
ANSWER: B
The Baby-Friendly Hospital Initiative is a global program established by UNICEF and WHO to promote healthy infant feeding and mother-baby bonding. The primary objective is to educate the public on the benefits of breastfeeding and encourage, promote, and facilitate breastfeeding as outlined in the UNICEF/WHO Ten Steps to Successful Breastfeeding chart. These steps promote breastfeeding to the public and provide guidelines for hospitals and birthing centers for the successful initiation and continuation of breastfeeding.
Baby-friendly facilities must have a written breastfeeding policy that is routinely communicated to all health care staff, and all health care staff must be trained in the skills necessary to implement this policy. All pregnant women should be informed about the benefits and management of breastfeeding. Mothers should be helped to initiate breastfeeding within an hour after birth and shown how to breastfeed and to maintain lactation, even if they are separated from their infants. Breastfeeding infants should not be given food other than breast milk, unless medically indicated. If mothers choose to give formula after appropriate education, they should be instructed in proper preparation and use.
Rooming in should be practiced, allowing mothers and infants to remain together 24 hours a day. Mothers should be encouraged to breastfeed on demand. Breastfeeding infants should not be given pacifiers or artificial nipples. Mothers should be referred to breastfeeding support groups on discharge from the hospital. In addition, the hospital must comply with the International Code of Marketing of Breast Milk Substitutes, which requires that formula companies cannot give free gifts to staff or mothers, that breast milk substitutes are not marketed in the maternity unit, and that breast milk supplements and infant feeding supplies are purchased at fair market price.
According to the recommendations of the American Heart Association, which one of the following patients requires endocarditis prophylaxis?
A) A 10-year-old female with a previous history of Kawasaki disease without valvular dysfunction
B) A 22-year-old female who underwent surgical repair of a ventricular septal defect 1 year ago
C) A 28-year-old female with mitral valve prolapse without regurgitation
D) A 35-year-old female with a history of infectious endocarditis in her 20s that was
related to intravenous drug use
E) A 42-year-old female with a history of rheumatic fever with chorea who has normal
cardiovascular findings
ANSWER: D
The American Heart Association and the American College of Cardiology have decreased the number of indications for antibiotic prophylaxis prior to dental procedures. Currently antibiotics are indicated for prosthetic cardiac valves, previous infective endocarditis, unrepaired cyanotic congenital heart disease or a repaired congenital defect with a residual shunt, and a cardiac transplant with valve regurgitation due to a structurally abnormal valve. Amoxicillin, 2 g, is the antibiotic prophylaxis of choice.
Which one of the following is most commonly associated with oligohydramnios?
A) Anencephaly
B) Esophageal atresia
C) Hydrops
D) Maternal -thalassemia
E) Posterior urethral valves
ANSWER: E
Amniotic fluid volume is regulated in part by fetal swallowing, inspiration, and urination. Some malformations of the urinary tract, including renal agenesis and persistent obstruction from posterior urethral valves, lead to oliguria or anuria, and are associated with marked oligohydramnios.
Anencephaly, esophageal atresia, heart failure, and maternal -thalassemia are associated with polyhydramnios. Anencephaly is probably the most common cause of polyhydramnios, via transudation from the exposed meninges; swallowing difficulties and excessive urination may also be contributing factors. Esophageal atresia is almost always associated with polyhydramnios due to an inability to swallow. Intrauterine heart failure, whether due to dysrhythmias, structural defects, or severe anemia, often leads to fetal hydrops, which is associated with polyhydramnios. -Thalassemia, relatively common in Asians, can also cause fetal hydrops and polyhydramnios.
You see a 47-year-old female for follow-up of a rash. She is a carpenter and was seen 4 days ago for increasing redness and tenderness of her anterior shin after hitting the area with a board 3 days earlier. She was afebrile during that visit and the area was red but not fluctuant. She chose observation rather than treatment at that time. The patient smokes 10 cigarettes daily. Past medical, surgical, and family histories are otherwise negative. Screening for diabetes mellitus was normal last year.
Today the patient’s anterior shin is still tender. She is afebrile and other vital signs are unremarkable. The extent of the infection was drawn 4 days ago with an indelible marker by your partner. Currently the area of redness extends beyond this border. There is no fluctuance or drainage of the wound. The skin appears mildly indurated.
Which one of the following would be best to provide coverage against Streptococcus pyogenes or methicillin-resistant Staphylococcus aureus (MRSA) in this patient?
A) Amoxicillin/clavulanate (Augmentin) and ciprofloxacin (Cipro)
B) Cephalexin and dicloxacillin
C) Dicloxacillin and fosfomycin (Monurol)
D) Doxycycline and trimethoprim/sulfamethoxazole (Bactrim)
E) Trimethoprim/sulfamethoxazole and cephalexin
ANSWER: E
Clindamycin or a combination of trimethoprim/sulfamethoxazole (or doxycycline or minocycline) plus cephalexin (or dicloxacillin or amoxicillin/clavulanate) should provide adequate coverage for Streptococcus and methicillin-resistant Staphylococcus aureus (MRSA) for mild to moderate cellulitis.
Doxycycline plus trimethoprim/sulfamethoxazole would provide inadequate coverage for streptococcal bacteria. Cephalexin plus dicloxacillin would provide inadequate coverage for MRSA. The primary indication for ciprofloxacin is treatment of infections with gram-negative rods. Fosfomycin is indicated only for urinary tract infections. Neither is typically used in the treatment of cellulitis
The mother of a newborn infant is concerned because her baby’s eyes are sometimes crossed. Assuming the intermittent eye crossing persists, which one of the following is the most appropriate age for ophthalmologic referral?
A) 10–14 days
B) 6 months
C) 12 months
D) 24 months
ANSWER: B
In many normally developing infants there may be imperfect coordination of eye movements and alignment during the early days and weeks of life, but proper coordination should be achieved by age 4–6 months. Persistent deviation of an eye in an infant requires evaluation.
A 47-year-old male presents with a 3-day history of fever, chills, low back pain, and urinary frequency. He does not have any nausea, vomiting, or abdominal pain. There is no significant past medical history.
The patient’s vital signs include a temperature of 38.1°C (100.6°F), a pulse rate of 88 beats/min, and a respiratory rate of 14/min. The examination reveals a mildly tender lower abdomen with no guarding or rebound tenderness; no costovertebral angle tenderness; and an enlarged, homogeneous, exquisitely tender prostate.
Which one of the following is indicated to help guide this patient’s treatment?
A) A serum prostate-specific antigen level
B) A culture of prostate secretions after massage of the prostate
C) A culture of midstream voided urine
D) CT of the abdomen and pelvis with intravenous and oral contrast
E) An ultrasound-guided prostate biopsy
ANSWER: C
This patient has clinically diagnosable acute bacterial prostatitis, and no further testing, including imaging, is required to establish the diagnosis. Culture of a midstream voided urine may aid in identifying the pathogen, but prostate massage should be avoided because it may increase the risk of bacteremia. A prostate biopsy is not indicated in the presence of acute infection, and a prostate-specific antigen level is not indicated because it is likely to be elevated in the presence of infection.
A 57-year-old female is admitted to the hospital with lower lobe pneumonia. She has no history of diabetes mellitus. She has not met sepsis criteria but had a blood glucose level of 172 mg/dL in the emergency department.
Insulin should be started if this patient has a persistent blood glucose level greater than or equal to
A) 120 mg/dL
B) 140 mg/dL
C) 160 mg/dL
D) 180 mg/dL
ANSWER: D
Insulin therapy should be initiated in hospitalized patients with persistent hyperglycemia, starting at a threshold of 180 mg/dL. Once insulin therapy is started, a target glucose range of 140–180 mg/dL is recommended for the majority of hospitalized patients, regardless of whether they have a critical illness.
A 69-year-old male presents for follow-up of hypertension treated with spironolactone (Aldactone) and amlodipine (Norvasc). His past medical history is remarkable only for a kidney stone several years ago. A physical examination is unremarkable. A comprehensive metabolic panel is unremarkable except for a calcium level of 12.0 mg/dL (N 8.0–10.0).
Which one of the following is the most likely cause of his elevated calcium level?
A) Excessive ingestion of calcium supplements
B) His current medication regimen
C) Occult malignancy
D) Primary hyperparathyroidism
E) Vitamin D deficiency
ANSWER: D
The most common cause of hypercalcemia is hyperparathyroidism. This is seldom symptomatic and is often discovered through routine blood testing. Hypercalcemia due to cancer can be caused by secretion of the parathyroid hormone–related protein and by osteoclastic bone resorption. Other causes of hypercalcemia include thiazide diuretics, lithium, vitamin D intoxication, hyperthyroidism, milk alkali syndrome from excessive calcium antacid ingestion, adrenal insufficiency, and lymphoma.
A 7-month-old male is admitted to the hospital for respiratory syncytial virus bronchiolitis. His temperature is 37.9°C (100.2°F), pulse rate 160 beats/min, respiratory rate 70/min, and oxygen saturation 92% on room air. Auscultation of the lungs reveals diffuse wheezing and crackles accompanied by nasal flaring and retractions.
Which one of the following interventions would most likely be beneficial?
A) Bronchodilators
B) Corticosteroids
C) Epinephrine
D) Nasogastric fluids
E) Oxygen supplementation to maintain O2 saturation above 95%
ANSWER: D
The mainstay of therapy for acute respiratory syncytial virus bronchiolitis is supportive care, and maintaining hydration is important. Infants with respiratory rates >60/min may have poor feeding secondary to difficulty breathing and oral rehydration may increase the risk of aspiration. In these cases, nasogastric or intravenous fluids should be administered. Oxygen saturation of 90% or more on room air is sufficient for infants with bronchiolitis, and using supplemental oxygen to maintain higher oxygen saturations only prolongs hospitalization because of an assumed need for oxygen. Bronchodilators should not be administered to infants with bronchiolitis, because they have not been shown to have any effect on the need for hospitalization, oxygen saturation, or disease resolution. In addition, there is no evidence to support the use of epinephrine or corticosteroids in the inpatient setting.
A healthy 43-year-old executive presents with problems falling asleep and staying asleep. Doxepin (Silenor) and extended-release melatonin have not helped.
In addition to behavioral interventions, which one of the following would be the most appropriate pharmacologic therapy for this patient’s insomnia at this time?
A) Diphenhydramine (Benadryl)
B) Doxylamine (Unisom)
C) Eszopiclone (Lunesta)
D) Olanzapine (Zyprexa)
E) Zaleplon (Sonata)
ANSWER: C
Although behavioral interventions are the mainstay of treatment for insomnia, they often need to be supplemented by pharmacologic therapy. When both doxepin and extended-release melatonin fail to provide benefit, a member of the Z-drug class should be tried next. Among the Z-drugs only eszopiclone provides an early peak onset and a long half-life, with a 1-hour approximate time to peak and a 6-hour half-life. While zaleplon has an equally short time to peak of 1 hour, it also has a 1 hour half-life. Antihistamines, including diphenhydramine and doxylamine, as well as atypical antipsychotics such as olanzapine, are not indicated unless used primarily to treat another condition.
A 21-year-old gravida 1 para 0 is diagnosed with overt hyperthyroidism early in the first trimester. The most appropriate management at this time is
A) observation only
B) methimazole (Tapazole)
C) propylthiouracil
D) radioactive iodine
E) thyroidectomy
ANSWER: C
Overt hyperthyroidism during pregnancy is associated with adverse effects to the mother and fetus, so treatment is required. Since methimazole is associated with birth defects when used in the first trimester, propylthiouracil is preferred. Methimazole should be considered after the first trimester because the risk of congenital anomalies is less than the risk of liver failure associated with propylthiouracil. Surgery and radioactive iodine should only be used if there is a clear indication, and radioactive iodine would not be appropriate during pregnancy.
A 45-year-old male with a 30-pack-year smoking history reports a chronic cough with a small amount of phlegm production and dyspnea with strenuous exercise. You order spirometry, which shows a pre- and postbronchodilator FEV1/FVC ratio of 0.6 and an FEV1 of 85% of predicted.
Which one of the following agents would be the best initial pharmacologic management?
A) An inhaled corticosteroid
B) A short-acting anticholinergic
C) A long-acting anticholinergic
D) A long-acting 2-agonist
E) Theophylline
ANSWER: B
This patient has COPD and is in a risk category of A (low risk, fewer symptoms) based on the Global Initiative for Chronic Obstructive Lung Disease (GOLD) combined assessment of COPD. As a result, either a short-acting anticholinergic or a short-acting 2-agonist should be selected as the initial pharmacologic management. Long-acting 2-agonists or long-acting anticholinergics are indicated for patients with a GOLD combined assessment category of B or worse. Long-acting inhaled corticosteroids are indicated for patients with a GOLD combined assessment category of C or worse. Due to its narrow therapeutic window, modest benefit, and need for monitoring, theophylline is not recommended as an initial agent and should be considered as an alternative only for patients with severe refractory symptoms.
A 30-year-old white male presents to the emergency department with a 4-day history of fever to 101°F, a sore throat, rhinorrhea, and cough. An examination reveals rhinorrhea and a boggy nasal mucosa, but is otherwise unremarkable. A chest radiograph shows a questionable infiltrate.
Which one of the following would help determine if antibiotic treatment would be appropriate?
A) A C-reactive protein level
B) A procalcitonin level
C) A WBC count with differential
D) An erythrocyte sedimentation rate
E) CT of the chest
ANSWER: B
Using a procalcitonin-guided therapy algorithm reduces antibiotic use by 3.47 days without increasing either morbidity or mortality in adults with acute respiratory infections. If the procalcitonin level is <0.10 mg/dL, a bacterial infection is highly unlikely and it is strongly recommended that antibiotics not be prescribed. If the procalcitonin level is 0.10–0.24 mg/dL a bacterial infection is still unlikely and it is recommended that antibiotics not be used. If the level is 0.25–0.50 mg/dL a bacterial infection is likely and antibiotics are recommended. It is strongly recommended that antibiotics be given if the level is >0.50 mg/dL, because a bacterial infection is very likely.
Additional workup or referral to an endocrinologist for evaluation of precocious puberty would be indicated in which one of the following patients?
A) A 7-year-old female with some pubic hair
B) An 8-year-old female with breast buds
C) An 8-year-old male with some pubic hair and axillary odor
D) An 8-year-old male with penile enlargement
E) A 10-year-old female who has recently begun having menses
ANSWER: D
Penile enlargement in an 8-year-old male is a sign of precocious puberty. Isolated sparse pubic and axillary hair growth and axillary odor is referred to as premature adrenarche, and represents high levels of dehydroepiandrosterone rather than activation of the hypothalamic-pituitary-gonadal axis that leads to puberty. The isolated findings of premature adrenarche are generally considered benign. An 8-year-old with breast buds and a 10-year-old with menarche are within the normal range of expected pubertal development. Penile enlargement typically represents full activation of the hypothalamic-pituitary-gonadal axis and warrants endocrinologic evaluation in boys younger than 9 years of age.
An 80-year-old male sees you for the first time. He is asymptomatic except for some fatigue. His pulse rate is 50 beats/min. An EKG shows a prolonged PR interval.
Which one of the following medications in his current regimen is the most likely explanation for these findings?
A) Donepezil (Aricept)
B) Escitalopram (Lexapro)
C) Lisinopril (Prinivil, Zestril)
D) Memantine (Namenda)
E) Zolpidem (Ambien)
ANSWER: A
The 2015 American Geriatrics Society Beers Criteria for potentially inappropriate medication use in older adults 65 years of age states that donepezil use should be avoided in patients with syncope, due to an increased risk of bradycardia (Moderate Evidence Level; Strong Strength of Recommendation). Donepezil is a cholinesterase inhibitor. Due to their cholinergic effect, these medications have a vagotonic effect on the sinoatrial and atrioventricular nodes. This can cause bradycardia or heart block in patients with or without underlying cardiac conduction abnormalities. Syncope has been reported with these medications.
83
Memantine is an N-methyl-D-aspartate receptor antagonist and is not associated with bradycardia. Escitalopram, lisinopril, and zolpidem are also not associated with bradycardia.
A 14-year-old female is brought to your office for an annual well child check and sports preparticipation physical examination. She says she does a lot of running during basketball practices and games but has trouble controlling her weight. Most of her family is overweight. She does not have any difficulty participating in sports, and has no symptoms such as chest pain, shortness of breath, or headaches. She has no significant past medical history.
On examination the patient’s height is 154 cm (61 in) and she weighs 63 kg (139 lb). Her BMI is 26.4 kg/m2, which places her in the 90th percentile for her age. Her blood pressure is 130/85 mm Hg, which places her between the 95th and 99th percentile for her age, height, and sex. Her chart reveals that her blood pressure was at this level at the last two visits. The physical examination is otherwise normal.
In addition to counseling and support for weight loss, which one of the following would be most appropriate at this point?
A) Informing the patient and her parents that she is prehypertensive and having her return for a blood pressure check in 3 months
B) Plasma renin and catecholamine levels
C) An imaging study of the renal arteries
D) A fasting basic metabolic panel, a lipid profile, and a urinalysis
E) Antihypertensive drug therapy
ANSWER: D
In a pediatric patient, blood pressure should be evaluated using comparisons based on age, sex, and height. Although this adolescent’s blood pressure is prehypertensive for an adult according to JNC 8 guidelines, it is stage 1 hypertension (between 95% and 99%) for her age, sex, and height. All pediatric patients with confirmed hypertension should have further evaluation to check for renal dysfunction as well as other cardiac risk factors. Additionally, renal ultrasonography is recommended to evaluate for renal disease and echocardiography to evaluate for end-organ damage that would affect treatment goals. Additional studies, such as plasma renin and catecholamine levels or renovascular imaging, may be indicated in children with abnormalities on initial evaluation that suggest secondary causes of hypertension.
Pharmacologic therapy is usually recommended for pediatric patients with symptomatic hypertension, secondary hypertension, target organ damage, diabetes mellitus, or persistent hypertension in spite of nonpharmacologic treatment. A low-sodium diet may be helpful for decreasing blood pressure, and given this patient’s obesity, intensive counseling about lifestyle changes is appropriate.
A 20-year-old college student comes to the urgent care clinic with right knee pain and swelling after injuring her knee in a recreational basketball game. Her feet were planted when another player collided with her, causing her upper torso to rotate. She felt immediate pain in the knee and was unable to complete the game.
Which one of the following is the most accurate and appropriate maneuver to detect an anterior cruciate ligament tear?
A) The anterior drawer test
B) The lever sign test
C) The Lachman test
D) The McMurray test
E) The pivot shift test
ANSWER: C
The Lachman test is the most accurate test for an anterior cruciate ligament (ACL) tear (SOR A). Accurate testing can lead to appropriate referral and treatment for ACL tears, and early detection can lead to better outcomes. The Lachman test has higher validity based on a sensitivity of 68% for partial ruptures and 96% for complete ruptures. The other two commonly used tests are the anterior drawer test, which has a sensitivity of 38% and a specificity of 81%, and the pivot shift test, which is more technically difficult than the other two tests and has a sensitivity ranging from 24% to 85%. The pivot shift test is effective if done correctly but should not be used alone to diagnose an ACL tear (SOR A).
85
The lever sign test is a newer test that holds promise for detecting ACL tears and is easily performed in the office. However, sensitivity and specificity reports vary (SOR B). The McMurray test is used to detect meniscal tears.
A 34-year-old female presents with a 1-month history of increasing foot pain. She does not have any significant past musculoskeletal history, and she started a new exercise program 6 weeks ago. She has pain in the lateral side of her heel that is present both with activity and at rest. On examination you note tenderness below the lateral malleolus extending to the midfoot.
Which one of the following is the most likely diagnosis?
A) Calcaneal apophysitis (Sever’s disease)
B) Calcaneal stress fracture
C) Peroneal tendinopathy
D) Plantar fasciitis
E) Tarsal tunnel syndrome
ANSWER: C
This patient most likely has peroneal tendinopathy, which is a degeneration of the peroneal tendon that involves pain or tenderness in the lateral calcaneus below the ankle along the path to the base of the fifth metatarsal. Initial treatment options include activity modification, decreasing pressure to the affected area, anti-inflammatory or analgesic medications, and eccentric exercises. Calcaneal apophysitis, or Sever’s disease, is a common growth-related injury that typically affects adolescents between 8 and 12 years of age. Symptoms often present after a growth spurt or starting a new high-impact sport or activity, and common examination findings include tight heel cords and a positive calcaneal squeeze test. A calcaneal stress fracture, which most commonly occurs immediately inferior and posterior to the posterior facet of the subtalar joint, involves pain that intensifies with activity and often worsens to include pain at rest. It typically follows an increase in weight-bearing activity or a switch to running or walking on a hard surface. Plantar fasciitis is characterized by sharp, shooting pain in the arch and medial aspect of the foot that often is worse upon arising and taking the first few steps of the morning. Examination of the foot reveals tenderness at the site and pain with dorsiflexion of the toes. Tarsal tunnel syndrome involves entrapment of the posterior tibial nerve and causes a burning, tingling, or shooting pain and numbness that radiates into the plantar aspect of the foot, often into the toes. The pain associated with tarsal tunnel syndrome typically worsens with activity and is relieved with rest.
A 30-year-old female with type 2 diabetes and obesity sees you for follow-up. She has experienced several episodes of symptomatic hypoglycemia, and because of this she stopped all of her medications except metformin (Glucophage). Her hemoglobin A1c has increased to 8.4%.
Which one of the following would be the best additional treatment for this patient?
A) Basal insulin (Lantus)
B) Rapid-acting insulin (Humalog)
C) Exenatide (Byetta)
D) Glipizide (Glucotrol)
E) Repaglinide
ANSWER: C
Exenatide is a GLP-1 receptor agonist that is not associated with hypoglycemia and can also assist with weight loss, which would be helpful in this patient with obesity. All of the other listed medications, including both types of insulin, sulfonylureas, and meglitinides, can be associated with hypoglycemia. Since this patient’s hemoglobin A1c is only moderately elevated at 8.4%, exenatide is reasonable, although it can be an expensive option. If her hemoglobin A1c was severely elevated, insulin would be indicated, with close monitoring for hypoglycemia.
Polypharmacy increases the risk of adverse health outcomes. According to the Choosing Wisely campaign, adding to a threshold of how many medications in a patient’s regimen should prompt a thorough review to determine if any of the medications can be discontinued?
A) 3 B) 5 C) 7
D) 10
ANSWER: B
Polypharmacy, which is defined as regular use of five or more medications, increases the risk of adverse medical outcomes. Patients who take five or more medications can find it difficult to understand and adhere to the complicated regimens. Risk factors for polypharmacy include having multiple medical conditions that are managed by multiple specialist or subspecialist physicians, residing in a long-term care facility, having poorly updated medical records, and using automated refill services. Inappropriate prescribing of drugs that are not discontinued after their usual effective or recommended period is known as legacy prescribing and can contribute to inappropriate polypharmacy.
According to the Choosing Wisely campaign, any prescriptions beyond a threshold of five medications should trigger a thorough review of the patient’s complete regimen, including over-the-counter medications and dietary supplements, to determine if any of the medications can be discontinued. Tools such as the Beers criteria list and the Medication Appropriateness Index can be used to identify potentially inappropriate medication use, but no single tool or strategy has been determined to be superior. If discontinuation of particular high-risk medications is not possible because of medical conditions, then dose reductions should be considered.
A 70-year-old female with a history of coronary artery disease, a femorofemoral bypass 3 years ago, and hypertension sees you for a follow-up visit. She has intermittent right arm pain that is worse with exercise. The pain increases with all arm exercises and improves with rest. The patient’s blood pressure is 140/70 mm Hg in the left arm and 120/64 mm Hg in the right arm.
Which one of the following would be the most appropriate next step?
A) Radiographs of the right shoulder
B) Arterial duplex ultrasonography of the upper extremities
C) MR angiography
D) No imaging, and referral to physical therapy
ANSWER: B
This patient has peripheral artery disease (PAD) of the right arm. PAD of the upper extremities is characterized by pain with exertion and can cause gangrene and ulceration. It is more common in patients who have had lower extremity occlusive disease. A blood pressure differential of 15 mm Hg between arms suggests stenosis and warrants further testing. Initial testing in symptomatic patients includes arterial duplex ultrasonography of the upper extremities. CT angiography and MR angiography may be appropriate to clarify the diagnosis or plan intervention. Neither radiography nor physical therapy would be appropriate.
A 19-year-old female presents with a 4-year history of intermittent facial acne. She tells you that her acne has never completely resolved, and it worsens during her menstrual period. She has tried various over-the-counter facial cleansers although she does not recall what they contained. On examination she has scattered open and closed comedones, and pustules on her forehead and around her mouth.
Which one of the following treatments would you recommend?
A) Topical adapalene (Differin)
B) Topical clindamycin (Cleocin T)
C) Oral doxycycline
D) Oral isotretinoin (Absorica)
ANSWER: A
This patient has mild inflammatory acne as indicated by her combination of comedones and pustules. She does not have extensive skin involvement and should benefit from the use of a topical agent. Topical retinoids, including adapalene, tretinoin, tazarotene, and trifarotene, are appropriate for the treatment of mild to moderate acne as single agents, although they may be more effective when combined with a topical antibiotic or benzoyl peroxide. Topical antibiotics can lead to bacterial resistance and should not be used as monotherapy. Oral antibiotics are appropriate for the treatment of moderate to severe acne that has failed to respond to topical treatment. Oral isotretinoin is reserved for the treatment of severe nodular acne.
An otherwise healthy 21-year-old male sees you for follow-up after a hospitalization for pneumonia. This was his second pneumonia infection of the year. He reports a history of multiple sinus infections and upper respiratory infections over the years that were treated with antibiotics on an outpatient basis. Laboratory studies reveal a normal CBC and a decreased IgA level. A trial of pneumococcal polysaccharide vaccine (PPSV23, Pneumovax 23) reveals no measurable response.
This presentation is most consistent with
A) selective IgA deficiency
B) common variable immunodeficiency
C) severe combined immunodeficiency
D) DiGeorge syndrome
E) Wiskott-Aldrich syndrome
ANSWER: B
Common variable immunodeficiency (CVID) is the only immunodeficiency condition listed that can present later in life, while severe combined immunodeficiency, DiGeorge syndrome, and Wiskott-Aldrich syndrome typically present prior to 6 months of age. CVID is a condition of impaired humoral immunity and thus should be considered in a patient this age in the setting of recurrent bacterial infections such as sinusitis or pneumonia. The blunted response to a vaccination challenge implies impaired IgG antibody response, which differentiates CVID from a selective IgA deficiency. Because severe combined immunodeficiency is associated with significant abnormalities of both T-cell and B-cell function, it presents very early in life with multiple severe, opportunistic infections, and failure to thrive. DiGeorge syndrome is associated with multiple other physical abnormalities such as cardiac malformations and dysmorphic facial features. Wiskott-Aldrich syndrome is linked to the X chromosome (primarily affecting males) and associated with eczema and thrombocytopenia.
Which one of the following vitamins is a well established therapy for the treatment of nausea and vomiting in pregnancy?
A) Vitamin A
B) Vitamin B6
C) Vitamin B12
D) Vitamin C
E) Vitamin E
ANSWER: B
About half of pregnant women experience nausea and vomiting during pregnancy, which increases the risk of dehydration, poor weight gain, and impaired fetal growth. Pregnancy-related nausea and vomiting often begins by 4 weeks estimated gestation and typically resolves by the end of 12 weeks estimated gestation. When treating common causes of nausea and vomiting during pregnancy, lifestyle modifications such as frequent small meals and avoidance of high-protein or fatty foods are considered the safest intervention and first-line therapy. If these conservative measures are not effective, other well established low-risk therapies can be added in a stepwise fashion. These options include vitamin B6 (pyridoxine), over-the-counter antihistamines such as doxylamine, and natural ginger (<1500 mg daily). In addition, combination doxylamine/pyridoxine is approved by the FDA for the prevention of nausea and vomiting in pregnancy. Prescription antiemetics such as metoclopramide or trimethobenzamide are reserved for severe or refractory cases. Vitamin A, vitamin B12, vitamin C, and vitamin E are not appropriate for the treatment of pregnancy-related nausea and vomiting.
A 62-year-old female sees you for a routine health maintenance visit and asks for your advice regarding vitamin D supplementation. She is healthy and active, and jogs 1–2 hours three times weekly.
Which one of the following would be the most appropriate advice regarding vitamin D supplementation in this patient?
A) It is not recommended because she is asymptomatic
B) It will reduce the risk of certain cancers
C) It will reduce the risk of depression
D) It will reduce the risk of diabetes mellitus
E) It will reduce the risk of fractures
ANSWER: A
The U.S. Preventive Services Task Force and the American Academy of Family Physicians concluded that evidence is insufficient for vitamin D testing and for vitamin D supplementation in asymptomatic adults. Vitamin D supplementation does not reduce the risk of cancer, depression, diabetes mellitus, or fractures.
A 56-year-old male who has heart failure with reduced ejection fraction sees you for follow-up. He is stable but over the past year has noted an increase in dyspnea with moderate activity. His blood pressure is well controlled today. His current medications include carvedilol (Coreg), losartan (Cozaar), and escitalopram (Lexapro).
Which one of the following additions to his current medication regimen has the best evidence for reducing his risk of mortality from heart failure?
A) Aspirin
B) Atorvastatin (Lipitor)
C) Furosemide (Lasix)
D) Hydrochlorothiazide
E) Spironolactone (Aldactone)
ANSWER: E
This patient has symptomatic New York Heart Association class II heart failure, and an escalation in therapy is warranted. Both -blockers and aldosterone antagonists have been shown to reduce mortality in patients with symptomatic heart failure (SOR A). Management of associated cardiovascular disease such as hyperlipidemia and hypertension is important to prevent disease progression, but of the medications listed (aspirin, atorvastatin, furosemide, hydrochlorothiazide, and spironolactone) spironolactone is the best choice to reduce heart failure–related mortality.
Chronic kidney disease is defined by abnormal kidney structure or function lasting a minimum of
A) 2 months
B) 3 months
C) 6 months
D) 12 months
E) 24 months
ANSWER: B
Chronic kidney disease (CKD) is one of the most common chronic disease states encountered by family physicians, affecting 15% of the total U.S. adult population, and substantially impacting health care costs as well as morbidity and mortality. In the United States, diabetes mellitus and hypertension are the most common causes. CKD is defined by abnormal kidney structure or function lasting greater than 3 months, with associated implications for health. Diagnostic criteria include a persistent glomerular filtration rate <60 mL/min/1.73 m2, albuminuria, urine sediment abnormalities, renal imaging abnormalities, and serum acid-base or electrolyte abnormalities.
A 55-year-old female with type 2 diabetes sees you because of early satiety, nausea, vomiting, bloating, and postprandial fullness that is sometimes accompanied by upper abdominal pain. Since these symptoms have developed she has also noted increasing difficulty with blood glucose control.
Which one of the following would be the best study for confirming the most likely diagnosis?
A) Gastric emptying scintigraphy with a solid meal
B) Hepatobiliary scintigraphy (HIDA)
C) An upper gastrointestinal series with small-bowel follow-through
D) Abdominal ultrasonography
E) Abdominal CT
ANSWER: A
Gastroparesis is a complication of diabetes mellitus, and presents with nausea, vomiting, early satiety, bloating, postprandial fullness, and/or upper abdominal pain. Gastric emptying scintigraphy with a solid meal is the first-line study for confirming the diagnosis. Hepatobiliary scintigraphy (HIDA) is used to evaluate biliary dyskinesia and is not indicated in this patient. An upper gastrointestinal radiographic series, abdominal ultrasonography, and CT of the abdomen can help to rule out obstructive pathology, biliary tract disease, and other gastrointestinal conditions but would not confirm the diagnosis. The patient should also undergo esophagogastroduodenoscopy to exclude obstruction.
The administrator in your practice recently attended a quality improvement conference. He would like to start a clinic-wide program to focus on lead screening in asymptomatic children 5 years of age and younger.
After reviewing U.S. Preventive Services Task Force guidelines, which one of the following should you tell the practice administrator?
A) There is insufficient evidence to recommend for or against lead screening in children 5 years of age and younger
B) All children 5 years of age and younger should undergo lead screening
C) All children who live in housing built before 1978 should undergo lead screening
D) There are accurate and reliable screening questionnaires to guide lead screening
ANSWER: A
The U.S. Preventive Services Task Force (USPSTF) has found adequate evidence that questionnaires and other clinical prediction tools to identify asymptomatic children with elevated blood lead levels are inaccurate. The USPSTF went on to conclude that the current evidence is insufficient to assess the balance of benefits and harms of screening for elevated blood lead levels in asymptomatic children 5 years of age and younger. Although children living in older housing with lead-based paint are at higher risk of elevated blood lead levels than those living in housing built after 1978, the USPSTF does not recommend routine screening in asymptomatic children based on this risk factor.
A 60-year-old male who has type 2 diabetes comes to your office with an acute onset of fever, chills, and malaise. He says that he is feeling progressively worse. His temperature is 40°C (104°F). An examination reveals redness, tenderness, and swelling of the penis, scrotum, and perineal area.
Which one of the following medications is most likely to cause this condition?
A) Dapagliflozin (Farxiga)
B) Exenatide (Byetta)
C) Insulin glargine (Lantus)
D) Pioglitazone (Actos)
E) Sitagliptin (Januvia)
ANSWER: A
Nectoritzing fascitis of the peineum !!!
SGLT2 inhibitors (canagliflozin, dapagliflozin, empagliflozin, and ertugliflozin) are associated with a higher rate of genitourinary infections, including necrotizing fasciitis of the perineum (Fournier’s gangrene). While rare, this is a life-threatening infection associated with this class of medications that is being used more frequently to treat diabetes mellitus. Because of this risk, the FDA issued a Drug Safety Warning in 2018 due to case reports. The drug classes that include exenatide, insulin glargine, pioglitazone, and sitagliptin are not associated with this condition.
Conditions associated with adhesive shoulder
Diabetes adn hypothyroidism
A 55-year-old male sees you because of a second flare of gout. He has also had an elevated blood pressure at the last few visits to your clinic and is hypertensive again today.
In addition to treating his gout flare, which one of the following would be the most appropriate agent to treat his hypertension in light of his presenting problem?
A) Atenolol (Tenormin)
B) Hydralazine
C) Hydrochlorothiazide
D) Lisinopril (Prinivil, Zestril)
E) Losartan (Cozaar)
ANSWER: E
The 2020 American College of Rheumatology guideline for the management of gout generated numerous recommendations, including the management of concurrent medications in patients with gout. In such patients, losartan is the preferred antihypertensive agent when possible (SOR C). Hydrochlorothiazide should typically be changed to another agent, such as losartan, when feasible in patients with gout (SOR C). Both hydrochlorothiazide and losartan are known to have effects on the serum urate concentrations, with hydrochlorothiazide causing an increase and losartan causing a decrease. The American College of Rheumatology guideline does not recommend for or against the use of atenolol, hydralazine, and lisinopril as antihypertensive treatment in patients with gout.
A 60-year-old male comes to your office with a 1-year history of the gradual onset of mild fatigue and dyspnea. There are no symptom triggers. He has a 20-pack-year history of cigarette smoking but stopped at age 35. An examination is significant only for a BMI of 30 kg/m2. Office spirometry reveals a decreased FVC and a normal FEV1/FVC ratio, and there are no changes after bronchodilator administration.
Which one of the following would you recommend at this point?
A) The 6-minute walk test
B) Bronchoprovocation testing such as a methacholine challenge test
C) Full pulmonary function testing
D) Bronchoscopy
E) A ventilation-perfusion scan
ANSWER: C
Family physicians are often required to manage dyspnea and evaluate common office spirometry results. The American Thoracic Society recommends full pulmonary function testing when office spirometry suggests a restrictive pattern, which is the case with this patient’s normal FEV1/FVC ratio and decreased FVC. Full laboratory pulmonary function testing gives further information about gas exchange and lung volumes, which allows a more definitive diagnosis.
To determine compliance with prescribed medications and detect use of illicit substances, your clinic uses urine drug screening with an immunoassay qualitative point-of-care test to monitor patients who are on long-term opioid therapy. Which one of the following is most likely to result in a false-negative result and require confirmatory testing for detection?
A) Cannabis
B) Cocaine
C) Codeine
D) Morphine
E) Oxycodone (OxyContin)
ANSWER: E
Immunoassay drug screenings can be performed at the point of care and are relatively inexpensive. Typical immunoassays can detect nonsynthetic opioids such as morphine and codeine, as well as illicit substances such as amphetamines, cannabinoids, cocaine, and phencyclidine. However, these immunoassays do not reliably detect synthetic or semisynthetic opioids such as oxycodone, oxymorphone, methadone, buprenorphine, and fentanyl, as well as many benzodiazepines. Confirmatory testing is needed in situations with an unexpected negative result in order to distinguish a false negative from a true negative.
A 30-year-old female who is an established patient calls your office to request a test for COVID-19. The patient spent several hours inside the home of another individual who just received a positive COVID-19 test result. She states that her sense of taste seems diminished, but she has no respiratory symptoms and otherwise feels well.
Which one of the following is the typical incubation period for COVID-19?
A) 1day
B) 5 days
C) 14 days
D) 30 days
ANSWER: B
SARS-CoV-2 is a respiratory coronavirus that is responsible for COVID-19. Knowledge of the natural history of the viral infection will inform testing strategies and many other aspects of counseling of patients. The incubation period measures the time from exposure to symptom onset. The typical incubation period for COVID-19 is approximately 4–5 days, though it can range from 1–14 days.
A 45-year-old female with a 4-year history of type 2 diabetes is taking only metformin (Glucophage) and maintaining a hemoglobin A1c of 6.6%. Her LDL-cholesterol level is 94 mg/dL. She has no complications related to diabetes and her medical history is otherwise unremarkable.
Which one of the following should be added to her current medication regimen?
A) A DPP-4 inhibitor
B) An SGLT2 inhibitor
C) A low-intensity statin
D) A moderate-intensity statin
E) A high-intensity statin
ANSWER: D
All patients between 40 and 75 years of age with diabetes mellitus and an LDL-cholesterol level 70 mg/dL should begin taking a moderate-intensity statin. It is not necessary to calculate a 10-year risk for atherosclerotic cardiovascular disease because the results do not alter the recommendation. This patient’s hemoglobin A1c is <7%, which is acceptable, and she does not need additional hypoglycemic medications. She has no diabetes-specific risk-enhancing conditions such as a long duration of illness, chronic kidney disease, retinopathy, neuropathy, or an ankle-brachial index <0.9. Older age and risk-enhancing conditions may require increasing the statin to high-intensity dosages. A DPP-4 inhibitor, an SGLT2 inhibitor, and a low-intensity statin would not be appropriate for this patient at this time.
A 23-year-old primigravida comes to your office for her initial obstetric visit. She is at 13 weeks gestation based on the dates of her last menstrual period. She is a nonsmoker and does not drink alcohol or use illicit substances. Her vital signs are remarkable for a blood pressure of 142/92 mm Hg and a BMI of 32 kg/m2. She says that she has been diagnosed with hypertension in the past but has not taken any medications for it.
In addition to a prenatal vitamin, which one of the following would you recommend for her?
A) No additional medications
B) Aspirin
C) Ferrous sulfate
D) Folic acid
E) Labetalol (Trandate)
ANSWER: B
The U.S. Preventive Services Task Force (USPSTF) recommends prescribing low-dose aspirin after 12 weeks gestation for asymptomatic women at high risk for preeclampsia. Women at high risk include those with a history of preeclampsia, chronic hypertension, multiple pregnancy, type 1 or 2 diabetes, renal disease, autoimmune disease, or any combination of these. Many women become iron deficient in pregnancy but not all will require additional iron supplementation beyond what is available in the prenatal vitamin. The USPSTF found insufficient evidence to recommend for or against routine iron supplementation for pregnant women. Additional folic acid is recommended for women with increased risk for neural tube defects (NTDs), and while obesity increases the risk for NTD it is not an indication alone for a higher dosage of folic acid than the levels found in prenatal vitamins. In pregnant patients with chronic hypertension, treatment with antihypertensive medications is recommended only when the blood pressure is >150/100 mm Hg, because aggressive blood pressure lowering may result in placental hypoperfusion.
34-year-old female with asthma sees you for routine follow-up. She tells you that she uses her short-acting -agonist (SABA) approximately twice a week.
Which one of the following management strategies would you recommend for prevention of exacerbations?
A) Continued use of a SABA as needed
B) An inhaled corticosteroid (ICS)/long-acting -agonist (LABA) as needed
C) A daily maintenance ICS/LABA
D) A daily maintenance ICS plus a SABA as needed
E) A daily maintenance ICS plus a daily leukotriene receptor antagonist
ANSWER: B
For patients with mild asthma, recent evidence has shown that an inhaled corticosteroid (ICS)/long-acting -agonist (LABA), such as budesonide/formoterol, as needed was as effective at preventing exacerbations as a daily maintenance ICS plus a short-acting -agonist (SABA) at one-fifth of the total corticosteroid dose. In addition, it was more effective at preventing exacerbations than continued use of a SABA alone as needed. A daily maintenance ICS inhaler plus either a LABA or a leukotriene receptor antagonist are
management strategies for persistent asthma.
A 42-year-old female sees you because of intermittent right upper abdominal pain that occurs after eating. The episodes have been gradually worsening and now last up to an hour. She has tried over-the-counter antacids, ibuprofen, and acetaminophen, which have not helped. She tells you that the last episode occurred earlier this week and the pain was so severe that it woke her up and she went to the emergency department (ED). A comprehensive metabolic panel, CBC, and lipase level performed in the ED were all normal. Right upper quadrant abdominal ultrasonography today is negative for gallstones but notable for increased echogenicity of the liver.
Which one of the following would be the most appropriate next step in the evaluation?
A) Plain radiography of the abdomen
B) CT of the abdomen
C) Hepatobiliary scintigraphy (HIDA)
D) Magnetic resonance cholangiopancreatography (MRCP)
E) Endoscopic retrograde cholangiopancreatography (ERCP)
ANSWER: C
This patient presents with classic biliary symptoms and normal right upper quadrant ultrasonography, liver enzymes, and pancreatic enzymes. Abdominal ultrasonography was negative for gallstones. The next most appropriate test is hepatobiliary scintigraphy, also known as a hepatobiliary iminodiacetic acid (HIDA) scan. While a normal HIDA scan does not exclude a diagnosis of functional gallbladder disease (also referred to as acalculous cholecystitis, biliary dyskinesia, and biliary dysmotility), an abnormal study identifies patients for whom cholecystectomy is strongly recommended. Plain radiography of the abdomen and CT of the abdomen are helpful to evaluate for other etiologies of abdominal pain but are not the most appropriate next step for a patient with classic biliary symptoms and a normal laboratory workup. Magnetic resonance cholangiopancreatography (MRCP) is reserved for suspected choledocholithiasis. Endoscopic retrograde cholangiopancreatography (ERCP) is an invasive test also used for choledocholithiasis and in conjunction with sphincterotomy and stone extraction.
A 6-month-old male is brought to your office by his mother for a well child examination. The mother does not have any concerns. Interactions between the mother and child are appropriate and the child appears well.
Which one of the following screenings is recommended at this visit?
A) Autism
B) Iron deficiency
C) Maternal depression
D) Otoacoustic emissions (OAE) testing
NSWER: C
The American Academy of Pediatrics (AAP) recommends formal screening for maternal depression with the Edinburgh Postnatal Depression Scale or the Patient Health Questionnaire–2 (PHQ-2) at the 1-, 2-, 4-, and 6-month well child visits. The AAP recommends screening for autism at 18 months, but the U.S. Preventive Services Task Force (USPSTF) finds insufficient evidence to recommend screening unless there are parental concerns. The AAP recommends screening for iron deficiency at 12 months, but the USPSTF finds insufficient evidence for screening at this time. Otoacoustic emissions (OAE) testing is performed during the newborn screening and is not recommended at 6 months of age.
A 42-year-old female presents to your office with heavy menstrual periods and pelvic pressure. Her symptoms began several years ago and have gradually worsened. Laboratory findings are notable for a mild microcytic anemia. Pelvic ultrasonography identifies a 7-cm submucosal mass. She wants to avoid a hysterectomy but desires a treatment that will provide symptom relief, decrease the volume of the mass, and have a sustained effect.
Which one of the following would be most appropriate for this patient?
A) Expectant management
B) A GnRH agonist
C) A selective estrogen receptor modulator
D) A levonorgestrel-releasing IUD (Mirena)
E) Uterine artery embolization and occlusion
ANSWER: E
This patient presents with a symptomatic fibroid. Although she does not express a desire to maintain fertility, she prefers uterine preservation. The Agency for Healthcare Research and Quality Effective Health Care Program review found consistent evidence that uterine artery embolization and occlusion is effective for reducing fibroid size, with lasting effects up to 5 years and moderate evidence for reducing bleeding and improving quality of life. Expectant management is an appropriate option only for patients who have asymptomatic fibroids. GnRH agonists are effective for providing symptom relief and reducing fibroid size, but their use results in a hypoestrogenized state and should not be continued long term for a sustained effect in premenopausal women. Treatment with a selective estrogen receptor modulator such as raloxifene does not affect fibroid size or bleeding patterns. There is limited data regarding the efficacy of a levonorgestrel-releasing IUD for the treatment of uterine fibroids.
A 58-year-old male with a history of a neurogenic bladder comes to your office as a new patient. He recently elected to have placement of a chronic indwelling urethral catheter rather than performing intermittent catheterization at home, and he asks how to reduce his risk of urinary tract infections (UTIs). His last UTI was approximately 1 year ago and required intravenous antibiotics.
Which one of the following is most effective for preventing UTIs in patients with chronic indwelling urethral catheters?
A) Routine daily hygiene of the meatal surface with soap and water
B) Daily periurethral cleaning with iodine
C) Daily oral antibiotics based on prior urine culture sensitivities
D) Routine instillation of an antimicrobial solution into the drainage bag
E) Regularly scheduled catheter exchanges at fixed intervals
ANSWER: A
Although use of chronic indwelling urethral catheters should be avoided whenever possible, there are still some patients that will require one. Prevention of catheter-associated urinary tract infections (CAUTIs) is important. The most important measure to prevent CAUTIs is routine cleaning of the meatal surface with soap and water while bathing or showering. Use of specific periurethral antiseptics or instillation of antiseptics into the drainage bag does not reduce rates of CAUTI. Daily oral antibiotics are not indicated to prevent CAUTIs. Catheters and drainage bags should only be changed when clinically indicated, such as when there is an infection or obstruction.
A 68-year-old male presents with a burn on his lower leg after trying to light a bonfire with kerosene. Examination of the affected leg reveals the presence of blistering, along with a denuded central area that does not blanch with pressure. The underlying fat and connective tissue are not involved.
Which one of the following is the proper classification of this burn?
A) Superficial burn
B) Superficial partial-thickness burn
C) Deep partial-thickness burn
D) Full-thickness burn
ANSWER: C
Decisions regarding the management of burn wounds depend on first identifying the depth of the burn. Superficial burns are red, painful, and blanching, and they do not blister. Superficial partial-thickness burns blister and blanch with pressure. Deep partial-thickness burns blister, but do not blanch with pressure. Full-thickness burns extend through the entire dermis and into the underlying tissues, and they are dry and leathery. Patients with deep partial-thickness or full-thickness burns should be evaluated by a burn specialist.
Superficial burn
Red painful and blanching, do not blister
Superficial partial thickness burn
blister and blanch with pressure
Deep partial thickness burns
blister but DO NOT blanch
Full thickness burns
Extend entire dermis into underlying tissue
A 71-year-old female with a history of well controlled hypertension, diabetes mellitus, and osteoporosis presents with a 2-day history of fever, chills, and a productive cough. She lives at home with her husband, who has not noted any confusion but says she has been weak and unable to bathe herself.
On examination the patient has a temperature of 38.2°C (100.8°F), a blood pressure of 110/68 mm Hg, unlabored respirations at a rate of 22/min, and an oxygen saturation of 94% on room air. You note that she has good air entry, there are no abnormal breath sounds, and there is no egophony or increased fremitus. The cardiovascular examination is unremarkable.
Laboratory Findings
WBCs elevated
Hemoglobin low
Platelets normal
Creatinine normal
BUN normal
14,000/mm3 (N4500–11,000) 12.5g/dL(N14.0–17.5) 250,000/mm3 (N150,000–350,000) 1.0mg/dL(N0.6–1.2)
14 mg/dL (N 8–23) Posteroanterior and lateral chest radiographs show an infiltrate in the right middle lobe.
Which one of the following would be the most appropriate treatment for this patient?
A) Azithromycin (Zithromax)
B) Amoxicillin plus metronidazole (Flagyl)
C) Amoxicillin/clavulanate (Augmentin) plus azithromycin
D) Azithromycin plus levofloxacin
E) Clindamycin (Cleocin) plus doxycycline
ANSWER: C
Community-acquired pneumonia (CAP) is an infection of the lung parenchyma that is not acquired in a hospital, long-term care facility, or other health care setting, and it is a significant cause of morbidity and mortality in adults. This patient has CAP in the presence of a significant comorbidity (diabetes mellitus). After CAP is diagnosed the first decision to make is whether hospitalization is needed. In all patients with CAP, mortality and severity prediction scores should be used to determine inpatient versus outpatient care (SOR A). This patient has a CURB-65 score of 1 (age 65 years), so she can be treated as an outpatient.
For outpatients with comorbidities, amoxicillin/clavulanate is a possible treatment option, but it should be paired with a macrolide. Macrolides such as azithromycin are the treatment of choice for previously healthy outpatients with no history of antibiotic use within the past 3 months. Azithromycin monotherapy, amoxicillin plus metronidazole, azithromycin plus levofloxacin, or clindamycin plus doxycycline would not be appropriate treatment strategies for this patient with a significant comorbidity.
An 8-year-old female is brought to your office because of left arm pain after she fell down on the sidewalk while roller skating. She has pain, swelling, and a mild deformity of her distal forearm over the radius. Posteroanterior and lateral radiographs confirm an incomplete compression fracture of the distal radius.
In addition to a short arm splint, which one of the following would be appropriate management of this fracture?
A) Ultrasonography in 3 weeks
B) Repeat radiography in 4 weeks
C) Return to activity in 4 weeks if she is pain free
D) Follow-up and reevaluation in 6 weeks
E) Referral to an orthopedist
ANSWER: C
This patient has a compression fracture of the distal radius, also known as a buckle fracture. There is no cortical disruption and these are inherently stable fractures. Radiography or ultrasonography may be used as the initial imaging study if a buckle fracture is suspected. Treatment consists of short arm immobilization, which is most easily performed with a removable splint or wrist brace. The Choosing Wisely campaign states that these fractures do not require repeat imaging if there is no longer any tenderness or pain with palpation after 4 weeks of splinting, and the patient can return to full activity as tolerated. These fractures do not require referral to an orthopedist and can be managed in the office.
An obese 32-year-old male is admitted to the hospital with a new onset of acute pancreatitis. A lipid panel reveals a triglyceride level of 1150 mg/dL and an HDL-cholesterol level of 30 mg/dL. Other laboratory studies are normal. His 10-year risk of atherosclerotic cardiovascular disease is <5%. His family history is positive for recurrent pancreatitis in his father and paternal grandfather.
In addition to lifestyle modifications, which one of the following would be most appropriate for this patient after he is discharged?
A) Atorvastatin (Lipitor)
B) Colesevelam (Welchol)
C) Ezetimibe (Zetia)
D) Fenofibrate (Tricor)
E) Omega-3-acid ethyl esters (Lovaza)
ANSWER: D
Fibrates reduce the likelihood and recurrence of pancreatitis due to severe hypertriglyceridemia when triglyceride levels are 500 mg/dL, measured in a fasting or nonfasting state (SOR A). This patient’s risk of atherosclerotic cardiovascular disease is <7.5% and his LDL-cholesterol level is within normal range, so initiating a statin or ezetimibe is not indicated. Colesevelam may be used to reduce LDL-cholesterol and glucose levels but is not considered a first-line treatment. Omega-3-acid ethyl esters will reduce the triglyceride levels but this patient has severe hypertriglyceridemia, so fibrate therapy is recommended to prevent recurrent pancreatitis.
A 23-year-old gravida 1 para 1 who is a single mother of a 3-day-old infant comes to your office for a newborn follow-up. She reports some sleep disturbance, mild depression without suicidal ideation, and financial concerns. Her past medical history is significant for persistent depressive disorder.
The U.S. Preventive Services Task Force recommends which one of the following to help prevent perinatal depression in patients such as this?
A) Exercise
B) Amitriptyline
C) Sertraline (Zoloft)
D) Referral for cognitive-behavioral therapy
ANSWER: D
The U.S. Preventive Services Task Force (USPSTF) recommends counseling interventions to prevent perinatal depression in patients who are at risk. This patient has risk factors for perinatal depression, including young age, single motherhood, and a history of depression. Other risk factors include low socioeconomic status and depressive symptoms. The USPSTF found that the benefits of counseling interventions outweigh the harms. The USPSTF could not find evidence that exercise, amitriptyline, or sertraline were beneficial.
A 34-year-old male sees you because he was recently informed that a partner he had unprotected sex with last month has been diagnosed with HIV. You would advise this patient to initiate ongoing antiretroviral therapy
A) immediately, because HIV testing is not necessary prior to initiation
B) at the time of diagnosis of HIV infection
C) when his CD4 cell count drops to <200 cells/ L
D) when his CD4 cell count drops to <500 cells/ L
E) when he develops an AIDS-defining illness
ANSWER: B
Antiretroviral therapy (ART) should be prescribed at the time of diagnosis of HIV infection unless the patient has expressed a desire to not initiate treatment. ART should not be delayed until the CD4 cell count drops to a predetermined level or until an AIDS-defining illness occurs. It is recommended to initiate prophylaxis for Pneumocystis pneumonia when the CD4 cell count drops below 200 cells/ L.
An 82-year-old female with atrial fibrillation treated with digoxin is started on verapamil sustained-release capsules (Calan SR) for hypertension and angina. Although she initially tolerates the medication and has a good clinical response, when you see her 1 month later she has lost 3 kg (7 lb) and reports persistent anorexia and nausea over the past 2–3 weeks. A serum chemistry profile, TSH level, and CBC are normal. Her serum digoxin level is 1.4 ng/mL (therapeutic range 0.8–1.5). Her vital signs are stable and a physical examination is notable only for rate-controlled atrial fibrillation.
Which one of the following would be most appropriate at this point?
A) Prescribe a therapeutic trial of an H2-blocker
B) Order an upper gastrointestinal contrast study
C) Withhold digoxin for several days and reinstitute at a lower dosage if necessary
D) Order imaging to look for a central nervous system abnormality
ANSWER: C
This patient presents with typical symptoms of digitalis toxicity, which is common in elderly patients and may occur when the serum level is in the suggested therapeutic range. When drug toxicity is suspected the first step in management would be to discontinue or reduce the dosage of the suspected agent. In this case the addition of verapamil will increase the serum level of digoxin, which will reach a new steady-state level in several days. Therefore, the side effects may not occur for several days while the level is increasing. Because this patient has a good response to verapamil, it is not advisable to stop it, as the digitalis toxicity should be reversed by a dosage reduction. A therapeutic trial of an H2-blocker, an upper gastrointestinal contrast study, or imaging to look for a central nervous system abnormality would not be appropriate at this time.
A 58-year-old male sees you for evaluation of left ankle pain after he slipped on some ice in his driveway last night. He felt immediate pain over the lateral ankle, which started swelling over the next hour. He elevated his foot, applied ice, and took ibuprofen. This morning the ankle remains swollen and also appears bruised. He is able to walk on it with some pain. On examination you note typical findings of an ankle sprain. He asks you for medication to manage his pain.
Which one of the following medications has the best evidence for providing pain relief while also minimizing side effects?
A) Topical diclofenac gel (Pennsaid)
B) Topical menthol gel
C) Oral hydrocodone/acetaminophen (Lortab)
D) Oral ibuprofen
E) Oral tramadol
ANSWER: A
In 2020 the American College of Physicians and the American Academy of Family Physicians published a guideline regarding the treatment of acute pain from musculoskeletal injuries (non–low back related). This systematic review found good evidence to support the recommendation that topical NSAIDs be used as first-line therapy to reduce pain and improve physical function. Topical NSAIDs were the only intervention that improved multiple outcomes and were not associated with a statistically significant increase in the risk for adverse events. Oral NSAIDs and acetaminophen were recommended as second-line therapies, as they were found to be effective for pain relief but were associated with an increased risk for adverse events. Topical menthol gel was not found to be effective as monotherapy but may be considered when combined with a topical NSAID. The guideline specifically recommends avoidance of opioids, including tramadol, noting a prevalence of 6% for prolonged opioid use resulting from an initial prescription. Nonpharmacologic approaches with evidence of benefit include specific acupressure and use of a transcutaneous electrical nerve stimulation (TENS) unit.
A 7-year-old male is brought to the urgent care clinic with a 2-day history of fever and sore throat, with no associated cough. His temperature is 38.3°C (100.9°F) and a rapid antigen test confirms a group A -hemolytic Streptococcus infection. A prescription for penicillin is sent to the pharmacy, but the medication is never picked up due to a lack of transportation. The patient is brought to your office 2 weeks later with a fever, joint pain, shortness of breath, and chest pain. His vital signs are significant for a temperature of 38.8°C (101.8°F) and a heart rate of 118 beats/min.
On examination the patient’s affect is appropriate, he has a 3/6 holosystolic murmur heard best over the apex, and he has tenderness and swelling of his knees bilaterally and of his left ankle. An antistreptolysin O titer is positive, his erythrocyte sedimentation rate is 124 mm/hr (N <10), and a chest radiograph is significant for cardiomegaly.
Which one of the following would be the most appropriate therapy?
A) Hydroxychloroquine (Plaquenil)
B) Methylprednisolone acetate (Depo-Medrol)
C) Naproxen
D) Intravenous immunoglobulins
E) Plasmapheresis
ANSWER: C
Using the Jones criteria for diagnosis, this patient has acute rheumatic fever, with two major criteria (carditis and polyarthritis) and two minor criteria (fever and positive erythrocyte sedimentation rate). NSAIDs such as naproxen can provide significant relief and should be administered as soon as acute rheumatic fever is diagnosed (SOR B). Hydroxychloroquine is not FDA approved for the treatment of acute rheumatic fever and would not be appropriate. Treatment with corticosteroids, intravenous immunoglobulins, and plasmapheresis is not considered appropriate for acute rheumatic fever but may be indicated for management of pediatric autoimmune neuropsychiatric disorders associated with streptococcal infections (PANDAS).
A 51-year-old female presents with concerns about a change in her cognition. She says she has difficulty retrieving words, loses her train of thought, and goes into a room and forgets why she came there. She also has had more frequent hot flashes and sleep disturbances. She still menstruates but has noticed a change from her previous pattern. A physical examination is unremarkable, and recent laboratory tests were all normal, including vitamin B12 and thyroid studies. Cognitive testing is normal.
Which one of the following would be the most appropriate next step?
A) Reassurance only
B) CT of the head
C) MRI of the brain
D) Hormone therapy
E) Referral to a neurologist
ANSWER: A
Women experience subjective cognitive difficulties during their menopausal transition. This may include retrieving numbers or words, losing one’s train of thought, forgetting appointments, and forgetting the purpose of behavior such as entering a room. Clinical studies of these women showed intact cognitive test performance. The treatment consists of patient education and reassurance, since studies have shown that 62% of women report subjective cognitive problems during their menopausal transition. Imaging and referral to a neurologist are not indicated, and there are no trials that support the use of hormone therapy.
An otherwise healthy 46-year-old female presents with a 10-day history of recurring pain in the right cheek and gums. She says the pain feels like electric shocks lasting a few seconds and recurring “hundreds of times a day.” She says that smiling and brushing her teeth can trigger the pain. She does not have a history of recent dental work, trauma, fever, or myalgia. She feels well aside from the facial pain. She has tried over-the-counter analgesics without relief. Her vital signs and an HEENT examination are normal. There is no pain with dental percussion, and the skin and mucous membranes of the nose and mouth are unremarkable.
Which one of the following would be the most appropriate treatment for this patient’s condition?
A) Carbamazepine (Tegretol), 200 mg twice daily
B) Prednisone, 40 mg daily
C) Sumatriptan (Imitrex), 6 mg subcutaneously
D) Valacyclovir (Valtrex), 1000 mg three times daily
E) High-flow oxygen for 20 minutes
ANSWER: A
Trigeminal neuralgia is a clinically diagnosed condition that is characterized by brief, sudden, unilateral pain in the distribution of one of the three branches of the trigeminal nerve. The pain, which is often triggered by minimal stimulus, is paroxysmal with episodes of remission that can last for months. First-line treatment for the condition is carbamazepine or oxcarbazepine (SOR C). Other suggested treatments include lamotrigine, baclofen, and surgical treatments, including microvascular decompression. There is a paucity of good evidence for treatments other than carbamazepine.
Valacyclovir would be an appropriate treatment for herpes zoster, which can also cause unilateral electrical shock–like pain in the face. However, patients with herpes zoster typically would have developed the characteristic lesions by 10 days. In addition, the pain is rarely paroxysmal in nature with herpes zoster. Short-term corticosteroids are not recommended for trigeminal neuralgia and there is minimal evidence of their effectiveness for herpes zoster. Sumatriptan is an appropriate treatment for migraines that are also unilateral but are associated with photophobia, phonophobia, nausea, and persistent severe pain.
High-flow oxygen is used to relieve cluster headaches that occur in the orbital, temporal, or supraorbital areas and persist for 15–180 minutes. Cluster headaches are accompanied by tearing and nasal discharge.
A 68-year-old male with a history of COPD, hypertension, and hyperlipidemia presents with a worsening cough and dyspnea with exertion over the past 3 months. His symptoms were previously well controlled with tiotropium (Spiriva) daily and albuterol (Proventil, Ventolin) as needed, and he has not had any COPD exacerbations in the past year until these symptoms began. He has not had any change in sputum production. Recently he has been using his albuterol inhaler several times a day to help relieve his shortness of breath with exertion.
A physical examination reveals a temperature of 37.0°C (98.6°F), a heart rate of 78 beats/min, a respiratory rate of 16/min, a blood pressure of 144/82 mm Hg, and an oxygen saturation of 95% on room air. A cardiac evaluation reveals a regular rate and rhythm and he has no peripheral edema or cyanosis. His lungs are clear with no wheezes or crackles, and there is a mild prolonged expiratory phase.
According to current GOLD guidelines, which one of the following would be the most appropriate next step in the management of this patient’s symptoms?
A) Add azithromycin (Zithromax)
B) Add inhaled fluticasone (Flovent)
C) Add inhaled salmeterol (Serevent)
D) Add inhaled fluticasone/salmeterol (Advair)
E) Discontinue tiotropium and start inhaled fluticasone
ANSWER: C
COPD is currently the third leading cause of death in the United States and is commonly treated by primary care providers. In patients on monotherapy with a long-acting bronchodilator such as a long-acting muscarinic agonist (LAMA) or long-acting -agonist (LABA) who have continued dyspnea, the Global Initiative for Chronic Obstructive Lung Disease (GOLD) guidelines recommend escalating therapy to two bronchodilators. This patient has persistent dyspnea and is being treated with a single agent, a LAMA, so his regimen needs to be escalated to include a LABA such as salmeterol. Once the symptoms are stabilized, treatment can be de-escalated to a single agent. For patients with frequent COPD exacerbations or with a diagnosis of asthma and COPD, the guidelines recommend adding an inhaled corticosteroid (ICS) such as fluticasone to a LABA, LAMA, or both. Triple therapy with a LABA, a LAMA, and an ICS is not indicated at this time as the patient has not yet been treated with a combination of a LAMA and LABA and has not had any recent exacerbations. The addition of azithromycin may be considered in patients who are already on triple therapy with a LABA, a LAMA, and an ICS and still having exacerbations. Monotherapy with an ICS is not indicated in COPD and has been shown to increase the risk of developing pneumonia.
A 69-year-old male presents with a several-week history of difficulty swallowing that has gradually worsened. At first he noted trouble with passing larger boluses of food through his mid-chest area. Now he states that even ice cream is a problem. He has a past history of GERD that he has treated with omeprazole (Prilosec) intermittently over the past several years. He also takes enteric-coated aspirin, 81 mg daily, for his heart. There is no history of tobacco use. He drinks about six beers a week. On examination you note a 5-kg (11-lb) weight loss over the past 3 months but the remainder of the examination is normal.
Which one of the following would be the most appropriate next step in the evaluation?
A) A trial of omeprazole, 40 mg daily, for 8 weeks
B) Chest CT with and without contrast
C) Barium esophagography
D) High-resolution esophageal manometry
E) Esophagogastroduodenoscopy
ANSWER: E
This patient has several risk factors for a significant problem such as esophageal cancer, including age over 50, weight loss, and progressive symptoms. Esophagogastroduodenoscopy (EGD) is needed without delay. This approach would allow biopsy of any lesions seen and therapeutic dilatation if a benign-appearing stricture is noted. Biopsies are also needed to diagnose eosinophilic esophagitis. If the EGD does not identify a problem, further workup should then proceed. CT may identify a source of extrinsic pressure. Barium esophagography may detect mild narrowing or esophageal webs missed on EGD. Esophageal motility disorders may be diagnosed with esophageal manometry. Waiting 8 weeks to see if his symptoms improve with regular use of a proton pump inhibitor would not be appropriate in this patient with symptoms that are worrisome for esophageal cancer.
An otherwise healthy 57-year-old male presents with mild fatigue, decreased libido, and erectile dysfunction. A subsequent evaluation of serum testosterone reveals hypogonadism.
Which one of the following would you recommend at this time?
A) No further diagnostic testing
B) A prolactin level
C) A serum iron level and total iron binding capacity
D) FSH and LH levels
E) Karyotyping
ANSWER: D
In men who are diagnosed with hypogonadism with symptoms of testosterone deficiency and unequivocally and consistently low serum testosterone concentrations, further evaluation with FSH and LH levels is advised as the initial workup to distinguish between primary and secondary hypogonadism. If secondary hypogonadism is indicated by low or inappropriately normal FSH and LH levels, prolactin and serum iron levels and measurement of total iron binding capacity are recommended to determine secondary causes of hypogonadism, with possible further evaluation to include other pituitary hormone levels and MRI of the pituitary. If primary hypogonadism is found, karyotyping may be indicated for Klinefelter’s syndrome.
A 35-year-old female comes to your office for evaluation of a tremor. During the interview you note jerking movements first in one hand and then the other, but when the patient is distracted the symptom resolves. Aside from the intermittent tremor the neurologic examination is unremarkable. She does not drink caffeinated beverages and takes no medications.
Which one of the following is the most likely diagnosis?
A) Parkinson’s disease
B) Cerebellar tremor
C) Essential tremor
D) Physiologic tremor
E) Psychogenic tremor
ANSWER: E
Psychogenic tremor is characterized by an abrupt onset, spontaneous remission, changing characteristics, and extinction with distraction. Cerebellar tremor is an intention tremor with ipsilateral involvement on the side of the lesion. Neurologic testing will reveal past-pointing on finger-to-nose testing. CT or MRI of the head is the diagnostic test of choice. Parkinsonian tremor is noted at rest, is asymmetric, and decreases with voluntary movement. Bradykinesia, rigidity, and postural instability are generally noted. For atypical presentations a single-photon emission CT or positron emission tomography may help with the diagnosis. One of the treatment options is carbidopa/levodopa.
Patients who have essential tremor have symmetric, fine tremors that may involve the hands, wrists, head, voice, or lower extremities. This may improve with ingestion of small amounts of alcohol. There is no specific diagnostic test but the tremor is treated with propranolol or primidone. Enhanced physiologic tremor is a postural tremor of low amplitude exacerbated by medication. There is usually a history of caffeine use or anxiety.
A nulliparous 34-year-old female comes to your office for evaluation of fatigue, hair loss, and anterior neck pain. These symptoms have been gradually worsening for the past few months. Her past medical history is unremarkable. She has gained 5 kg (11 lb) since her last office visit 18 months ago. Examination of the thyroid gland reveals tenderness but no discrete nodules. Her TSH level is 7.5 U/mL (N 0.4–4.2), her T4 level is low, and her thyroid peroxidase antibodies are elevated.
Which one of the following would be the most appropriate next step?
A) Continue monitoring TSH every 6 months
B) Begin thyroid hormone replacement and repeat the TSH level in 6–8 weeks
C) Begin thyroid hormone replacement and repeat the TSH level along with a T3 level in
6–8 weeks
D) Order ultrasonography of the thyroid
E) Order fine-needle aspiration of the thyroid
ANSWER: B
This patient has thyroiditis with biochemical evidence for autoimmune (Hashimoto’s) thyroiditis. The most appropriate plan of care is to begin thyroid hormone replacement and monitor with a repeat TSH level 6–8 weeks later. It is not necessary to include a T3 level when assessing the levothyroxine dose. There is no need to routinely order thyroid ultrasonography when there are no palpable nodules on a thyroid examination. Fine-needle aspiration may be necessary to rule out infectious thyroiditis when a patient presents with severe thyroid pain and systemic symptoms.
A 70-year-old male presents to your office for follow-up after he was hospitalized for acute coronary syndrome. He has not experienced any pain since discharge and is currently in a supervised cardiac rehabilitation exercise program. His medications include aspirin, lisinopril (Prinivil, Zestril), and metoprolol, but he was unable to tolerate atorvastatin (Lipitor), 40 mg daily, because he developed muscle aches.
Which one of the following would you recommend?
A) Evolocumab (Repatha)
B) Ezetimibe/simvastatin (Vytorin)
C) Fenofibrate (Tricor)
D) Niacin
E) Omega-3 fatty acid supplements
ANSWER: B
High-intensity statin therapy is recommended for patients younger than 75 years of age with known coronary artery disease. For those who are intolerant of high-intensity statins, a trial of a moderate-intensity statin is appropriate. There is evidence to support ezetimibe plus a statin in patients with acute coronary syndrome or chronic kidney disease. Omega-3 fatty acids, fibrates, and niacin should not be prescribed for primary or secondary prevention of atherosclerotic cardiovascular disease because they do not affect patient-oriented outcomes. PCSK9 inhibitors such as evolocumab are injectable monoclonal antibodies that lower LDL-cholesterol levels significantly and have produced some promising results, but more studies are needed to determine when this would be cost effective.
A 30-year-old gravida 1 para 0 develops erythematous patches with slightly elevated scaly borders during her first trimester. There was a 2-cm herald patch 2 weeks before multiple smaller patches appeared. The rash on the back has a “Christmas tree” pattern. She has not had any prenatal laboratory work.
This condition is associated with
A) no additional pregnancy risk
B) a small-for-gestational-age newborn
C) congenital cataracts
D) multiple birth defects
E) spontaneous abortion
ANSWER: E
This patient has classic pityriasis rosea. This is generally a benign disease except in pregnancy. The epidemiology and clinical course suggest an infectious etiology. Pregnant women are more susceptible to pityriasis rosea because of decreased immunity. Pityriasis rosea is associated with an increased rate of spontaneous abortion in the first 15 weeks of gestation. It is not associated with an increased risk for a small-for-gestational-age newborn, congenital cataracts, or multiple birth defects.
In asymptomatic patients with sarcoidosis, which one of the following organ systems should be examined yearly to detect extrapulmonary manifestations of the disease?
A) Cardiac
B) Neurologic C) Ocular
D) Integumentary
ANSWER: C
Sarcoidosis has numerous extrapulmonary manifestations. Because inflammation of the eye can result in permanent impairment and is often asymptomatic, patients require yearly eye examinations as well as additional monitoring with disease flares. Although skin involvement is common it is usually readily apparent and rarely has serious sequelae. Cardiac sarcoidosis can potentially lead to progressive heart failure and sudden death, but evaluation is needed only in patients who are symptomatic. Similarly, evaluation for neurologic involvement is needed only in patients who are symptomatic.
A 57-year-old male presents with left posterior heel pain that started several weeks ago. An examination reveals a nodular appearance at the site of insertion of the Achilles tendon to the calcaneus, and local tenderness of the distal tendon.
Which one of the following would be the safest and most appropriate initial management?
A) Local injection with a corticosteroid
B) Local injection with platelet-rich plasma
C) Physical therapy with eccentric calf-strengthening exercises
D) Immobilization of the ankle in a cast or boot for 4–6 weeks
E) Surgical debridement of the calcification about the distal tendon
ANSWER: C
This patient has typical symptoms and findings of Achilles tendinopathy. The best management involves eccentric calf-strengthening exercises. A local injection with corticosteroids or with platelet-rich plasma is ineffective and may increase the risk of a tendon rupture. Immobilization and surgical debridement may be considered if more conservative therapies have failed.
A 32-year-old female presents with a 4-month history of nasal drainage, congestion, and loss of her sense of smell. She reports having a cold about 4 months ago that never resolved. On examination the nasal turbinates are swollen and you note mucopurulent drainage on the right.
Which one of the following is the most likely cause of her symptoms?
A) Chronic rhinosinusitis
B) Granulomatosis with polyangiitis (Wegener’s granulomatosis)
C) Nasal polyposis
D) Sarcoidosis
E) Seasonal allergic rhinitis
ANSWER: A
The American Academy of Otolaryngology defines chronic rhinosinusitis as the presence of two of four cardinal symptoms, which include nasal drainage, nasal obstruction, facial pain or pressure, and hyposmia or anosmia, along with objective signs on examination or radiographic studies. This patient has three cardinal symptoms of chronic rhinosinusitis and objective evidence on the physical examination. No nasal polyps were seen on the examination. Granulomatosis with polyangiitis and sarcoidosis can both present similarly but are uncommon causes of chronic rhinosinusitis. Allergic rhinitis can be associated with chronic rhinosinusitis but would also present with allergic symptoms.
A 19-year-old female member of a college cross-country team presents with a 1-week history of right knee pain. She does not have any acute injury to the knee. An examination reveals no deformity and she has a normal gait. She has tenderness and subtle swelling localized 1 cm distal to the right medial joint line, and examinations of the knee and hip are otherwise normal.
Which one of the following is the most likely diagnosis?
A) Fibular head stress fracture
B) Iliotibial band syndrome
C) Medial meniscal tear
D) Pes anserine bursitis
E) Tibial apophysitis (Osgood-Schlatter disease)
ANSWER: D
This patient has medial knee pain related to repetitive use, most likely caused by pes anserine bursitis. Iliotibial band syndrome is often related to overuse but causes pain in the lateral knee. The fibular head is also lateral to the knee joint. Osgood-Schlatter disease is also often related to overuse but causes pain at the insertion of the patellar ligament on the midline proximal tibia. A medial meniscal tear would localize to the medial joint line rather than distal to the joint line and would more likely be associated with positive findings from other examinations, such as a McMurray test.
When titrating the dosage of opioids, the CDC recommends that you should also consider prescribing naloxone when the opioid dosage reaches what morphine milligram equivalent (MME) per day threshold?
A) 30 B) 50 C) 80 D) 90 E) 100
ANSWER: B
To mitigate the risk of opioid harm, it is essential to understand morphine milligram equivalents (MME). The evidence shows that the risk of an opioid overdose increases at the threshold of 50 MME/day. It is therefore recommended by the CDC that a prescription for naloxone be ordered when an opioid dosage reaches 50 MME/day, which is a high dosage. In general one should avoid prescribing 90 MME/day because of the substantially higher risk of an overdose at this dosage level.
A 62-year-old female with stage 3 chronic kidney disease and an estimated glomerular filtration rate of 37 mL/min/1.73 m2 is found to have a mildly low ionized calcium level. Which one of the following would you expect to see if her hypocalcemia is secondary to her chronic kidney disease?
A) Elevated parathyroid hormone (PTH) and elevated phosphorus
B) Elevated PTH and low phosphorus
C) Low PTH and elevated phosphorus
D) Low PTH and low phosphorus
ANSWER: A
Chronic kidney disease–mineral and bone disorder (CKD-MBD) is found in many patients with CKD and is associated with an increased risk of bone fractures and cardiovascular events due to vascular calcification. In patients with CKD, phosphate is not appropriately excreted and the subsequent hyperphosphatemia leads to secondary hyperparathyroidism and binding of calcium. Decreased production of calcitriol in patients with CKD also leads to hypocalcemic hyperparathyroidism. Patients with CKD stages 3a–5 should have phosphorus, calcium, parathyroid hormone, and 25-hydroxyvitamin D levels checked regularly, and consultation with a nephrologist or endocrinologist should be obtained if CKD-MBD is suspected.
A 63-year-old female sees you for evaluation of recurrent right foot swelling and redness. She has a history of obesity and type 2 diabetes with retinopathy, nephropathy, and peripheral neuropathy. She presented with similar symptoms 2 weeks ago and was diagnosed with cellulitis and treated with a 10-day course of amoxicillin/clavulanate (Augmentin). Her symptoms seemed to initially improve with this therapy along with elevation of the foot but then worsened. She does not have any pain in the foot, fever, or chills. She does not recall any trauma or other inciting event.
The patient’s vital signs include a temperature of 37.1°C (98.8°F), a pulse rate of 72 beats/min, and a blood pressure of 124/82 mm Hg. Her right foot appears swollen, red, and warm to the touch, and is not tender to palpation. There are no open sores or calluses. Her dorsalis pedis pulse is 2+. Monofilament testing confirms a diagnosis of peripheral neuropathy. A WBC count is normal. Radiographs reveal soft-tissue edema with no other abnormalities.
The most appropriate treatment at this point would be
A) immobilization
B) antibiotics
C) bisphosphonates
D) corticosteroids
E) surgical repair
ANSWER: A
This patient has acute Charcot neuroarthropathy, an inflammatory condition that occurs in obese patients with peripheral neuropathy and ultimately leads to foot deformities (the classic rocker-bottom foot) and resultant ulcerations and infections. Its clinical appearance can easily be initially mistaken for cellulitis. However, the absence of tenderness and other signs of infection such as fever, an elevated WBC count, and inflammatory markers is not consistent with cellulitis. Radiography is an appropriate initial imaging modality but the results are often interpreted as normal early in the disease process. MRI is the modality of choice for a definitive diagnosis and may demonstrate periarticular bone marrow edema, adjacent soft-tissue edema, joint effusion, and microtrabecular or stress fractures.
The treatment of acute Charcot neuroarthropathy is immobilization with total contact casting, which increases the total surface area of contact to the entire lower extremity, distributing pressure away from the foot. Immobilization is typically required for at least 3–4 months but in some cases may be needed for up to 12 months. Bisphosphonates were found to be ineffective as adjunctive therapy in acute Charcot neuroarthropathy. Corticosteroids and antibiotics have no role in the treatment of Charcot foot but would be appropriate therapy for cellulitis or gout, which are important alternative diagnoses to consider. The role of surgery is more controversial but may be indicated in the acute phase of Charcot neuroarthropathy in patients with severe dislocation or instability.
A 38-year-old female with a 6-month history of mild shortness of breath associated with some intermittent wheezing during upper respiratory infections presents for follow-up. You previously prescribed albuterol (Proventil, Ventolin) via metered-dose inhaler, which she says helps her symptoms. You suspect asthma. Pulmonary function testing reveals a normal FEV1/FVC ratio for her age.
Which one of the following would be the most appropriate next step?
A) Consider an alternative diagnosis
B) Assess her bronchodilator response
C) Perform a methacholine challenge
D) Prescribe an inhaled corticosteroid
E) Proceed with treatment for COPD
ANSWER: C
Spirometry is central to confirming the diagnosis of asthma, which is characterized by a reversible obstructive pattern of pulmonary function. In this case the patient’s FEV1/FVC ratio is normal, which neither confirms nor rules out asthma. A methacholine challenge is recommended in this scenario to assess for the airway hyperresponsiveness that is the hallmark of asthma. Methacholine is a cholinergic agonist. Bronchoconstriction (defined as a reduction in FEV1 20%) observed at low levels of methacholine administration (<4 mg/mL) is consistent with asthma. If the FEV1/FVC ratio is reduced on initial spirometry, a bronchodilator response should be tested. A fixed or partially reversible obstructive pattern suggests an alternative diagnosis such as COPD, and full reversal after bronchodilator use is consistent with asthma. Inhaled corticosteroids are not appropriate for intermittent asthma.
A 67-year-old male diagnosed with polymyalgia rheumatica is started on long-term prednisone therapy. Which one of the following is the recommended first-line agent to prevent steroid-induced osteoporosis?
A) Alendronate (Fosamax)
B) High-dose vitamin D
C) Raloxifene (Evista)
D) Teriparatide (Forteo)
ANSWER: A
Patients are at risk of developing glucocorticoid-induced osteoporosis if they are on long-term glucocorticoid therapy, defined as >2.5 mg of prednisone for a duration of 3 months or longer. The American College of Rheumatology recommends pharmacologic treatment for these patients, as well as for patients receiving glucocorticoids who have a bone mineral density T-score –2.5 at either the spine or the femoral neck and are either male and 50 years of age or female and postmenopausal. Therapy is also recommended in patients 40 years of age who do not meet these criteria but have a 10-year risk of major osteoporotic fracture of at least 20% or a risk of hip fracture of at least 3% according to the FRAX tool.
Oral bisphosphonates are recommended as first-line agents for preventing glucocorticoid-induced osteoporotic fractures, although intravenous bisphosphonates can be used if patients are unable to use the oral forms. Supplementation of calcium (800–1000 mg) and vitamin D (400–800 IU) is also recommended. Raloxifene and teriparatide are options when bisphosphonate therapy fails or is contraindicated (SOR A).
At a routine well child check, the mother of an 18-month-old female expresses concern about the child’s development. Which one of the following should prompt consideration of a developmental delay?
A) A vocabulary of less than six words
B) Failure to point to pictures or body parts when named
C) Inability to follow one-step directions
D) Inability to run well
E) Inability to copy a vertical line
ANSWER: C
At 18 months of age a child should follow one-step directions. Approximately 90% of 18-month-olds say at least three words, and 50%–90% say six words. The ability to point to body parts or pictures after they are named is expected at 2 years of age. Not walking at 18 months would be a red flag for delay, but running well may not yet be accomplished. At 18 months a child would be expected to scribble spontaneously but not to copy a vertical line.
A 22-year-old male presents to your office the morning after falling onto his outstretched right hand as he tripped while leaving a bar. He has a deep, dull ache in the right wrist on the radial side. The pain is worsened by gripping and squeezing. On examination there is some wrist fullness and the wrist is tender to palpation over the anatomic snuffbox. Radiographs of the wrist are negative.
Which one of the following would be most appropriate at this time?
A) Rest, ice, compression, elevation, and NSAIDs with no specific follow-up
B) Rest, ice, compression, elevation, and NSAIDs with a follow-up examination in 2 weeks
C) Placement of a thumb spica splint, with a follow-up examination in 2 weeks
D) CT of the wrist to detect an occult fracture
E) Ultrasonography of the wrist to detect a ligament injury
ANSWER: C
The history, symptoms, and physical examination findings in this case suggest a scaphoid fracture. The scaphoid bone is the most commonly fractured carpal bone and a fall on an outstretched hand can produce enough force to cause this fracture. This fracture is most common in males 15–30 years of age.
The finding of anatomic snuffbox tenderness is highly sensitive but not specific for a scaphoid fracture. Initial radiographs often do not demonstrate a fracture. When there is a high clinical suspicion for a scaphoid fracture but radiographs are negative, it is reasonable to immobilize in a thumb spica splint and reevaluate in 2 weeks.
A 28-year-old white female comes to your office at 37 weeks gestation with a 24-hour history of painful vesicles on the vulva. She does not have a past history of similar lesions. You make a presumptive diagnosis of genital herpes.
Of the following, the most sensitive and specific test is
A) exfoliative cytology (Tzanck test)
B) a polymerase chain reaction (PCR) test
C) an enzyme-linked immunosorbent assay (ELISA)
D) HSV serology (IgG/IgM)
ANSWER: B
When genital herpes occurs during pregnancy, the best method of diagnosis is either a tissue culture or a polymerase chain reaction (PCR) test, which is more sensitive. Enzyme-linked immunosorbent assays are sensitive, but not as sensitive or specific as PCR.
A 55-year-old patient with a history of alcoholism is admitted through the emergency department with acute pancreatitis. Which one of the following tests performed at the time of admission can best predict the severity of pancreatitis?
A) Hematocrit
B) C-reactive protein
C) Serum amylase
D) Serum lipase
E) CT of the abdomen
ANSWER: A
Knowing the severity of pancreatitis helps predict how aggressive management should be. Hematocrit, BUN, and creatinine levels are the most useful predictors of the severity of pancreatitis, reflecting the degree of intravascular volume depletion. C-reactive protein is often elevated, but it is not as useful as hematocrit for predicting severity. Serum amylase and lipase have no prognostic value. CT evidence of severe pancreatitis lags behind clinical and laboratory evidence, and early CT underestimates the severity of the acute process.
A 34-year-old male with sickle cell disease has a new onset of mild to moderate thirst and polyuria. He ate a large meal about 2 hours ago.
An examination reveals a BMI of 32 kg/m2. Results of a urinalysis performed by your staff include 3+ glucose and no ketones. His blood glucose level is 288 mg/dL and his hemoglobin A1c is 5.2%.
Which one of the following would be most appropriate at this point to help diagnose and monitor this patient’s glycemic control?
A) A serum fructosamine level
B) A repeat hemoglobin A1c
C) A 2-hour glucose tolerance test
D) Hemoglobin electrophoresis
E) Referral to an endocrinologist
ANSWER: A
This patient with sickle cell disease has a new onset of diabetes mellitus. Hemoglobinopathies falsely lower hemoglobin A1c as a result of hemolysis and abnormal glycation. Fructosamine correlates well with hemoglobin A1c levels and is recommended instead of hemoglobin A1c for monitoring glucose control in patients with diabetes and hemoglobinopathies. A 2-hour glucose tolerance test or hemoglobin electrophoresis would not provide useful information. Referral to an endocrinologist is not indicated at this point because the patient has not failed primary care management.
A 14-year-old male sees you for a well child examination. He had one dose of HPV vaccine at his last well child examination 1 year ago.
Which one of the following is true regarding HPV vaccine for this patient?
A) He does not require additional HPV vaccine
B) He should receive one dose of the vaccine now and no additional HPV vaccine in the
future
C) He should receive the vaccine now and again in 4 months
D) He should receive the vaccine now and again in 6 months
E) He should receive the vaccine now, in 2 months, and in 4 months
ANSWER: B
HPV vaccine is currently recommended for males and females at age 11. Catch-up vaccination is recommended until age 21 in males and 26 in females. Children who receive the first dose of the vaccine before the age of 15 and receive two doses are considered adequately vaccinated. If the first dose is given after age 15, a three-dose series is recommended.
A 34-year-old female presents to your office after she was bitten on the hand by a neighbor’s cat. The patient has no allergies and has been in good health. You decide to treat the patient with a prophylactic antibiotic.
Which one of the following would be the antibiotic of choice?
A) Amoxicillin/clavulanate (Augmentin)
B) Azithromycin (Zithromax)
C) Cephalexin (Keflex)
D) Clindamycin (Cleocin)
E) Metronidazole (Flagyl)
ANSWER: A
Prophylactic antibiotics should be given for all closed-fist injuries unless the skin has not been penetrated, and for puncture wounds caused by cat bites. The antibiotic should have both aerobic and anaerobic activity and include Pasteurella coverage for animal bites and Eikenella coverage for human bites. Suggested regimens include amoxicillin/clavulanate. If the patient is allergic to penicillin, clindamycin plus levofloxacin or moxifloxacin, which has anaerobic coverage, can be used. Azithromycin, cephalexin, and metronidazole are not first-line antibiotics following a cat bite.
A 30-year-old female presents for follow-up after an emergency department visit for an episode of symptomatic supraventricular tachycardia that was diagnosed as Wolff-Parkinson-White syndrome. Which one of the following would be most appropriate for the initial long-term management of this patient?
A) Adenosine (Adenocard)
B) Amiodarone (Cordarone)
C) Diltiazem (Cardizem)
D) Metoprolol
E) Catheter ablation
ANSWER: E
Catheter ablation is the most appropriate treatment for a patient with symptomatic Wolff-Parkinson-White syndrome (WPW). Catheter ablation has a very high immediate success rate (96%–98%). The most significant risk associated with the procedure is permanent atrioventricular block, which occurs in approximately 0.4% of procedures. Adenosine and amiodarone are used for the acute management of supraventricular tachycardia, but not for long-term management. Node-blocking medications such as diltiazem and metoprolol should not be used for the long-term treatment of WPW, due to the increased risk of ventricular fibrillation.
A 7-year-old male received one dose of trivalent inactivated influenza vaccine at another health care facility 5 weeks ago. This was the first time he received influenza vaccine, and it resulted in soreness at the injection site. His mother reports that he has had mild hives after eating peanuts and eggs in the past. Your office has stocked only quadrivalent inactivated influenza vaccine.
Which one of the following would you recommend?
A) No further influenza immunization this year
B) Immunization now with quadrivalent inactivated vaccine
C) Delaying immunization until trivalent inactivated vaccine can be given
D) Delaying immunization until 8 weeks after the first vaccine
E) No immunization because of a potential egg allergy
ANSWER: B
The CDC’s Advisory Committee on Immunization Practices recommends that patients with egg allergy receive influenza vaccination. Previously unvaccinated patients ages 6 months to 8 years should receive two doses of either trivalent or quadrivalent vaccine separated by 1 month.
Three weeks after he had knee surgery, a 64-year-old male presents for follow-up of an emergency department visit for a pulmonary embolism. He has no previous history of pulmonary embolism and is otherwise in good health. He is being treated with apixaban (Eliquis).
The recommended duration of anticoagulation therapy for this patient is
A) 1 month
B) 3 months
C) 6 months
D) 9 months
E) 12 months
ANSWER: B
Patients who have a venous thromboembolism (VTE) require anticoagulation therapy for treatment and prevention of recurrence. The risk of recurrence is greatest in the first year after the event and remains elevated indefinitely. The risk for VTE recurrence is dependent on patient factors, such as active cancers and thrombophilia. Current guidelines recommend treatment for at least 3 months. In patients who have a reversible provoking factor such as surgery, anticoagulation beyond 3 months is not recommended.
A 30-year-old gravida 2 para 1 in her second trimester is evaluated for hypothyroidism. The normal TSH range in pregnancy is
A) lower than in the nonpregnant state
B) higher than in the nonpregnant state
C) the same as in the nonpregnant state
D) not useful for evaluating hypothyroidism after the first trimester
ANSWER: A
The TSH reference range is lower during pregnancy because of the cross-reactivity of the -subunit of hCG. Levels of hCG peak during weeks 7–13 of pregnancy, and hCG has mild TSH-like activity, leading to slightly high free T4 levels in early pregnancy. This leads to a feedback decrease in TSH.
A 69-year-old male presents for an annual health maintenance examination. His medical history is significant for hypertension and worsening back pain over the last 6 months. Laboratory studies reveal a hemoglobin level of 8.6 g/dL (N 14.0–18.0) and a mean corpuscular volume of 88 m3 (N 80–94). The remainder of the CBC is normal. A peripheral smear and a ferritin level are both normal. A comprehensive metabolic panel is normal except for a serum creatinine level of 1.6 mg/dL (N 0.7–1.3).
Which one of the following would be the most appropriate next step in the evaluation of this patient?
A) A vitamin B12 level
B) A haptoglobin level
C) Serum protein electrophoresis
D) Flow cytometry
E) A bone marrow biopsy
ANSWER: C
This patient’s laboratory results and back pain suggest multiple myeloma (MM). He has a normocytic anemia and evidence of renal insufficiency, which can indicate MM. The laboratory findings along with worsening back pain indicate a need to order serum protein electrophoresis to look for MM. Flow cytometry is generally used in patients with an elevated WBC count and suspected lymphoma. The remainder of this patient’s CBC is normal, which makes a bone marrow issue less likely. His mean corpuscular volume is also normal, making vitamin B12 deficiency less likely. A haptoglobin level could be ordered, but protein electrophoresis is a better choice because the peripheral smear demonstrated no evidence of a hemolytic problem.
A 32-year-old female presents with heat intolerance, excessive weight loss, and anxiety. She gave birth 6 months ago and recently stopped breastfeeding. On examination her thyroid gland is slightly diffusely enlarged and nontender. Laboratory studies reveal a decreased TSH level and elevated free T3 and T4 levels. You suspect that she has postpartum thyroiditis.
Which one of the following tests would be most useful to confirm the diagnosis?
A) Radioactive iodine uptake
B) Thyroid peroxidase antibody levels
C) Thyroid ultrasonography
D) Thyrotropin receptor antibody levels
ANSWER: A
Postpartum thyroiditis is defined as a transient or persistent thyroid dysfunction that occurs within 1 year of childbirth, miscarriage, or medical abortion. Release of preformed thyroid hormone in the bloodstream initially results in hyperthyroidism. During the hyperthyroid phase, radioactive iodine uptake will be low, which can help to confirm the diagnosis. Pregnancy and breastfeeding are contraindications to radionuclide imaging. Thyroid peroxidase antibody levels are elevated with chronic autoimmune thyroiditis (Hashimoto’s thyroiditis), and patients present with symptoms of hypothyroidism. The Endocrine Society and American Association of Clinical Endocrinologists do not recommend routine thyroid ultrasonography in patients with abnormal thyroid function tests if there is no palpable abnormality of the thyroid gland. Thyrotropin receptor antibody levels are positive in Graves disease.
A patient comes to your outpatient clinic with a persistent migraine that she has been unable to treat effectively at home. The symptoms began several hours ago and are typical for her. She has already tried her usual treatments of ibuprofen, 800 mg, and rizatriptan (Maxalt), 10 mg, but they have not provided any relief. She took a second dose of rizatriptan 2 hours later without benefit. She is in significant pain, which is causing mild nausea, and she has photophobia and phonophobia.
Which one of the following would be most appropriate at this point?
A) Oral butalbital/acetaminophen/caffeine (Fioricet)
B) Oral ergotamine/caffeine (Cafergot)
C) Subcutaneous sumatriptan (Imitrex)
D) Intramuscular morphine
E) Intramuscular prochlorperazine
ANSWER: E
Multiple studies have determined that parenteral antiemetics have benefits for the treatment of acute migraine beyond their effect on nausea. Most outpatient clinics do not have the ability to administer intravenous metoclopramide, which is the preferred treatment. However, most clinics do have the ability to administer intramuscular prochlorperazine or promethazine. Due to concerns about oversedation, misuse, and rebound, treatment with parenteral opiates is discouraged but may be an option if other treatments fail. Oral butalbital/acetaminophen/caffeine and oral ergotamine/caffeine have less evidence of success in the treatment of acute migraine. Sumatriptan is contraindicated within 24 hours of the use of rizatriptan.
A 48-year-old male with an 8-week history of the gradual onset of nonradiating, worsening left heel pain sees you for follow-up. He started running on his neighborhood streets 3 months ago to train for a 10K race. The pain limits his training significantly. His symptoms improved when he began taking ibuprofen and took 2 weeks off from running a month ago. A radiograph of the left foot 4 weeks ago was normal. There is no other pain and he feels well otherwise.
On examination you note that his left heel is slightly swollen compared to the right and very tender when squeezed on the sides. The anterior aspect and Achilles tendon insertion of the heel are nontender. There is no erythema or warmth and the remainder of the left lower extremity examination is normal. His vital signs are normal.
Which one of the following would you recommend at this point?
A) A C-reactive protein level
B) Nerve conduction velocity testing
C) A repeat radiograph
D) Ultrasonography
E) MRI
ANSWER: E
This patient has a calcaneal stress fracture as suggested by the history of increased running on a hard surface, improvement with rest, and a positive calcaneal squeeze on examination. A delay in diagnosis increases the risk of delayed union. MRI is the preferred imaging modality because radiographs often do not detect a calcaneal stress fracture. A C-reactive protein level could be indicated if there were symptoms or signs of infection or autoimmune illness. The clinical picture does not suggest a neurologic condition, so nerve conduction velocity testing is not appropriate. While there are some case reports of the diagnosis of stress fractures using ultrasonography, this is not the preferred imaging method.
A 48-year-old female presents with the nail findings shown below. Her past medical history is significant for systemic lupus erythematosus.
This finding is known as
A) clubbing
B) koilonychia
C) leukonychia
D) onychomycosis
E) squamous cell carcinoma
ANSWER: B
Koilonychia, also known as spoon nail, is a finding that can be associated with multiple systemic conditions such as iron deficiency, hypothyroidism, and systemic lupus erythematosus. Clubbing of the nails involves thickening of the soft tissue proximal to the nail. Leukonychia is a white discoloration of the nail plate and can be a normal variant. Onychomycosis is a fungal infection generally presenting as a thickened, yellow, dystrophic nail. Squamous cell carcinoma presents as an erythematous, proliferating mass that disrupts normal nail morphology.
A previously healthy 45-year-old female presents with upper abdominal pain and dysphagia. An upper GI series reveals no significant reflux. On esophagogastroduodenoscopy the esophagus has a ringed appearance and a biopsy reveals >15 eosinophils/hpf. Helicobacter pylori testing is negative. She does not currently take any medications.
Which one of the following would be the best initial treatment?
A) Budesonide oral suspension, 1 mg twice daily
B) Fexofenadine, 180 mg daily
C) Pantoprazole (Protonix), 40 mg once daily
D) Prednisone, 40 mg daily for 7 days
E) Ranitidine (Zantac), 150 mg once daily
ANSWER: A
The clinical presentation and esophagogastroduodenoscopy findings indicate eosinophilic esophagitis (EoE) in this patient. In the absence of other causes of eosinophilia, the presence of >15 eosinophils/hpf is considered diagnostic. Application of corticosteroids to the esophagus is generally the treatment of choice, either in the form of an oral suspension of budesonide or an inhaled corticosteroid sprayed into the mouth and swallowed. Although EoE can occur in patients with other atopic illnesses, this patient does not have any symptoms of allergies or asthma, so an antihistamine such as fexofenadine is not needed. EoE does not respond to proton pump inhibitor therapy or H2-blocker therapy and systemic corticosteroids are not necessary.
A 72-year-old female presents for a routine health maintenance visit. Which one of the following medications in her current regimen places her at risk for osteoporosis?
A) Atorvastatin (Lipitor)
B) Hydrochlorothiazide
C) Metformin (Glucophage)
D) Phenytoin (Dilantin)
E) Ranitidine (Zantac)
ANSWER: D
Medications reported to be associated with osteoporosis and increased fracture risk include antiepileptic drugs, long-term heparin, cyclosporine, tacrolimus, aromatase inhibitors, glucocorticoids, gonadotropin-releasing hormone agonists, thiazolidinediones, excessive doses of levothyroxine, proton pump inhibitors, SSRIs, parenteral nutrients, medroxyprogesterone contraceptives, methotrexate, and aluminum antacids. Atorvastatin, hydrochlorothiazide, metformin, and ranitidine are not associated with osteoporosis.
A 46-year-old male who uses injectable heroin daily presents to establish primary care. He reports no symptoms and feels well overall. He does not smoke cigarettes, use alcohol, or use any drugs other than heroin. He works as an accountant for the federal government and has not had any arrests for illegal activity.
In addition to assessing his readiness for treatment of his addiction, you should order testing for
A) HIV only
B) HIV, hepatitis B, and hepatitis C
C) HIV, hepatitis B, hepatitis C, and TB
D) HIV, hepatitis A, hepatitis B, hepatitis C, and TB
E) HIV, hepatitis A, hepatitis B, hepatitis C, TB, gonorrhea, and Chlamydia
ANSWER: D
Persons who inject drugs are at increased risk for HIV, hepatitis A, hepatitis B, hepatitis C, and latent tuberculosis. These patients should be screened at the initial visit and treated for any conditions found, according to routine guidelines. If titers are low or absent for hepatitis A or B, the patient should be vaccinated. Although gonorrhea and Chlamydia screening is recommended for females under 25 years of age who use injectable drugs, it is not appropriate in this asymptomatic patient.
A 37-year-old female presents with a 3-day history of left thumb pain. She first noted diffuse pain in her left thumb after a fall while skiing. She does not recall the mechanism of injury. The pain is greatest at the medial metacarpophalangeal joint, but there is no mass or instability. A radiograph does not show any fractures.
An examination reveals mild laxity in her ulnar collateral ligament with 30° of joint opening when abduction stress is applied to the distal thumb while stabilizing the metacarpal. The right side has only 15° of joint opening.
Which one of the following would be most appropriate at this point?
A) Reassurance and follow-up if symptoms do not improve
B) Anti-inflammatory medication for 7–10 days
C) Immobilization of the left thumb in a thumb spica cast or brace for 6 weeks
D) Immediate referral to an orthopedic surgeon for surgical repair
ANSWER: C
Ulnar collateral ligament (UCL) disruption, or “skier’s thumb,” should be suspected in traumatic thumb injuries. It is important to recognize and treat this injury because it can lead to joint instability and a weak pincer grip if untreated. Initial treatment of UCL disruption involves immobilization of the affected thumb in a thumb spica cast or brace for 6 weeks. In the absence of an avulsion fracture, indications for referral to an orthopedic surgeon would include 35°–40° of joint opening or no end point on stress abduction testing. A Stener lesion (entrapment of the UCL outside of the adductor aponeurosis) would usually present with joint instability and a tender mass and would necessitate an orthopedic referral.
A 65-year-old female with hypertension, osteoporosis, and GERD presents to your office for a well woman visit. She reports no new symptoms or concerns. A review of laboratory work performed prior to her visit reveals lipid levels at goal, normal glucose and sodium levels, a calcium level of 10.6 mg/dL (N 8.6–10.3), an albumin level of 4.1 g/dL (N 3.6–5.1), and a 25-hydroxyvitamin D level of 35 ng/mL (N 20–50). Her calcium level was 10.5 mg/dL on a basic metabolic panel 6 months ago.
The patient’s medications include hydrochlorothiazide, 12.5 mg daily; lisinopril (Prinivil, Zestril), 10 mg daily; alendronate (Fosamax), 70 mg weekly; omeprazole (Prilosec), 20 mg daily as needed; and vitamin D, 2000 IU daily. The patient’s blood pressure is 110/60 mm Hg. An examination is normal.
In addition to ordering follow-up laboratory studies and scheduling a follow-up visit in 1 month, which one of the following would be most appropriate?
A) Discontinue alendronate
B) Discontinue hydrochlorothiazide
C) Discontinue lisinopril
D) Discontinue omeprazole and begin ranitidine (Zantac)
E) Increase vitamin D to 5000 IU daily
ANSWER: B
This patient has hypercalcemia with a normal albumin level. Hydrochlorothiazide can cause drug-induced hypercalcemia. Alendronate, lisinopril, and omeprazole do not cause hypercalcemia. A high vitamin D level can cause hypercalcemia, so increasing vitamin D is not appropriate at this point. A laboratory evaluation can help differentiate between PTH- and non–PTH-mediated hypercalcemia.
A 60-year-old patient is admitted to the hospital for a COPD exacerbation. For this patient, which one of the following would be the most appropriate prednisone dosage?
A) 40 mg daily for 5 days
B) 40 mg daily for 10 days
C) 60 mg daily, tapered over 6 days
D) 60 mg daily for 10 days
Item 137
ANSWER: A
For patients with a COPD exacerbation, systemic glucocorticoids can improve FEV1, improve oxygenation, shorten recovery time, and reduce the length of hospitalization (level of evidence A). Prednisone, 40 mg daily for 5 days, is recommended for COPD exacerbations. Studies have shown that oral administration is equally efficacious compared to the intravenous route. The duration of therapy should not be longer than 5–7 days (level of evidence A).
A 17-year-old female presents to your office with open and closed comedones on her nose, forehead, and chin. No cystic lesions are noted and the acne does not extend to her back or chest.
Of the following, which one would be most effective?
A) Topical adapalene (Differin) and benzoyl peroxide
B) Topical clindamycin (Cleocin T)
C) Topical salicylic acid
D) Topical tazarotene (Tazorac)
E) Oral norethindrone/ethinyl estradiol
ANSWER: A
Acne affects 85% of 12- to 25-year-olds in the United States. This patient has mild acne as evidenced by the absence of cystic lesions and localization to the facial T-zone. Topical retinoids are first-line treatment for any level of severity of acne (SOR A). Adapalene is an effective retinoid and is available over-the-counter. Benzoyl peroxide is also very effective in the control of acne because it reduces the concentration of cystic acne with no risk of bacterial resistance. The combination of a topical retinoid and benzoyl peroxide is more effective than either agent alone.
Topical antibiotics such as clindamycin and erythromycin should not be used as monotherapy due to high rates of microbial resistance. There is little evidence that salicylic acid is effective in combating acne despite its widespread use.
Tazarotene is effective in the treatment of acne but is teratogenic (pregnancy category X) and should be avoided in women of reproductive age. Combined oral contraceptives can be effective, but norethindrone and other first-generation progestins are androgenic and can worsen acne.
A 48-year-old male is brought to the medical tent of a marathon after he became confused and dizzy in the middle of the race. On examination his rectal temperature is 41.1°C (106.0°F).
Which one of the following should be initiated immediately?
A) Passive cooling in an air-conditioned space
B) Gradual cooling with a cold water spray and a fan
C) Ice packs applied to the groin, neck, and axilla
D) Full-body immersion in an ice bath
E) Cold intravenous fluids
ANSWER: D
Heatstroke can be nonexertional from prolonged exposure to a high heat index, or it can be exertional, as in this case. A core temperature >40°C (104°F) is consistent with heatstroke. In treating patients with either clinical variant of heatstroke, cold or ice-water immersion is the most effective treatment and should be initiated as soon as possible, without delaying for transfer to the hospital setting (SOR A). Treatment should continue until the core body temperature is <39°C (102°F). If cold water immersion is not possible other forms of cooling such as cold intravenous fluids, ice packs, cold water immersion of the extremities, and evaporative cooling have been shown to have some benefit. Once the body temperature is decreased patients should be transferred to a hospital for evaluation for known complications of heatstroke, including coagulopathy, renal and hepatic dysfunction, hypoglycemia, electrolyte disturbance, and rhabdomyolysis.
A 25-year-old landscaper presents with a 1-day history of pain and swelling in his left hand. Three days ago he sustained a puncture wound in the palm of the hand when he was stuck by a large thorn. He has generalized achiness and chills in addition to the hand pain. He has no significant past medical history.
On examination the patient’s temperature is 37.9°C (100.2°F). His left third finger is diffusely swollen, erythematous, and held in flexion. There is tenderness along the third tendon in the palm. A radiograph of the hand is negative for a fracture or foreign body.
Which one of the following would be the most appropriate next step?
A) Amoxicillin/clavulanate (Augmentin), 875/125 mg twice daily for 10 days
B) Incision and drainage in the office with a culture of the wound
C) A hand splint and nonurgent referral to an orthopedic surgeon
D) Urgent MRI of the hand
E) Urgent surgical consultation
ANSWER: E
Pyogenic flexor tenosynovitis usually develops 2–5 days after a penetrating hand injury. The flexor tendon sheath has a poor vascular supply and the synovial fluid is a prime growth medium for bacteria. Flexor tenosynovitis is a clinical diagnosis characterized by the four “Kanavel” signs: pain with passive extension, tenderness with palpation of the tendon sheath, flexed position of the involved finger, and fusiform swelling of the finger. Treatment includes prompt intravenous antibiotics and surgical debridement and irrigation.
Flexor tenosynovitis requires urgent surgical consultation and treatment. Patients with suspected flexor tenosynovitis should be seen by a surgeon within 72 hours of symptom onset (SOR C). Oral antibiotics and splinting of the hand alone are insufficient treatments for the condition. Incision and drainage would also not be sufficient to clear the infection. Ordering MRI can unnecessarily delay surgical consultation, although the surgeon may obtain one to guide treatment.
You are administering a mental status examination to a 92-year-old male with suspected dementia. You give the patient a pencil and ask him to show how it is used. He gives you a bewildered look and eventually puts the pencil in his mouth and demonstrates using it as a toothbrush.
This task assesses
A) executive functioning
B) gnosia
C) orientation
D) praxis
E) visuospatial proficiency
ANSWER: D
Praxis is the ability to carry out intentional motor acts and is commonly assessed by giving the patient a common object such as a hairbrush or pencil and asking the patient to show how it is used. A patient unable to carry out such motor acts is referred to as having apraxia (SOR C). Several other common components of the cognitive assessment will be impaired in persons with dementia. Executive functioning is the ordering and implementation of cognitive functions necessary to engage in appropriate behavior and is often assessed by asking a patient to draw a clock with the hands set at a certain time.
Gnosia is the ability to name objects and their function and is often assessed by showing a patient a common object such as a pen, watch, or stethoscope and asking whether he or she can identify it and describe how it is used. Orientation is the ability of the patient to recognize his or her place in time and space. Orientation is commonly assessed by asking a patient the date, the current location, his or her name, and his or her place of birth. Visuospatial proficiency is the ability to perceive and manipulate objects and shapes in space. It is often assessed by asking the patient to copy intersecting pentagons or a three-dimensional cube on paper.
An 8-year-old male was brought to your office 7 months ago because of frequent diarrhea and abdominal discomfort that had been present for 11⁄2–2 years. An examination revealed no abnormalities. A CBC, a serum iron level, and a metabolic panel were normal. Serology was consistent with celiac disease and a duodenal biopsy confirmed the diagnosis. The patient’s symptoms resolved after his parents took him to a nutritionist who recommended a gluten-free diet. Today you see the child for preventive care and he remains asymptomatic.
In addition to normal well child care, which one of the following would you recommend?
A) No further testing
B) Gradual reintroduction of gluten into the diet
C) IgG antigliadin antibody
D) IgA tissue transglutaminase antibody
E) A duodenal mucosal biopsy to ensure healing
ANSWER: D
Celiac disease affects approximately 1% of the U.S. population and can affect all ages. Individuals with northern European ancestry are most commonly affected. The condition is caused by autoimmunity induced by gluten-containing foods in susceptible individuals. Untreated celiac disease is associated with anemia, malabsorption, osteoporosis, weight loss, and gastrointestinal lymphomas. In children, growth stunting and delayed puberty are also common. With strict adherence to a gluten-free diet most complications from celiac disease are preventable and, in children, growth and development return to normal. The World Gastroenterology Organisation recommends annual monitoring of children and adolescents with celiac disease by anthropometry, pubertal development, and celiac serology. The preferred serology is IgA antibody to tissue transglutaminase (IgA anti-tTG) due to its balance of good performance and low cost. Failure of IgA anti-tTG titers to decrease in 6 months suggests continued ingestion of gluten. Repeat duodenoscopy with a biopsy would be unnecessary and would subject the child to potential harm.
A 13-year-old male is admitted to the hospital with diabetic ketoacidosis. Aggressive fluid resuscitation with normal saline was initiated in the emergency department and the following laboratory results were obtained:
Glucose Sodium Potassium Bicarbonate Aniongap
400mg/dL
136 mEq/L (N 136–145) 2.8 mEq/L (N 3.5–5.1) 15mEq/L(N22–29) 14mEq/L(N10–20)
In addition to continued fluid resuscitation, which one of the following would be the most appropriate next step in the management of this patient?
A) Administration of sodium bicarbonate
B) Potassium replacement
C) An intravenous insulin drip
D) Subcutaneous insulin using a basal/bolus technique
E) Bedside ketone capillary measurement
ANSWER: B
Correcting the potassium level is the best treatment choice for this patient. A low serum potassium level in diabetic ketoacidosis (DKA) indicates a significant potassium deficiency, placing the patient at risk for a cardiac arrhythmia, among other complications. Potassium deficiency is usually the product of urinary losses due to glucose osmotic diuresis and secondary hyperaldosteronism. However, serum potassium can remain normal when there is a whole body deficiency, as a result of movement of potassium out of cells in response to the acidosis, insulin deficiency, and hyperosmolality. This patient’s serum potassium is low, which indicates severe deficiency.
55
Sodium bicarbonate can be used in some cases of DKA but is not the most important next step in this case. If insulin therapy is initiated prior to potassium replacement the insulin can force more potassium into cells, worsening the serum deficiency. If the serum potassium level is <3.3 mEq/L potassium should be replaced prior to insulin therapy. Ketone measurement can also be part of DKA management but is not the most appropriate next step for this patient.
A 25-year-old male presents to your office with anxiety. During questioning he reveals that he struggles with opioid addiction since he underwent an appendectomy 2 years ago, and is anxious because of random drug screens in his workplace. He averages using about 30 mg of hydrocodone daily and goes through cycles of use and withdrawal. Screening is negative for alcohol and substance use disorders other than opioids, and for depression or other mental health disorders. You consider maintenance treatment for opioid use disorder with buprenorphine.
For this patient, buprenorphine therapy
A) is inferior to methadone maintenance therapy
B) should not be combined with naloxone therapy because of potential side effects
C) should be initiated when he is in mild to moderate withdrawal from opioids
D) can be initiated only after inpatient detoxification
E) should be continued for 1 year and then tapered
ANSWER: C
Buprenorphine therapy is an important option for maintenance therapy for patients with opioid use disorder. It can be initiated in the outpatient setting but should be done when the patient is in mild to moderate withdrawal in order to avoid the risk of precipitated withdrawal (SOR C). Buprenorphine therapy is more convenient than methadone maintenance therapy and is equally as effective. Buprenorphine/naloxone combinations are preferred over buprenorphine monotherapy due to lower abuse potential, except when naloxone is contraindicated such as in patients who are pregnant or breastfeeding (SOR C). Because relapse rates are higher in patients who discontinue medication-assisted therapy for opioid use disorder, long-term use is recommended.
A 45-year-old female presents with pain in her right hand and wrist. She is right hand–dominant and most symptoms are in the right hand and wrist. She has been awakening at night with numbness of the hand that improves after she shakes her wrist. Her work as a software engineer involves lots of typing.
On examination you note a positive Tinel’s sign at the right wrist consistent with carpal tunnel syndrome. She has an important project at work over the next several months and will not be able to take any time off.
Which one of the following treatments has the best evidence for delaying the need for definitive surgical therapy?
A) Night splints
B) Physical therapy
C) Therapeutic ultrasound
D) Oral corticosteroids
E) Corticosteroid injection
ANSWER: E
Carpal tunnel syndrome of mild to moderate severity can be treated nonsurgically. Patients with severe symptoms or nerve damage seen on electromyography should be referred for surgical therapy. Nonsurgical management options include splinting, physical therapy, therapeutic ultrasound, and corticosteroids (oral or injection). Oral prednisone, 20 mg daily, for 10–14 days improves symptoms and function compared with placebo, for up to 8 weeks, but oral corticosteroids are less effective than corticosteroid injections. In a 2013 double-blind, placebo-controlled, randomized clinical trial comparing methylprednisolone injection (40 mg and 80 mg) to saline injection, patients in the 80-mg injection group were less likely to have surgery at 12 months.
While corticosteroid injections have the best evidence for delaying the need for surgery, night splints, physical therapy, and therapeutic ultrasound have some evidence of benefit. Splinting was found to be effective in a Cochrane review. It is low cost and safe and especially recommended in pregnancy-related disease. Limited evidence supports the use of physical therapy to treat carpal tunnel syndrome. Nerve glide exercises are simple hand and finger movements that are easy to learn, can be performed at home, and can be combined with other treatments such as splinting. Therapeutic ultrasound also has limited evidence of benefit. It requires an experienced therapist and requires multiple sessions, typically 5 days/week for 2–4 weeks.
Which one of the following is an indication to consider ultrasonography to screen for developmental dysplasia of the hip in a newborn?
A) A firstborn female infant with a vertex presentation at delivery
B) A firstborn male infant with a vertex presentation at delivery
C) A large-for-gestational-age male infant with a vertex presentation at delivery
D) A male infant who was in a breech position until 28 weeks gestation
E) A female infant with a first degree relative with developmental dysplasia of the hip
ANSWER: E
Screening for developmental dysplasia of the hip (DDH) is somewhat controversial because the benefit of treatment remains somewhat unclear. Despite the widespread practice of screening for DDH, ethical newborn practices are difficult to determine. The American Academy of Family Physicians and the U.S. Preventive Services Task Force have found insufficient evidence to recommend routine screening for DDH. The American Academy of Pediatrics, however, recommends routine screening of all newborns with physical examination maneuvers, and targeted screening ultrasonography for infants who were breech in the third trimester, have a family history of DDH, or have a personal history of instability. Given this, decisions should be individualized. Additional risk factors include female sex, firstborn status, oligohydramnios, and a large-for-gestational-age infant.
Which one of the following is recommended regarding oral fluoride supplementation?
A) Supplementation for all children whose primary water source is well water
B) Starting supplementation at 6 months of age if the primary water supply is fluoride
deficient
C) Starting supplementation at 2 years of age even in children who have received topical
fluoride varnish
D) No supplementation if fluoride varnish is applied once all primary teeth have come in
or by 3 years of age, then yearly thereafter
E) No supplementation for most children
ANSWER: B
The U.S. Preventive Services Task Force recommends oral fluoride supplementation for the prevention of dental caries beginning at age 6 months for children whose primary water supply is fluoride deficient (B recommendation). Well water may be fluoridated naturally depending on the aquifer, but the water is highly variable and should be tested before deciding on the need for supplementation. Testing well water is also advisable because excessive fluoride may lead to fluorosis of the bones. Bottled water is variable, making it undependable as an adequate source of fluoride. Topical fluoride, in toothpaste or applications of fluoride varnish, is effective in preventing tooth decay in children and can be used in addition to properly fluoridated water. Twice-yearly application of fluoride varnish to primary teeth should begin when the first tooth comes in and repeated every 6 months thereafter in children (SOR B).
A 48-year-old female presents to your office for follow-up of painful rectal bleeding with bowel movements. She has increased her fiber consumption and is using a stool softener as you recommended at her last visit 2 weeks ago. She reports that her pain has worsened since yesterday and she is very uncomfortable. An examination reveals a firm and tender right posterior hemorrhoid below the dentate line.
Which one of the following would be the most appropriate next step in providing relief for this thrombosed hemorrhoid?
A) Topical corticosteroids
B) Bioflavonoid supplements
C) Oral antibiotics
D) Rubber band ligation
E) Excision
ANSWER: E
This patient has tried first-line treatment for hemorrhoids with increased fiber intake but has returned with symptoms of a thrombosed external hemorrhoid. Office-based surgical excision of the thrombosed external hemorrhoid within 2–3 days of symptom onset may provide significant symptomatic relief (SOR B) and result in a lower risk of recurrence. While conservative treatment with topical therapies such as corticosteroids may be helpful, symptomatic relief is prolonged with excision of the thrombosed hemorrhoid.
Bioflavonoids are used outside the United States for symptomatic treatment of hemorrhoids but evidence is lacking and they are not approved by the FDA for this use. Oral antibiotic therapy has no role in the treatment of thrombosed external hemorrhoids but may be beneficial in treating an abscess, which would present with a gradual onset of pain and a fluctuant rectal mass. Rubber band ligation is an appropriate treatment for grades I–III internal hemorrhoids (SOR A).
A 15-year-old male presents with a 2-day history of dark-colored urine, lower extremity edema, and fatigue. Approximately 2 weeks ago he said he had a “bad sore throat” that was treated empirically with amoxicillin. On examination his blood pressure is 144/92 mm Hg, his pulse rate is 76 beats/min, and his other vital signs are normal. Other than mild dependent edema there are no additional significant physical examination findings. A urinalysis dipstick shows 3+ hematuria.
Which one of the following findings on microscopic evaluation of the urine sediment would help to confirm the diagnosis in this patient?
A) Gram-positive cocci in chains
B) RBC casts
C) WBC casts
D) Eosinophils
E) Oxalate crystals
ANSWER: B
This is a classic presentation for acute poststreptococcal glomerulonephritis (APSGN), with the onset of gross hematuria associated with hypertension and systemic edema. This is most commonly seen in school-age children, usually 1–2 weeks after an episode of pharyngitis or 3–4 weeks after an episode of impetigo, caused by so-called nephritogenic strains of Group A -hemolytic Streptococcus. The hematuria is caused by immune complex–mediated glomerular injury.
66
Bacteriuria may be seen in both upper and lower urinary tract infections, but may also be a spurious finding, especially with the combined presence of epithelial cells. The classic finding on microscopic urinalysis for acute glomerulonephritis is the presence of RBC casts. WBC casts are seen with acute pyelonephritis. The presence of urinary eosinophils indicates acute interstitial nephritis. Calcium oxalate makes up the most common type of kidney stones.
Antibiotics prescribed for antecedent pharyngitis do not prevent APSGN. Treatment is supportive, controlling blood pressure and edema with a thiazide or a loop diuretic. The prognosis for resolution and full recovery of the vast majority of patients with APSGN is excellent, especially in the pediatric age group.
You have diagnosed a Chlamydia infection in a 24-year-old male. He reports having three sexual partners in the last month. He does not feel comfortable revealing their names or genders at this time.
In almost all U.S. states, which one of the following would be the most appropriate plan for timely treatment of the involved parties with azithromycin (Zithromax) as a single dose?
A) One prescription in the patient’s name
B) One prescription in the patient’s name and instructions to the patient to inform his
partners
C) One prescription in the patient’s name with three refills
D) One prescription in the patient’s name and three prescriptions for expedited partner
therapy
E) One prescription in the patient’s name and three blank prescriptions
ANSWER: D
Expedited partner therapy (EPT) is the clinical practice of treating the sex partners of patients diagnosed with Chlamydia or gonorrhea by providing prescriptions or medications to the patient to take to his/her partner without the health care provider first examining the partner. When patients have been diagnosed with gonorrhea or Chlamydia, EPT has been shown to reduce the overall burden of disease in a given population when the partners cannot be linked to care. In this case, as the partners’ identities are unknown, it is impossible for the clinician to examine them or even contact them, so four prescriptions should be written, one with the patient’s name and the other three for EPT. The CDC states that EPT is particularly effective in treating the female partners of infected males. The CDC considers having the partners visit a health care provider to be the optimal course of treatment but this is not often practically feasible due to a lack of resources or social factors.
Although conventional practice is to treat only the patient, this does not provide timely treatment for the patient’s partners. A prescription written to “EPT” can be filled at the pharmacy without the individual’s name or date of birth. Kentucky and South Carolina are the only states that do not allow this practice. A prescription with three refills would be unethical, as presumably the physician would be advising the patient to distribute medications that had been prescribed to only the patient. Blank prescriptions would require the partners to reveal their identities, which may lead to a reluctance to fill the prescriptions.
A 24-year-old male presents with swelling of the right testicle. The pain started yesterday and has persistently worsened. A physical examination reveals swelling on the right side of his scrotum. His entire testicle is painful, and elevation of the scrotum improves his pain. Ultrasonography reveals hyperemia, swelling, and increased blood flow to the testis and epididymis.
Which one of the following would be the most appropriate management?
A) Doxycycline
B) Levofloxacin (Levaquin)
C) Trimethoprim/sulfamethoxazole (Bactrim)
D) Doxycycline plus ceftriaxone
E) Urgent surgical evaluation
ANSWER: D
This patient presents with acute epididymitis. Typical symptoms develop gradually over 1–2 days with posterior scrotal pain and swelling. Additional symptoms may include fever, hematuria, dysuria, and urinary frequency. The pain may radiate to the lower abdomen. Physical examination findings may include tenderness of the epididymis and testis along with swelling of the scrotum. Elevation of the scrotum may decrease the pain (Prehn sign). Typical ultrasound findings include hyperemia, swelling, and increased blood flow to the epididymis.
With testicular torsion the pain is often sudden in onset and severe, with associated nausea and vomiting and no other urologic symptoms. A physical examination often demonstrates a high-riding testis that may lie transversely in the scrotum. The cremasteric reflex may be absent. Ultrasound findings would demonstrate decreased or absent blood flow with testicular torsion.
In sexually active adults <35 years of age, gonorrhea and Chlamydia are the most common causative organisms of acute epididymitis. Ceftriaxone, 250 mg intramuscularly or intravenously once, with oral doxycycline, 100 mg twice daily for 10 days, would be the appropriate treatment for acute epididymitis (SOR C). In men over the age of 35 or those with a history of recent urinary tract surgery or instrumentation, enteric organisms are the most likely cause and monotherapy with oral levofloxacin or ofloxacin for 10 days would be the recommended treatment.
A 70-year-old female comes to your office with a 10-day history of a subjective fever at home, facial and tooth pain, sinus pressure, and a green nasal discharge. There has been no change in her symptoms. The patient has a history of allergic rhinitis treated with immunotherapy, and a history of developing hives while taking penicillin. On examination her temperature is 38.1°C (100.6°F).
In addition to nasal saline and analgesics, which one of the following would be the most appropriate management?
A) Standard-dose amoxicillin/clavulanate (Augmentin)
B) High-dose amoxicillin/clavulanate
C) Azithromycin (Zithromax)
D) Doxycycline
ANSWER: D
This patient has a history of symptoms consistent with acute bacterial rhinosinusitis that have persisted for 10 days, warranting empiric antibiotic therapy (SOR B). Doxycycline is an appropriate alternative to amoxicillin/clavulanate for a patient with a history of a reaction to penicillin. Macrolides and trimethoprim/sulfamethoxazole are not recommended as empiric therapy because of high rates of resistance.
A 24-year-old patient wants to start the process of transitioning from female to male. He has been working with a psychiatrist who has confirmed the diagnosis of gender dysphoria.
Which one of the following would be the best initial treatment for this patient?
A) Clomiphene
B) Letrozole (Femara) C) Leuprolide (Eligard)
D) Spironolactone (Aldactone) E) Testosterone
ANSWER: E
For patients with gender dysphoria or gender incongruence who desire hormone treatment, the treatment goal is to suppress endogenous sex hormone production and maintain sex hormone levels in the normal range for their affirmed gender. For a female-to-male transgender patient this is most easily accomplished with testosterone. When testosterone levels are maintained in the normal genetic male range, gonadotropins and ovarian hormone production is suppressed, which accomplishes both goals for hormonal treatment without the need for additional gonadotropin suppression from medications such as leuprolide.
Clomiphene can increase serum testosterone levels, but only in the presence of a functioning testicle. Letrozole is an estrogen receptor antagonist, but it would not increase serum testosterone levels. Spironolactone has androgen receptor blocking effects and would not accomplish either of the hormone treatment goals.
Based on American Cancer Society guidelines for cervical cancer screening, when should HPV DNA co-testing first be performed along with Papanicolaou testing?
A) At the onset of sexual activity
B) At age 21
C) At age 25
D) At age 30
E) At age 35
ANSWER: D
According to American Cancer Society guidelines for cervical cancer screening, Papanicolaou (Pap) testing should begin at age 21 irrespective of sexual activity and should be continued every 3 years until age 29. The preferred screening strategy beginning at age 30 is Pap testing with HPV co-testing, which should be continued every 5 years until age 65. Cervical screening may be discontinued at that time if the patient’s last two tests have been negative and the patient was tested within the previous 5 years.
A 24-year old female presents to your office with a 3-month history of difficulty sleeping. She says that she struggles to fall asleep and wakes up multiple times at night at least three times a week. She tries to go to bed at 10:00 p.m. and wakes up at 6:30 a.m. to start her day. She lies awake for an hour in bed before falling asleep and spends up to 2 hours awake in the middle of the night trying to fall back asleep. Lately she has been feeling fatigued and having difficulty concentrating at work. You conduct a full history and physical examination and tell her to return in 2 weeks with a sleep diary. At this follow-up visit you see from her diary that she is sleeping an average of 51⁄2 hours per night.
Which one of the following would be the most appropriate recommendation?
A) Set her alarm for 5:30 a.m.
B) Add a mid-afternoon nap
C) Move her bedtime to 9:00 p.m.
D) Move her bedtime to 12:30 a.m.
E) Stay up for an hour if she wakes up at 3:00 a.m.
ANSWER: D
This patient presents with symptoms of chronic insomnia. Cognitive-behavioral therapy for insomnia (CBT-I) and brief behavioral therapy for insomnia (BBT-I) are effective nonpharmacologic treatments for chronic insomnia. Modified CBT-I and BBT-I can be administered by a primary care physician. The basic principles include stimulus control (sleep hygiene) and sleep restriction. Reducing time in bed increases sleep efficiency. In this case, 6 hours of time in bed would improve the patient’s sleep efficiency and a bedtime of 12:30 a.m. would accomplish this goal. Generally, reduced time in bed is accomplished by postponing bedtime rather than getting up earlier. Naps generally do not improve sleep efficiency. While getting out of bed is recommended after being in bed for 30 minutes without falling asleep, or being awake for 30 minutes after being asleep, staying up for a prescribed period of time is not recommended.
A 32-year-old female who is one of your longtime patients calls you because of a 24-hour history of painful urination with urinary frequency and urgency. She is otherwise healthy and does not have any fever, chills, back pain, or vaginal discharge. She uses an oral contraceptive pill and states that her last menstrual period was normal and occurred last week.
Which one of the following would be most appropriate at this time?
A) Empiric antibiotic treatment
B) A urinalysis
C) A urine culture
D) Plain abdominal radiographs
E) Pelvic ultrasonography
ANSWER: A
This patient has symptoms of acute simple cystitis and does not have any symptoms that would suggest a complicated urinary tract infection or vaginal infection. In these cases treatment with oral antibiotic therapy may be prescribed without further evaluation (SOR B). Simple cystitis is a clinical diagnosis and a urinalysis and urine culture are not necessary. The patient does not have any symptoms that warrant evaluation with abdominal radiographs or pelvic ultrasonography.
A 70-year-old female develops thrombocytopenia during a prolonged hospitalization for endocarditis. Her current medications include scheduled unfractionated heparin injections for venous thromboembolism prophylaxis. You suspect heparin-induced thrombocytopenia (HIT).
Assuming that her thrombocytopenia is caused by HIT, which one of the following is the most likely complication?
A) Anaphylaxis
B) Disseminated intravascular coagulation
C) Hemorrhage
D) Sepsis
E) Thrombosis
ANSWER: E
Heparin-induced thrombocytopenia (HIT) is an immune-mediated process that occurs in approximately 1 in 5000 hospitalized patients. Patients are at highest risk 7–10 days after exposure to unfractionated heparin, and the risk is particularly high after cardiac surgery, which is associated with an estimated rate of 1%–3%. In contrast to other causes of thrombocytopenia, HIT places patients at a paradoxically increased risk of thrombotic complications, with clotting events occurring in roughly 50% of confirmed cases of HIT. Lower-extremity deep vein thrombosis and pulmonary embolism are the most common thrombotic complications, followed by arterial thromboses, stroke, and myocardial infarction, in descending order of frequency. Thromboses often occur concurrently with the development of thrombocytopenia or shortly thereafter. The risk of HIT can be determined with the 4T scoring system, which evaluates the acuity of thrombocytopenia, timing of onset, presence of thrombosis, and alternative causes of thrombocytopenia. Patients with an intermediate or high pretest probability should be managed with prompt discontinuation of heparin and initiation of full-dose anticoagulation with a non-heparin anticoagulant, such as argatroban, danaparoid, fondaparinux, or bivalirudin, pending results of further HIT diagnostic evaluation. Anaphylaxis, disseminated intravascular coagulation, hemorrhage, and sepsis are all less common complications of HIT compared to thrombotic events.
A 57-year-old male with a history of heart failure sees you for follow-up. He describes symptoms of mild dyspnea on exertion with ordinary activities such as shopping or yard work. An echocardiogram shows an ejection fraction of 37%.
According to the New York Heart Association criteria, this patient’s heart failure would be classified as which one of the following?
A) Class I
B) Class II
C) Class III
D) Class IV
ANSWER: B
The appropriate classification of heart failure is important for monitoring the disease. The most common currently used system is the New York Heart Association (NYHA) functional classification. In this system, class I is defined as heart disease in a patient with no symptoms and no limitations of physical activity. Patients with class II heart failure have mild symptoms with normal physical activity. Class III heart failure refers to significant limitations of activity, including symptoms with less than normal activities. Patients with class IV heart failure have symptoms at rest and are unable to carry on activity without discomfort.
A 24-year-old female presents with progressively worsening vulvar pain for 3 days. On examination a 3×3-cm tender, fluctuant mass is noted on the right labia minora. She had a similar episode of this problem last year.
Which one of the following would be the most appropriate management?
A) Expectant management
B) Fine-needle aspiration
C) Incision and drainage
D) Marsupialization
E) Excision under general anesthesia
ANSWER: D
The most appropriate management of a recurrent Bartholin gland abscess would be marsupialization, which has a 0% recurrence rate at 6 months. Local anesthesia can be used in the office to effectively treat Bartholin gland abscesses and sedation is not required (SOR A). If the Bartholin gland abscess is >5 cm, referral to a gynecologist is recommended. Expectant management, fine-needle aspiration, or incision and drainage would likely lead to recurrence.
A 25-year-old male presents with a 4-month history of crampy abdominal pain, diarrhea, and fatigue. His symptoms began gradually but have become more severe and he is now experiencing rectal bleeding. He says that his abdominal pain seems to temporarily improve after eating. He has smoked five cigarettes per day for the past 8 years. He is surprised to learn that he has lost 7 kg (15 lb) when he is weighed today.
His vital signs include a blood pressure of 116/70 mm Hg, a heart rate of 76 beats/min, a respiratory rate of 12/min, and a temperature of 37.7°C (99.9°F). A physical examination reveals abdominal tenderness and mild distention. An anorectal examination is significant for a perianal fistula. A laboratory evaluation is notable for mild anemia. His kidney and liver function are normal.
Which one of the following is the most likely diagnosis?
A) Celiac disease
B) Chronic pancreatitis
C) Crohn’s disease
D) Irritable bowel syndrome
E) Ulcerative colitis
ANSWER: C
Crohn’s disease may present insidiously with diarrhea, abdominal pain, rectal bleeding, fever, weight loss, and fatigue. Red-flag symptoms include perianal lesions, a first degree relative with inflammatory bowel disease, weight loss of 5% of the patient’s usual weight, abdominal pain for more than 3 months, nocturnal diarrhea, fever, the absence of abdominal pain for 30–45 minutes after eating, and the absence of rectal urgency. This patient exhibits symptoms consistent with Crohn’s disease. While anemia is also common in celiac disease, rectal bleeding is not. Chronic pancreatitis does not generally present with improved pain after eating. Irritable bowel syndrome is not associated with fever, rectal bleeding, anemia, or perianal fistulas. Ulcerative colitis is not associated with perianal lesions.
A 34-year-old female at 32 weeks gestation presents with a right-sided, pounding headache that began 8 hours ago and is similar to headaches she has had in the past. She is sensitive to light and sound, and has vomited several times since the onset of pain. She has taken acetaminophen without relief. She takes prenatal vitamins but no other routine medications. On examination her blood pressure is normal.
Which one of the following would be the most appropriate treatment for this patient?
A) Dihydroergotamine
B) Metoclopramide (Reglan)
C) Naproxen
D) Oxycodone (OxyContin)
E) Sumatriptan (Imitrex)
ANSWER: B
Metoclopramide and acetaminophen are the only two medications considered safe for abortive migraine treatment during pregnancy (SOR B). The dopamine antagonist antiemetics are considered second-line abortive treatments in the general population. Dihydroergotamine should not be used during pregnancy due to its oxytocic properties and the potential risk of intrauterine growth restriction with its use. NSAIDs are not considered safe during pregnancy, particularly in the first and third trimesters. Opioids are only moderately useful for migraine treatment and should be avoided during pregnancy due to their abuse potential. Triptans are generally considered safe during the first trimester but not in the second and third trimesters. Their use has been associated with uterine atony, increased risk of bleeding during delivery, and increased risk of preterm birth.
Which one of the following U-100 insulin products has the longest duration of action?
A) Degludec (Tresiba)
B) Glargine (Lantus)
C) Isophane NPH (Humulin N)
D) Lispro (Humalog)
E) Regular (Humulin R)
ANSWER: A
Among the available U-100 insulin products, the one with the longest duration of action is ultralong-acting degludec, which lasts 42 hours. The duration of action of rapid-acting lispro is 3–6.5 hours, short-acting regular is 5–8 hours, intermediate-acting isophane is 12–16 hours, and long-acting glargine is 11–24 hours.
When performing a geriatric assessment, which one of the following is an instrumental activity of daily living?
A) Bathing
B) Dressing
C) Transferring between the bed and a chair
D) Using the telephone
E) Using the toilet
ANSWER: D
The foundation of geriatric assessment is assessing the individual’s ability to perform tasks required for living. Activities of daily living are self-care activities that are performed daily, such as eating, bathing, dressing, transferring between the bed and a chair, and toileting, including bladder and bowel function. Instrumental activities of daily living include activities necessary to live independently, such as using a telephone, doing housework, preparing meals, taking medications properly, and managing finances.
A 9-year-old male with a history of moderate persistent asthma is brought to the emergency department with an acute exacerbation. His symptoms began with a runny nose and nasal congestion 2 days ago. His parents state that he has not had any fevers or chills and he was eating and drinking well until a few hours ago when his breathing started to appear more labored. After multiple treatments with inhaled albuterol (Proventil, Ventolin) and oral prednisolone he remains tachypneic and wheezy.
Which one of the following intravenous medications should be added to the patient’s current treatment to reduce the likelihood of hospital admission?
A) Ketorolac
B) Magnesium sulfate
C) Methylprednisolone
D) Omalizumab (Xolair)
E) Theophylline
ANSWER: B
Children who present to the emergency department with an asthma exacerbation and fail to improve adequately with inhaled short-acting bronchodilators and corticosteroids may benefit from treatment with intravenous (IV) magnesium sulfate. A 2016 Cochrane review of three randomized, controlled trials found that this reduced hospital admissions by 68%. Ketorolac is not known to have any benefit in the treatment of asthma. Oral administration of corticosteroids is as effective as IV administration, so there is no reason to give IV methylprednisolone. Omalizumab may be used to prevent exacerbations in patients with severe asthma who do not achieve adequate control with high-dose inhaled corticosteroids, but it has no role in the management of acute exacerbations. IV theophylline is not recommended for asthma exacerbations given its safety profile and poor efficacy compared to short-acting bronchodilators.
A 62-year-old female sees you for a routine health maintenance examination. She has a history of breast cancer diagnosed 6 years ago that was treated with lumpectomy, radiation, and endocrine therapy. She is feeling well today and has no symptoms of concern. There is no family history of breast, ovarian, colon, or prostate cancers.
In addition to mammography, which one of the following annual tests would improve this patient’s chance of survival?
A) No tests
B) CT of the chest
C) MRI of the breast
D) Breast ultrasonography
E) A bone scan
ANSWER: A
Breast cancer, the most common noncutaneous malignancy among women, has a 5-year survival rate of almost 90%, so medical care of such patients is increasingly common. To help provide guidance to primary care physicians, the American Cancer Society and the American Society of Clinical Oncology published their joint Breast Cancer Survivorship Care Guideline in 2016. This guideline includes a recommendation for annual mammography for women with prior treatment for breast cancer to screen for local recurrence or a new primary breast cancer. MRI is not recommended in the absence of specific high-risk criteria such as a BRCA mutation. Similarly, other imaging modalities such as ultrasonography are not recommended in the absence of symptoms. Imaging is not indicated to screen for metastatic disease. Though breast cancer most commonly metastasizes to the lung, bone, and liver, there is no evidence that screening CT or a bone scan provides mortality or quality-of-life benefits.
A 50-year-old male with no significant past medical history presents with a 5-day history of fever to 101°F, chills, and mild diffuse joint and muscle pains. He also reports a mild headache but has not had any sore throat, rhinorrhea, cough, shortness of breath, nausea, vomiting, or change in bowel habits. He noticed a round red rash (shown below) a few days ago on his leg that has grown in size since then. It is minimally pruritic but not painful. He has no other rashes. He recently traveled to Vermont for his family’s annual fall hiking trip but does not recall any insect bites. He does not take any medications and has no drug allergies. He has a blood pressure of 120/74 mm Hg, a pulse rate of 84 beats/min, and a temperature of 37.8°C (100.0°F). Cardiac, pulmonary, musculoskeletal, and abdominal examinations are normal.
Which one of the following is the most likely diagnosis?
A) Adenovirus
B) Ehrlichiosis
C) Influenza
D) Lyme disease
E) Rocky Mountain spotted fever
ANSWER: D
This patient has a classic presentation for Lyme disease. Lyme disease is transmitted by a tick bite, but not all patients remember being bitten by a tick. The classic erythema migrans lesion appears a couple of weeks after the tick bite. The first-line treatment for Lyme disease is either amoxicillin or doxycycline. Macrolides can be used if patients have true allergies to -lactams and doxycycline, but they are less effective. This patient is not exhibiting the respiratory symptoms typically associated with adenovirus. Ehrlichiosis and Rocky Mountain spotted fever typically present with headaches and fevers but not with an erythema migrans rash. Although influenza should be considered in the differential diagnosis, it would not present with an erythema migrans rash.
Which one of the following oral iron preparations is most effective for the treatment of iron deficiency anemia in a patient with non–dialysis-dependent chronic kidney disease?
A) Ferric citrate (Auryxia)
B) Ferrous fumarate
C) Ferrous gluconate
D) Ferrous sulfate
ANSWER: A
Oral iron supplements, including ferrous fumarate, ferrous gluconate, and ferrous sulfate, are generally ineffective when used by hemodialysis patients and are only modestly effective when used by patients with non–dialysis-dependent chronic kidney disease. The one exception is ferric citrate, which is highly efficacious in all patients with chronic kidney disease. Intravenous iron preparations such as iron dextran are similarly effective in both groups and are considered the gold standard for the treatment of iron deficiency in patients on chronic hemodialysis.
One of your patients will turn 65 in 2 weeks and your practice manager routinely encourages scheduling a Welcome to Medicare preventive visit soon after patients’ 65th birthdays. This patient continues to work full time and is currently insured through his employer-sponsored health insurance.
In order to bill for a Welcome to Medicare visit after this patient turns 65, which one of the following is true?
A) The patient must be enrolled in Medicare Part A
B) The patient must be enrolled in Medicare Part B
C) The patient must be enrolled in Medicare Part D
D) The patient must be over age 65 and the specific type of Medicare enrollment is not relevant
ANSWER: B
The Welcome to Medicare preventive visit, also known as an Initial Preventive Physical Examination (IPPE), is a one-time service that can be provided within the first year of a patient’s enrollment in Medicare Part B. Medicare Part B covers provider visits and outpatient services such as laboratory testing. Beneficiaries are automatically enrolled in Part A when they apply to Medicare, which provides coverage for hospital-based and hospice care. Because Part A does not typically carry a monthly premium, some working older adults who continue to have insurance through their employer may opt to obtain Part A only, and wait on Part B coverage, which does have a monthly premium. Medicare Part D is prescription drug coverage.
Routine follow-up blood tests for colorectal cancer survivors should include
A) carcinoembryonic antigen (CEA) levels only
B) liver function tests only
C) CBCs and CEA levels only
D) CBCs and liver function tests only
E) CBCs, CEA levels, and liver function tests
ANSWER: A
The Choosing Wisely campaign recommends checking only carcinoembryonic antigen (CEA) levels following curative treatment for colorectal cancer (SOR C). No routine laboratory studies such as a CBC or liver function tests should be ordered for follow-up.
You are advising a group of medical students who are planning a tobacco cessation program for expectant mothers. The medical students want to build an advertising program that touts the pregnancy benefits of tobacco cessation.
You tell the medical students that evidence suggests that tobacco cessation in pregnant women
A) decreases the risk for cesarean delivery
B) decreases the risk for preeclampsia
C) decreases the need for epidural anesthesia
D) increases infant birth weight
E) increases the risk for preterm delivery
ANSWER: D
Smoking during pregnancy increases the risk for fetal growth restriction. The U.S. Preventive Services Task Force identified evidence that tobacco cessation increases infant birth weight and decreases the risk for preterm delivery. There is no evidence that tobacco cessation in pregnant women decreases the risk for cesarean delivery, the risk for preeclampsia, or the need for epidural anesthesia.
A 13-year-old female is brought to your office by her adoptive mother. They do not know the patient’s biological family history. They are concerned because, unlike all of her friends, she has not yet started to menstruate. Breast development began 2 years ago. On examination her breasts show a secondary mound from the areola and nipple above the contour of her breast. She has dark, coarse hair covering the mons pubis consistent with a stage 4 sexual maturity rating.
If her sexual development continues to be normal, at what age should you recommend evaluation for primary amenorrhea?
A) 13 B) 14 C) 15 D) 16 E) 17
ANSWER: C
Delayed puberty in girls is defined as the absence of breast development by age 13. Typically, menarche starts 2.5 years after the onset of breast development, with an average age of 12.5 years (normal range 9–15 years). In girls with otherwise normal sexual development, the absence of menarche by 15 years of age should prompt an evaluation for primary amenorrhea.
A 57-year-old female with a history of diabetes mellitus, hypertension, and depression sees you for a routine follow-up visit. Her vital signs include a heart rate of 88 beats/min, a blood pressure of 162/84 mm Hg, and a BMI of 32 kg/m2.
The recommended antihypertensive regimen for reducing cardiovascular events in this patient is an ACE inhibitor plus
A) an -blocker
B) an angiotensin receptor blocker
C) a calcium channel blocker
D) a loop diuretic
ANSWER: C
The ACCOMPLISH trial demonstrated that an ACE inhibitor (ACEI) in combination with a calcium channel blocker (CCB) reduced both fatal and nonfatal cardiovascular events in patients with diabetes mellitus and hypertension. The benefit of an ACEI and a CCB for reducing cardiovascular events was greater than that of an ACEI and a thiazide diuretic. Evidence has shown that combination therapy for most patients should include a CCB, an ACEI or angiotensin receptor blocker (ARB), or a thiazide diuretic (SOR A). The American College of Cardiology/American Heart Association guidelines recommend against centrally acting medications such as -blockers for first-line therapy. Combining ACEIs and ARBs is not recommended, as the risk of side effects such as hyperkalemia outweighs the benefits. Loop diuretics are not considered first-line antihypertensive agents.
An elderly homeless male is brought to the emergency department. He is clearly hypothermic due to cold exposure and has superficial frostbite of his extremities. He is still conscious and shivering.
In addition to rewarming him, which one of the following should you administer?
A) Acetazolamide (Diamox Sequels)
B) Amitriptyline
C) Ceftriaxone
D) Ibuprofen
E) tPA
ANSWER: D
Frostbite is a freezing injury that occurs when initial cooling causes vasoconstriction and localized ischemia. Continued cold exposure leads to ice crystal formation, which causes cellular lysis, electrolyte abnormalities, and microvascular occlusion. Rewarming creates an inflammatory response. Ibuprofen is the most appropriate agent for the treatment of frostbite until the wounds heal or surgery is performed (SOR C). Acetazolamide can cause frostbite at high altitudes. Amitriptyline is used to treat the pain of immersion foot (also called trench foot), which is a nonfreezing injury that happens when the foot is exposed to prolonged wet conditions above 0°C (32°F). Antibiotics are indicated if open or dirty wounds are present (SOR B). tPA has a role in treating patients with frostbite, but it is used only to decrease the risk of amputation when rewarming patients with grade 3, grade 4, or deep frostbite (SOR B).
A 72-year-old female presents with bothersome palpitations. She is otherwise healthy and is not taking any medications. A physical examination is normal, including thyroid and eye examinations. Laboratory studies reveal a serum TSH level of 0.2 U/mL (N 0.4–4.0) and normal T3 and free T4 levels. An EKG reveals frequent premature atrial contractions but is otherwise normal. Ultrasonography of the thyroid does not reveal any nodules, thyroid scintigraphy shows diffuse uptake, and an anti–thyrotropin-receptor (thyroid-stimulating immunoglobulin) antibody level is significantly elevated.
Which one of the following is the most likely diagnosis?
A) Central hypothyroidism
B) Graves disease
C) Iodine deficiency
D) Solitary toxic thyroid nodule
E) Toxic multinodular goiter
ANSWER: B
This patient has subclinical hyperthyroidism caused by Graves disease. A positive anti–thyrotropin-receptor (thyroid-stimulating immunoglobulin) antibody result is virtually diagnostic of Graves disease. Central hypothyroidism is associated with a low TSH level and low T3 and T4 levels. Iodine deficiency is associated with goiter and hypothyroidism. Nodular thyroid disease is unlikely given the imaging results. Treatment of this patient’s mild Graves disease is probably indicated, given her age and cardiac symptoms.
An EKG performed on a patient with a history of heart failure and an ejection fraction of 30% reveals sinus rhythm with a heart rate of 55 beats/min and new left bundle branch block with a QRS interval of 160 msec. The patient is taking a -blocker but has a history of acetylcholinesterase inhibitor–induced angioedema.
Which one of the following would be most appropriate at this point?
A) Amiodarone
B) Ivabradine (Corlanor)
C) Sacubitril/valsartan (Entresto)
D) Sinoatrial node ablation
E) Cardiac resynchronization therapy
ANSWER: E
Cardiac resynchronization therapy is strongly recommended for patients with symptomatic heart failure, an ejection fraction <35%, and a left bundle branch block with a QRS interval >150 msec. Amiodarone is an antiarrhythmic and would not be indicated for this patient. Ivabradine, a sinoatrial node modulator, is used in patients with symptomatic heart failure as an add-on therapy to decrease the heart rate. It is indicated in patients with a heart rate >70 beats/min despite -blockade. Sacubitril/valsartan is contraindicated in patients with a history of angioedema. Sinoatrial node ablation is indicated for some patients with sinus node dysfunction.
A 66-year-old female with multiple medical problems has routine laboratory work performed during a regularly scheduled clinic visit. All of the laboratory values are normal except for a serum calcium level of 11.0 mg/dL (N 8.5–10.2).
Which one of the following medications in her current regimen is most likely to cause an elevated calcium level?
A) Alendronate (Fosamax)
B) Lithium
C) Omeprazole (Prilosec)
D) Sertraline (Zoloft)
E) Spironolactone (Aldactone)
ANSWER: B
Hypercalcemia is a commonly encountered laboratory abnormality. It is important for family physicians to be aware of common medications that can cause elevated calcium levels. Of the options listed, lithium is the only medication that can cause high calcium levels. In addition, thiazide diuretics, excluding aldosterone receptor antagonists such as spironolactone, often cause elevated calcium levels. Hypercalcemia is not a side effect of alendronate, omeprazole, sertraline, or spironolactone.
A 58-year-old athletic trainer presents with acute low back pain. He thinks the pain began after he started a new weightlifting regimen 3 weeks ago. The pain does not radiate, and prolonged standing exacerbates the pain. He does not have any bowel or bladder incontinence, fever, or saddle anesthesia. He asks which type of therapy is most likely to shorten the course of the pain.
Which one of the following would be the best recommendation?
A) Ice
B) Targeted exercises
C) Oral analgesics
D) Spinal manipulation
E) Evidence does not support the superiority of any modality
ANSWER: E
Recommendations for the treatment of back pain often include ice, targeted exercises, oral analgesics, and spinal manipulation. Although there is some evidence that certain modalities are better than placebo, there is very little evidence to show that one modality is superior to another in relieving back pain or shortening the course of the pain. There is some evidence that spinal manipulation is superior to sham manipulation.
A 23-year-old male presents for evaluation of wrist pain after he fell while skateboarding. On examination he has left wrist pain with snuffbox tenderness. A radiograph of his wrist shows a fracture through the midportion of the scaphoid with 2 mm of displacement.
Which one of the following would be the most appropriate treatment of this injury?
A) A thumb spica cast
B) A short arm cast
C) A long arm cast
D) Closed reduction followed by a thumb spica cast
E) Surgical fixation
ANSWER: E
Scaphoid fractures have a high risk for nonunion because the blood supply arises distally from branches of the radial artery. The proximal pole of the scaphoid is entirely dependent on this distal blood supply. To improve healing and decrease the risk of nonunion and avascular necrosis, displaced fractures should be treated with surgical fixation. Nondisplaced fractures of the distal third of the scaphoid may be treated with a short arm thumb spica cast for 4–6 weeks. Middle and proximal fractures should be treated with a long arm thumb spica cast for 6 weeks, followed by a short arm thumb spica cast.
A 21-year-old female comes to your office because of concerns about excess hair growth. She has dark hairs on her upper lip and chin and around her nipples that have been consistently present since puberty, and she is dissatisfied with the cosmetic results of various hair removal methods. She has regular menses and does not wish to become pregnant at this time. An examination is consistent with some terminal hairs in the distribution she describes and is otherwise unremarkable.
Which one of the following would be most appropriate at this point?
A) An early morning total testosterone level
B) A full hormonal workup including thyroid function testing, prolactin,
17-hydroxyprogesterone, and dehydroepiandrosterone sulfate (DHEAS) levels
C) Transvaginal ultrasonography
D) A 6-month trial of oral contraceptive pills
ANSWER: D
This patient most likely has benign idiopathic hirsutism. It is estimated that approximately 50% of women with mild hirsutism have idiopathic hirsutism. In the absence of other worrisome findings on the history or examination, such as a rapid onset, virilization, or a high degree of hirsutism, the most appropriate next step is a trial of pharmacologic therapy, using oral contraceptive pills as the first-line agent if the patient does not desire pregnancy. A minimum 6-month trial is needed because of the length of the hair growth cycle. An early morning total testosterone level, a full hormonal workup, and transvaginal ultrasonography would be appropriate if there were other signs and symptoms of hyperandrogenism on the history or examination.
You are working in an urgent care clinic when a 68-year-old male with chronic hypertension sees you for refills of his medications. He has been out of his medication for the past month and could not get an appointment with his primary care physician for refills. His blood pressure is persistently 190/115 mm Hg, even after he has rested in a quiet room for 30 minutes. His blood pressure previously had been well controlled. He has a moderate headache but otherwise feels well. An examination, including a funduscopic examination, is normal.
Which one of the following management options would be most appropriate at this time?
A) Refill his usual medications and arrange for follow-up in 1 week
B) Administer oral labetalol (Trandate) every 30 minutes until his blood pressure is
<180/110 mm Hg
C) Administer oral nifedipine (Procardia) every 30 minutes until his blood pressure is
<180/110 mm Hg
D) Administer sublingual nifedipine every 30 minutes until his blood pressure is <180/110
mm Hg
E) Refer for immediate hospitalization for intravenous antihypertensive treatment
ANSWER: B
This patient has a hypertensive urgency, defined as symptomatic acute severe hypertension without evidence of acute end-organ injury. Hypertensive urgencies may be managed in the ambulatory setting. Emergent intravenous treatment at the hospital is not indicated. This patient should be treated with an oral agent with a fairly rapid onset of action, such as clonidine, labetalol, captopril, or prazosin. Topical nitroglycerin is also an option. Nifedipine may cause unpredictable blood pressure reduction and should be avoided. The patient may be discharged to resume his usual medications after his symptoms have improved and his blood pressure is below 160–180/110 mm Hg, with follow-up within a week.
The U.S. Preventive Services Task Force recommends which one of the following for breast cancer screening?
A) Annual screening mammography for women ages 40–75
B) Annual screening mammography for women ages 50–75
C) Biennial screening mammography for women ages 35–75
D) Biennial screening mammography for women ages 50–75
E) Biennial screening mammography for women ages 45–50 and annual screening
mammography for women ages 51–75
ANSWER: D
Current U.S. Preventive Services Task Force guidelines for breast cancer screening recommend biennial screening mammography for women ages 50–75 (B recommendation). Biennial screening mammography can be considered for women age 40–49 after discussing the risks and benefits with the patient (C recommendation).
At a routine visit, a 65-year-old male former smoker reports shortness of breath and a cough that has been worsening slowly over the last 6 months. On examination you hear bibasilar inspiratory crackles. An EKG, chest radiograph, and echocardiogram are normal. CT of the chest shows multiple bilateral patchy areas of consolidation.
The most appropriate next step is to
A) take a detailed history of medication use and lifetime environmental exposures
B) start an antibiotic
C) start furosemide (Lasix)
D) start an inhaled short-acting -agonist as needed
E) refer for pulmonary rehabilitation
ANSWER: A
Idiopathic pulmonary fibrosis occurs most often in male former smokers over the age of 60. For patients with newly diagnosed interstitial lung disease (ILD) with suspected idiopathic pulmonary fibrosis, the American Thoracic Society recommends taking a detailed history of medication use and environmental exposures over the patient’s lifetime. In an observational study of 1084 patients, 47% were identified as having hypersensitivity pneumonitis on a detailed assessment of new-onset ILD with an unknown cause. Laboratory testing for connective tissue disease is also recommended. Antibiotics would be appropriate to treat a bacterial infection. Furosemide is used to treat heart failure. An inhaled short-acting -agonist and pulmonary rehabilitation would not be appropriate at this time.
In a patient with documented Helicobacter pylori infection, which one of the following studies should be performed to document clearance of the infection?
A) An H. pylori IgG level
B) An H. pylori IgM level
C) A stool antigen test 1 week after the completion of antibiotic therapy
D) A urea breath test 1 month after the completion of antibiotic therapy
E) A gastric biopsy
ANSWER: D
After treatment for a Helicobacter pylori infection it is essential to document clearance of the infection. This is typically done with a stool antigen test or a urea breath test performed 1 month after the completion of antibiotic therapy. If the patient is taking a proton pump inhibitor it should be discontinued prior to the test. H. pylori IgG or IgM levels and gastric biopsies are not appropriate for documenting clearance of H. pylori infection.
A 26-year-old female with a history of bipolar disorder sees you for follow-up of chronic joint pain. The review of systems is positive for intermittent rashes on the dorsal hands and face and left-sided pleuritic chest pain. An examination reveals tender swelling of the hand and wrist joints bilaterally. Examination of the lungs reveals diminished breath sounds at the left base with point-of-care ultrasonography findings consistent with a small pleural effusion. Initial laboratory tests are significant for a platelet count of 96,000/mm3 (N 130,000–450,000), a positive antinuclear antibody test with a 1:80 titer (N <1:40), and negative Lyme disease and HIV tests.
Which one of the following would help confirm your suspected diagnosis?
A) Elevated anticyclic citrullinated peptide antibodies
B) Elevated anti–smooth muscle antibodies
C) Elevated anti-centromere antibodies
D) Low complement levels
E) Positive HLA-B27
ANSWER: D
The clinical findings on examination, including symmetric polyarthritis, thrombocytopenia, positive antinuclear antibodies (ANAs), and pleural effusion, meet the American College of Rheumatology criteria for a diagnosis of systemic lupus erythematosus (SLE). The rash and the patient’s neuropsychiatric history may also factor into the diagnosis, but they are not described specifically in this case. A positive ANA is sensitive but not specific for SLE. Although additional laboratory testing may not be needed to confirm SLE in this case due to classic clinical findings, low complement levels help confirm SLE and may be helpful because the ANA is only mildly elevated.
49
Elevated anticyclic citrullinated peptide antibodies help confirm a diagnosis of rheumatoid arthritis. Anti–smooth muscle antibodies are used to confirm autoimmune hepatitis, which can also cause an elevated ANA. Anti-centromere antibodies, a subset of ANAs, are more closely associated with systemic sclerosis. A positive HLA-B27 test is associated with the seronegative spondyloarthropathies, such as psoriatic arthritis.
A 36-year-old male sees you for follow-up of progressive fatigue and lightheadedness that has worsened over the past 3 months. He has lost 5 kg (11 lb) during this time. On examination he has a BMI of 21 kg/m2, a blood pressure of 88/48 mm Hg, and a pulse rate of 66 beats/min. A skin examination is notable for patches of nonpigmented skin on the hands. Initial laboratory testing is significant for a sodium level of 132 mEq/L (N 135–145) and a potassium level of 5.3 mEq/L (N 3.5–5.0).
Which one of the following tests would confirm the most likely diagnosis?
A) 17-hydroxyprogesterone
B) ACTH stimulation
C) Dexamethasone suppression
D) Late night salivary cortisol
E) Plasma renin and aldosterone
ANSWER: B
This clinical case is consistent with Addison’s disease, or adrenal insufficiency. This case is most likely the result of autoimmune disease, given the concurrent vitiligo, but it may also be idiopathic or secondary to cancers such as lymphoma or infections such as tuberculosis. Adrenal insufficiency is suggested by a low morning cortisol level, but the test of choice to confirm this diagnosis is the ACTH stimulation test.
17-Hydroxyprogesterone deficiency causes congenital adrenal hyperplasia, which typically presents in childhood. This test is part of newborn screening in the United States. Acquired 17-hydroxyprogesterone deficiency can present in adulthood as adrenal insufficiency but a low 17-hydroxyprogesterone level does not confirm adrenal insufficiency. Late night salivary cortisol is an initial test for corticosteroid excess (Cushing syndrome) and the dexamethasone suppression test is used to confirm that disorder. Renin and aldosterone levels can be helpful to characterize mineralocorticoid deficiency but they are not diagnostic.
A patient born in 1954 requests screening for hepatitis C. His anti-HCV antibody screen is positive.
Which one of the following would be most appropriate at this point?
A) A confirmatory second anti-HCV test in 4–6 weeks
B) A qualitative HCV RNA test
C) Quantitative HCV genotyping
D) Treatment with a dual antiviral regimen pending additional testing
E) Referral for a percutaneous liver biopsy
ANSWER: B
The CDC recommends that all adults 18 years of age or older receive a one-time screening for hepatitis C virus (HCV). Persons with risk factors for HCV exposure should be screened periodically, based on risk level. If the anti-HCV antibody screen is positive, then a qualitative HCV RNA test is the next step (SOR C). Prior to initiating treatment, a quantitative HCV RNA and genotype testing is necessary (SOR A). In addition, assessing the degree of fibrosis will provide information regarding the urgency of treatment. Percutaneous liver biopsy is generally the preferred evaluation after obtaining quantitative and genotype results, all of which can guide treatment decisions.
You are caring for an 84-year-old female with Alzheimer’s dementia. She is disoriented to time but usually oriented to person and place. She is homebound but is frequently visited by family. The patient’s daughter feels conflicted but wants her mother to go to a skilled care facility. The patient is adamant that she does not want to leave her home. During a recent home visit you noted that she smelled of urine and had a severe intertriginous rash suspicious for Candida. Her bedside Mini-Mental State Examination score was 24/30.
Which one of the following ethical principles should be the primary consideration when determining whether this patient should continue to live at home?
A) Autonomy B) Competence C) Harm
D) Justice E) Safety
ANSWER: A
Four widely recognized principles of medical ethics include respect for autonomy, beneficence, nonmaleficence, and justice. Respect for patient autonomy is central to medical ethics and to the doctor-patient relationship. Physicians should involve patients in health care decisions commensurate with the patient’s capacity to understand and make decisions. Even when a condition affects a patient’s decision-making capacity, the patient may be able to participate in some aspects of the decision-making process. Competence (or incompetence) is a legal term, which can be used to refer to a patient being granted autonomy or not. Harm and safety are not ethical principles, but are related to the principle of nonmaleficence, or “do no harm.” The principle of justice addresses the question of who ought to receive the benefits of an intervention and/or bear its burdens. An injustice occurs when a person who is entitled to a benefit is denied without good reason or when some burden is imposed unduly.
A 38-year-old male presents with a 2-month history of increased postprandial nausea and nonbloody, painless, loose stools. He feels well otherwise. Ten months ago he underwent bariatric surgery, which involved creating a small stomach pouch. There were no complications from the surgery and he does not take any medications. A review of his diet reveals that he has three small meals and two snacks daily. He has one serving of vegetables or bread with each meal. He has 16 oz of coffee with breakfast, 12 oz of soda with lunch, 12 oz of beer with dinner, and a cup of water or milk with snacks. His snack is usually cheese or peanut butter and a cracker. An examination is unremarkable.
You are concerned with his diet habits and your recommendations include
A) no fluid for 15 minutes before or after meals and snacks
B) limiting carbonated beverages to 8 oz per meal or snack
C) increasing daily servings of fibrous vegetables and whole grain breads
D) eating one meal and three snacks daily
E) eating a diet lower in fat
ANSWER: A
Hundreds of thousands of Americans have undergone bariatric surgery, and family physicians are often asked to provide long-term postoperative management. Many bariatric surgery procedures create a small stomach pouch. Dietary compliance is essential to minimize feeding intolerance symptoms such as postprandial nausea, emesis, and diarrhea. Post bariatric surgery diet recommendations typically include the following:
Ref:
*
* *
*
Avoid fluid 15 minutes before and after meals. Fluids with meals will move food more quickly through the pouch and decrease the feeling of fullness.
Avoid carbonated beverages entirely.
Eat three small protein-rich meals and one or two snacks daily. Lower fat diets are not typically recommended.
Whole grains and fibrous vegetables often exacerbate symptoms so there is no need to increase these foods.
A healthy 52-year-old male who runs for 40 minutes 4–5 times per week comes to your office because he has recently noted pain in the inner aspect of his right knee along the joint line. He also reports mild swelling and stiffness in his knee along with a “clicking” sensation when he flexes and extends it. In addition, he says that at unpredictable times he feels like his knee “might give way.” During the examination he reports pain along the medial joint line with palpation, and the McMurray test is positive. A subsequent MRI demonstrates a degenerative tear in the lateral portion of the medial meniscus with no significant degenerative arthritis. You begin to discuss a management plan with him, considering various surgical and nonsurgical options.
Given this patient’s age and clinical findings, which one of the following has been shown to produce better long-term outcomes than conservative management?
A) Intra-articular corticosteroids
B) Hylan GF 20 (Synvisc)
C) Arthroscopic meniscectomy
D) Arthroscopic meniscus repair
E) No additional treatment strategies
ANSWER: E
A systematic review found that arthroscopic procedures for degenerative meniscal tears in middle-aged adults with little or no arthritis do not significantly improve long-term pain or function compared to conservative management consisting of physical therapy and a standardized exercise program. No studies have compared conservative management to intra-articular injections of corticosteroids or hylan GF 20 for managing degenerative meniscal tears, either alone or when combined with conservative management.
n a patient on chronic warfarin (Coumadin) therapy who has a stable INR in the therapeutic range, which one of the following antibiotics would be most likely to elevate the INR?
A) Cephalexin (Keflex)
B) Clindamycin (Cleocin)
C) Penicillin G
D) Rifampin (Rifadin)
E) Trimethoprim/sulfamethoxazole (Bactrim)
ANSWER: E
Trimethoprim/sulfamethoxazole is one of the antimicrobials most likely to increase the INR of a patient taking warfarin. If trimethoprim/sulfamethoxazole is used in a patient on warfarin, reducing the warfarin dosage by 25%–40% is recommended, with close monitoring of the INR. The patient’s INR should be checked within 3–5 days of starting or stopping any antimicrobial.
55
First generation cephalosporins such as cephalexin, fourth generation cephalosporins, clindamycin, and penicillin G have a lower likelihood of affecting the INR. Rifampin decreases the INR and the warfarin dosage should be increased if rifampin is started. Other antimicrobials that significantly affect the INR include metronidazole and fluconazole. Azithromycin, ciprofloxacin, clarithromycin, and levofloxacin may impact the INR with a variable patient-specific effect.
A 6-month-old male is brought to your office by his parents for a well child check. He is their first child. On examination you note that he has two erupted teeth. The family lives in a town with fluoridated water and the parents would like to know how to care for their child’s teeth.
Which one of the following is the most effective preventive measure for this child’s newly erupted teeth?
A) No care is needed at this point
B) A parent wiping or brushing the teeth with water twice daily
C) A parent brushing the teeth with a smear of low-fluoride toothpaste twice daily
D) A fluoride supplement daily
E) Topical sealants
ANSWER: C
Early and consistent dental care in infants has been shown to reduce the rate of early childhood caries. The American Academy of Pediatric Dentistry recommends the use of low-fluoride toothpaste for tooth cleaning, starting with newly erupted teeth. According to the CDC, dental caries is one of the most prevalent chronic conditions among children in the United States. Dental caries can start soon after eruption of the first teeth. Brushing an infant’s teeth twice daily with a “smear” of fluoridated toothpaste is recommended. Fluoridated toothpaste has been found to be safe and effective for infants’ teeth. The use of fluoride in toothpaste is more effective for the prevention of caries than wiping or brushing the teeth with water. A daily fluoride supplement is not appropriate for this patient because the family’s local water source already contains fluoride. Topical sealants are intended for molar teeth, not incisors.
A 36-year-old male presents with a 6-week history of a mildly pruritic rash in his groin. An examination reveals small red-brown macules and larger patches with a sharp border. A Wood’s lamp examination reveals coral-red fluorescence.
Which one of the following would be the most appropriate treatment for this condition?
A) 0.1% triamcinolone cream
B) 2.5% hydrocortisone cream
C) Nystatin cream
D) Erythromycin gel
E) Fluconazole (Diflucan) orally
ANSWER: D
Erythrasma is a superficial infection caused by Corynebacterium minutissimum. It presents as small, red-brown macules that may coalesce into larger patches with sharp borders. It fluoresces coral red on Wood’s lamp examination. Cutaneous erythrasma is treated with erythromycin (topical, twice daily until the rash resolves, or oral, 250 mg four times daily for 2 weeks). Topical clindamycin, Whitfield ointment, and antibiotic soaps may also be beneficial. Triamcinolone cream, hydrocortisone cream, nystatin cream, and oral antifungals are not effective treatments for this bacterial infection.
A 34-year-old female comes to your office for follow-up after an emergency department visit because of anxiety. She notes persistent anxiety, poor focus, and palpitations. She also reports that she is not hungry and has lost several pounds. She reports “odd things happening” such as sudden weakness in her legs, falling, and getting lost.
When taking her history you note that the patient is hyperverbal and displays tangential speech. She has a temperature of 37.4°C (99.3°F), a heart rate of 134 beats/min, a respiratory rate of 20/min, and a blood pressure of 117/69 mm Hg. A physical examination reveals an anxious-appearing female who is tremulous at rest. A HEENT examination shows exophthalmos but no thyromegaly. A cardiac examination is unremarkable aside from tachycardia. A pulmonary examination reveals faint bibasilar crackles. An EKG shows sinus tachycardia. Laboratory results are as follows:
CBC withinnormallimits Basicmetabolicpanel withinnormallimits TSH <0.08 U/mL(N0.35–3.00) FreeT4 4.51ng/dL(N0.89–1.80) FreeT3 >19.0pg/dL(N2.3–4.2)
Which one of the following would be most appropriate at this point?
A) Start methimazole (Tapazole)
B) Check for thyroid receptor antibody
C) Obtain a radioactive iodine uptake scan
D) Refer her to endocrinology
E) Admit her to the hospital
ANSWER: E
Hyperthyroidism is a common condition with a generally favorable prognosis. However, it is important to remember that life-threatening complications such as thyrotoxicosis, also known as thyroid storm, can occur. Symptoms of thyroid storm include fever, central nervous system dysfunction, gastrointestinal or liver dysfunction, and cardiovascular complications such as tachycardia and heart failure. The diagnosis is made using the Burch-Wartofsky Point Scale, which produces a total score based on the presence or absence of various diagnostic criteria. In this case, the patient has a score of 45, which is highly suggestive of thyroid storm. This acute, life-threatening condition typically requires care in an intensive-care unit. It would therefore be inappropriate to start treatment with an agent such as methimazole prior to hospitalization. While a thyroid receptor antibody test may be useful in identifying the cause of the condition it should not delay hospitalization. A radioactive iodine uptake test is also useful for identifying the underlying cause of hyperthyroidism but should be avoided until the thyroid storm has resolved. This patient requires hospitalization, so a referral to endocrinology would not be most appropriate at this time.
A 58-year-old male with uncontrolled type 2 diabetes sees you for follow-up after a recent hospitalization for urosepsis treatment with intravenous antibiotic therapy. His hospital course was complicated by Clostridioides (Clostridium) difficile colitis, and he completed oral vancomycin (Vancocin) therapy 2 weeks ago. He was nearly back to his baseline but has had recurrent watery diarrhea for the past 3 days. You confirm a recurrent infection.
Which one of the following would be the most appropriate treatment?
A) Probiotics
B) Fidaxomicin (Dificid), 200 mg twice daily for 10 days
C) Metronidazole (Flagyl), 500 mg three times daily for 10 days
D) Vancomycin, 125 mg four times daily for 10 days
E) Fecal microbiota transplantation
ANSWER: B
This patient presents with his first recurrence of Clostridioides (Clostridium) difficile infection, which was previously treated with vancomycin. Initial episodes can be treated with vancomycin (strong recommendation, high quality of evidence), fidaxomicin (strong recommendation, high quality of evidence), or metronidazole if the other two treatments are unavailable (weak recommendation, high quality of evidence). However, fidaxomicin is recommended for recurrent infection if vancomycin was prescribed for the initial episode (weak recommendation, moderate quality of evidence). If available, a prolonged tapered course of vancomycin could be used if a 10-day course was prescribed initially (weak recommendation, low quality of evidence). Vancomycin is only recommended for a first recurrent episode if metronidazole was used initially (weak recommendation, low quality of evidence). Metronidazole is not recommended for recurrent episodes. Probiotic administration is not recommended due to insufficient data. Fecal microbiota transplantation is only recommended for a second or subsequent recurrent infection (strong recommendation, moderate quality of evidence).
A 48-year-old female presents with concerns about hair loss. She has noticed gradual thinning of the hair on the top of her head for the last year. Her scalp is now visible through the hair. She is not taking any oral medications. A levonorgestrel IUD (Mirena) was placed 4 years ago. She has not had any recent illnesses or stressors and a review of systems is negative. Her mother had similar hair loss starting in her fifties. On examination you note thin hair on the top of the scalp, an intact frontal hair line, and growth of thin, wispy hairs on her crown.
Which one of the following would be most appropriate for this patient?
A) Removal of the levonorgestrel IUD
B) Spironolactone (Aldactone), 50 mg daily for 6 months
C) Finasteride (Proscar), 5 mg daily indefinitely
D) Minoxidil (Rogaine) 2% solution for 6 months
E) Minoxidil 5% foam indefinitely
ANSWER: E
Hair thinning on the crown of the head with the presence of small, wispy hairs among the regular hair is characteristic of female pattern hair loss (FPHL). A family history of similar issues is often present but not necessary for the diagnosis.
Topical minoxidil is the mainstay of treatment for FPHL (SOR A). It is available in a 2% solution or 5% foam for women (the 5% solution is indicated only for men). Treatment for FPHL as well as the male equivalent, androgenic alopecia, must be continued long term. With treatment there is often an initial period of increased hair loss. Regrowth is noticeable around 6 months. Discontinuation of treatment results in loss of regrown hair.
There is no clear association between hormone status and FPHL. Removal of this patient’s levonorgestrel IUD is unlikely to affect hair loss. Spironolactone has also been used for FPHL but evidence is lacking regarding its effectiveness. Finasteride is approved by the FDA only for males with hair loss. There is a high risk of teratogenicity with its use. It has been used in women but evidence of efficacy is minimal.
A 38-year-old male comes to your office for follow-up of his diabetes mellitus. He takes metformin (Glucophage) and dulaglutide (Trulicity) and his hemoglobin A1c is 6.5%. He has an LDL-cholesterol level of 120 mg/dL, an HDL-cholesterol level of 55 mg/dL, and a triglyceride level of 190 mg/dL. He asks your advice about statin therapy for cholesterol management.
You advise him to start
A) a low-intensity statin now
B) a moderate-intensity statin now
C) statin therapy at 40 years of age
D) statin therapy when his 10-year atherosclerotic cardiovascular disease (ASCVD) risk is >5%
E) statin therapy when his 10-year ASCVD risk is >7.5%
ANSWER: C
There is high-quality evidence from randomized, controlled trials that a moderate-intensity statin should be initiated for all patients age 40–75 years with diabetes mellitus regardless of their calculated 10-year atherosclerotic cardiovascular disease (ASCVD) risk. There is not strong evidence supporting the use of statins before age 40 in patients with diabetes unless their LDL-cholesterol level is very high. The ASCVD risk score is valid for patients over 40 years of age and cannot be calculated before then.
A 46-year-old male who injects heroin daily presents with a 6-month history of progressive dyspnea on exertion, a productive cough, and fatigue. He does not have any fever, chills, malaise, or hemoptysis. He has not had any sick contacts and has never smoked. On physical examination he shows no signs of distress, has a normal oxygen saturation on room air, and has normal breath sounds. A chest radiograph reveals bilateral perihilar shadowing.
A subsequent lung biopsy will most likely show
A) adenocarcinoma
B) branching hyphae
C) foreign body granulomas
D) caseating granulomas
E) noncaseating granulomas
ANSWER: C
Although persons who inject drugs are at high risk for a variety of pulmonary infectious diseases, this patient’s presentation, including the relatively slow development of symptoms, is most consistent with pulmonary foreign body granulomas. These result from the injection of crushed pills, talc, or other foreign substances, which are then deposited in the vasculature of the lungs. Adenocarcinoma is not as likely given the patient’s age and nonsmoking history. Branching hyphae would be seen in aspergillosis but this patient does not have fevers or malaise. Caseating granulomas are seen in tuberculosis, which is less likely given the absence of fever and hemoptysis. Noncaseating granulomas, seen in sarcoidosis, would not be more likely in this patient than in the general population.
A 68-year-old female with a history of diabetes mellitus, hypertension, and heart failure with preserved ejection fraction presents with a long-standing diabetic foot ulcer. The patient reports no signs of any gastrointestinal bleeding, no blood in her urine, no bleeding gums, and no vaginal bleeding. Her medications include metformin (Glucophage), insulin glargine (Lantus), lisinopril (Prinivil, Zestril), atorvastatin (Lipitor), and furosemide (Lasix).
A CBC reveals a WBC count of 7600/mm3 (N 4500–11,000), a hemoglobin level of 9.7 g/dL (N 14.0–17.5), a mean corpuscular volume of 89 m3 (N 80–100), and a platelet count of 412,000/mm3 (N 150,000–400,000).
To further assess the patient’s anemia you obtain the following laboratory results:
Ferritin Serumiron Transferrin Reticulocytecount VitaminB12 Haptoglobin
293 ng/mL (N 22–275) 43 g/dL(N50–175) 190 mg/dL (N 177–264) 3.2%(N0.5–1.5) 564pg/mL(N230–1050) 198mg/dL(N63–273)
Which one of the following is the most likely cause of her anemia?
A) Anemia of chronic disease
B) Bone marrow suppression
C) Hemolysis
D) Iron deficiency
E) Vitamin B12 deficiency
ANSWER: A
This patient has findings most consistent with anemia of chronic disease, also known as anemia of inflammation. This condition is thought to be primarily a disorder of iron distribution in response to systemic inflammation, which also biases hematopoiesis toward myeloid cell production rather than erythropoiesis and shortens the erythrocyte lifespan. Anemia of chronic disease is a normocytic and normochromic anemia. Iron studies typically show evidence of iron restriction without systemic iron deficiency. A common challenge in diagnosis is when true iron deficiency coexists with anemia of chronic disease.
This patient’s normal WBC and platelet counts make bone marrow suppression less likely. The normal haptoglobin level and low reticulocyte count are not consistent with hemolysis. She has a normocytic rather than microcytic anemia and her ferritin level is elevated. These two factors make iron deficiency less likely despite her low serum iron level. The low normal transferrin level is also consistent with anemia of chronic disease rather than iron deficiency. Her normal vitamin B12 level makes a deficiency unlikely. Her history of a chronic foot ulcer and elevated inflammatory markers (ferritin and platelets) are consistent with anemia of chronic disease.
A 60-year-old male presents with left lower quadrant abdominal pain. His medical and surgical histories are remarkable only for a history of hypertension controlled with hydrochlorothiazide and lisinopril (Prinivil, Zestril), and no polyps seen on screening colonoscopy 5 years ago. He is afebrile, and a physical examination is notable only for mild abdominal tenderness in the left lower quadrant without peritoneal signs. A urinalysis is normal. You diagnose mild diverticulitis.
Which one of the following management options would be indicated at this time?
A) Rest and clear liquids
B) Avoidance of seeds, nuts, and popcorn
C) Abdominal CT
D) Referral for colonoscopy
E) Hospital admission for intravenous fluids and intravenous antibiotics
ANSWER: A
In patients with mild diverticulitis, outpatient management with rest and oral fluids is preferred. Avoidance of seeds, nuts, and popcorn does not reduce recurrence rates. CT of the abdomen may be indicated if the diagnosis is uncertain or if complications are suspected. Colonoscopy is contraindicated acutely and is only necessary for follow-up when age-appropriate cancer screening is indicated, or in cases of complicated disease. Antibiotics may not be necessary in all cases, and hospital admission is unnecessary for mild cases.
A 60-year-old retired dock worker presents to your office with chronic low back pain due to multiple herniated lumbar discs, with radicular pain down both legs. He rates his pain as a 3 out of 10. He currently takes oxycodone (OxyContin), 10 mg every 12 hours; pregabalin (Lyrica), 150 mg every 12 hours; acetaminophen, 1000 mg every 8 hours; meloxicam (Mobic), 15 mg daily; and cyclobenzaprine, 10 mg every 8 hours. These medications have been prescribed by another physician for the past 5 years. He tells you that his pain is tolerable but his sex drive and energy level have steadily decreased since starting these medications.
Which one of the following medications in this patient’s regimen would be most likely to decrease his libido?
A) Acetaminophen B) Cyclobenzaprine C) Meloxicam
D) Oxycodone E) Pregabalin
ANSWER: D
Opioid use may cause numerous adverse reactions, including drowsiness, pruritus, nausea, and constipation. In addition to these well known side effects, chronic opioid use can lead to hypogonadism through inhibition of gonadotropin-releasing hormone and an increase in prolactin. A recently published study found that long-term opioid users were nearly twice as likely to be diagnosed with hypogonadism as short-term opioid users. Given the large number of opioid users in the United States, prescribers should be aware of this adverse effect and screen for hypogonadism when appropriate. None of the other medications in this patient’s regimen, including acetaminophen, cyclobenzaprine, meloxicam, and pregabalin, are the likely cause of his decreased libido.
A 45-year-old female sees you for follow-up after an emergency department visit in which CT of the abdomen and pelvis was performed to detect kidney stones. Kidney stones were not seen and her flank pain was determined to be musculoskeletal in origin. She is feeling better now. However, the CT showed a simple-appearing 5.2-cm ovarian cyst with assessment limited by artifact. She does not have any symptoms, pelvic pain, bloating, fevers, night sweats, or unintentional weight loss. There is no family history of ovarian or breast cancer. She reports regular menstrual cycles.
Which one of the following would be most appropriate at this point?
A) Reassurance only
B) A CA-125 level
C) Ultrasonography of the pelvis
D) MRI of the pelvis
E) Referral to a gynecologic oncologist
ANSWER: C
Ovarian incidentalomas are very common, and appropriate management depends upon the size and appearance of the incidentaloma as well as the menopausal status of the patient. The Society of Radiologists in Ultrasound states that simple cysts 5 cm in premenopausal women and simple cysts 3 cm in postmenopausal women are considered normal and do not require follow-up. The American College of Radiology recommends that immediate ultrasonography be performed in the evaluation of simple-appearing cysts that are incompletely characterized by CT and are >5 cm in premenopausal women or >3 cm in postmenopausal women. This patient’s cyst appears benign, but further evaluation is recommended due to the large size. CA-125 levels have low sensitivity and specificity in premenopausal women and would not be indicated in this case. The initial imaging of choice is pelvic ultrasonography, so MRI of the pelvis is not necessary. Referral to a gynecologic oncologist is not indicated because there is no current evidence to suggest malignancy
A 56-year-old male comes to your office because of right shoulder pain for the past week. It started when he lifted a heavy piece of furniture while helping a friend move. He felt immediate sharp pain in his shoulder, which has since radiated down the biceps toward the right radial forearm. On examination there is no deformity of the shoulder or arm. He has increased pain with palpation in the anterior shoulder near the bicipital groove of the humerus.
You suspect biceps tendinitis. Anterior shoulder pain with which one of the following examination maneuvers of the right arm would be most consistent with this diagnosis?
A) Active or passive cross adduction of the arm at the shoulder
B) Shooting pain to the thumb with axial compression of the head with the neck flexed
toward the right shoulder
C) Resisted extension of the elbow with the shoulder in a neutral position
D) Resisted internal rotation of the shoulder with the elbow flexed to 90°
E) Resisted supination of the hand with the elbow flexed to 90°
ANSWER: E
This patient’s symptoms are consistent with bicipital tendinitis, which causes pain with abduction and external rotation of the arm, and tenderness of the bicipital groove with palpation. Resisted supination of the hand with the elbow flexed to 90° is the Yergason test, and anterior shoulder pain with this maneuver is consistent with bicipital tendinitis. Anterior shoulder pain with cross adduction of the arm is more consistent with acromioclavicular arthritis. Axial compression of the slightly flexed neck is the Spurling test for cervical radiculopathy. Extension of the elbow would activate the triceps, and internal rotation of the shoulder with the elbow flexed would result in less activation of the biceps than resisted supination.
Ref: Churgay CA: Diagnosis and treatment of biceps tendinitis and tendinosis. Am Fam Physician 2009;80(5):470-476. 2) Jameson JL, Fauci AS, Kasper DL, et al (eds): Harrison’s Principles of Internal Medicine, ed 20. McGraw-Hill, 2018, p 2647.
A 35-year-old male first presented to your office 4 months ago with a persistent chronic cough. He is a nonsmoker with no significant past medical history. Over the past few months he has been evaluated for GERD, asthma, eosinophilic bronchitis, and upper airway cough syndrome without symptomatic relief or diagnosis.
Which one of the following is recommended for chronic refractory cough in this otherwise healthy male?
A) Cyclobenzaprine
B) Duloxetine (Cymbalta)
C) Gabapentin (Neurontin)
D) Lorazepam (Ativan)
E) Propranolol
ANSWER: C
In randomized, controlled trials, gabapentin has demonstrated benefit for treating a refractory chronic cough after 4 weeks of treatment (SOR C). Chronic cough may be due to a hypersensitivity of the cough reflex, either centrally or peripherally. Cyclobenzaprine, duloxetine, lorazepam, and propranolol have not proven to be beneficial in reducing or eliminating chronic cough.
A 36-year-old female presents to your office with a 24-hour history of redness in her right eye. It is associated with mild pain but no drainage. On examination her visual acuity is 20/20 bilaterally, her pupillary reflex is normal, extraocular movements are intact, and there is no discharge noted. There is a focal area of hyperemia of the episcleral blood vessels noted along the medial aspect of the eye. Fluorescein staining is normal.
This patient’s presentation is most consistent with which one of the following?
A) Bacterial conjunctivitis
B) Viral conjunctivitis
C) Episcleritis
D) Iritis
E) Keratitis
ANSWER: C
An acute red eye is a common presentation in primary care and it is critical to differentiate serious causes from benign causes. Episcleritis is a self-limited condition that can be idiopathic and presents with mild discomfort and focal hyperemia. Conjunctivitis is typically associated with a discharge that is clear in viral cases and mucopurulent in bacterial cases. Iritis is associated with significant pain, a poorly reactive pupil, diminished vision, and photophobia. This patient does not have changes in visual acuity, photophobia, or severe pain as seen in keratitis, which would also cause an abnormal fluorescein stain showing corneal ulceration.